[obm-l] ENQUETE - BELEZA MATEMTICA

2003-08-14 Por tôpico Cludio \(Prtica\)
Title: Help



Caros colegas:

Aqui vai a minha lista. Acredito que se tivesse encontrado os resultados l 
mencionados durante o meu 2o. grau, talvez tivesse decidido cursar matemtica e 
no engenharia, como acabei fazendo.

1. O princpio das Casas de Pombos,
no pelo princpio em si, que  altamente intuitivo, mas pelas 
consequncias surpreendentes:
i) Em todo grupo de 6 pessoas, existem 3 que se conhecem mutuamente ou 3 
que se desconhecem mutuamente - este  o ponto de partida pra teoria de Ramsey, 
que ainda tem inmeros problemas em aberto;
ii) Se um paciente tem que tomar 48 plulas em 30 dias, sendo que ele toma 
pelo menos uma plula por dia, ento existe uma sequncia de dias 
consecutivosnos quaisele toma exatamente 11 plulas - este problema 
no-trivial foi o que chamou a minha ateno para o PCP;
iii) Toda sequncia de nmeros reais possui uma subsequncia 
montona- uma bela aplicao  Anlise;
iv) Se a  irracional, ento o conjunto A = {m + n*a; m, n inteiros}  
denso em R - resultado interessante por si s, mas que  o "pulo do gato" pra se 
provar o inusitado
v)Dada uma sequnciaqualquer de algarismos, existe uma potncia 
de 2 cuja representao decimal comea com aquela sequncia.

2. O teorema de Bezout: "Se a e b so dois inteiros quaisquer, ento 
mdc(a,b)  o menor inteiro positivo que pode ser expresso na forma a*x + b*y, 
com x e y inteiros",
por ser uma das primeiras aplicaes no-triviais do axioma da Boa 
Ordenao e ser usado pra provar que:
i) Se a e b so inteiros primos entre si e se a divide b*c (c inteiro), 
ento a divide c;
ii) Se p  primo, ento cada inteiro primo com p tem um inverso (mod 
p);
Estes dois resultados, por sua vez, so usados pra provar:
iii) O pequeno teorema de Fermat;
iv) O teorema de Wilson;
Os quais, juntamente com o PCP, provam o sensacional:

3.Todo primo da forma 4k+1  soma de dois quadrados.
Sem comentrios. Se isso no  bonito, ento o que ?

4. N  perfeito par == N = 2^(p-1)*(2^p - 1), onde 2^p - 1 
 primo,
pela sacada simples mas brilhante de Euler. Como  que os gregos no viram 
essa?

5. O caso n = 4 do ltimo Teorema de Fermat,
por ser um belo exemplo de aplicao da "descida infinita" (uma variante do 
axioma da Boa Ordenao) e porpressupor um conhecimento da bela teoria 
sobre osternos Pitagricos.

6. Postulado de Bertrand: "Se x  1, ento existe (pelo menos) um primo 
entre x e 2x",
por ser inusitado, e ter uma demonstraao que, apesar de meio longa,  
muito engenhosa e totalmente elementar.

7. Teoremas sobre cardinalidade de conjuntos infinitos, tais como:
card(N) = card(Q),
card(N)  card(R),
card(R) = card(R^2),
card(X)  card(Partes(X)), onde X  um conjunto qualquer,
card(Partes(N)) = card(R),
por serem surpreendentes para quem os v pela primeira vez e pela 
engenhosidade das demonstraes, especialmente o mtodo da diagonal.

8. A existncia e unicidade dos 5 poliedros regulares,
no s pela beleza mas pela importncia histrica, pois foi o teorema que 
Euclides escolheu pra encerrar os seus Elementos.

9. A desigualdade do rearranjo,
pela demonstrao essencialmente combinatria, por nose aplicar 
apenas a nmeros positivos,e por implicar numa srie de outras 
desigualdades, inclusive a das mdias geomtrica e aritmtica (apesar de 
existirem demonstraes mais simples desta ltima).

10. A desigualdade isoperimtrica,
pela beleza do encadeamento lgico - passo a passo - da demonstrao.

*

Pro Frederico: Sobre o TNP, eu tambm acho a relao entre primos e 
logaritmos altamente surpreendente, mas os pr-requisitos para que um aluno 
normal de 2o. grau entenda quo especiais so os logaritmos naturais e o nmero 
"e" talvez sejam pesados demais. Assim, eu achei melhor no incluir nem mesmo as 
desigualdades de Chebichev envolvendo n/ln(n).

Por outro lado, um que quase inclu foi o fato de que a srie dos 
recprocos dos primos  divergente.

Tambm fiquei muito tentado a incluir os interessantssimos teoremas de 
Desargues e Pappusda geometria projetiva, mas no fundo eu gosto mais de 
lgebra e teoria dos nmeros...


Um abrao,
Claudio.





[obm-l] problemas

2003-08-14 Por tôpico elton francisco ferreira

O número de anagramas formados com as letras da
palavra ROMA de modo que não apareça vogais ou
consoantes juntas é igual a:

4!
4
8
2

um atirador deu 49 tiros, pagando 10 de multa por tiro
fora do alvo e recebendo 0,25 de premio pr tiro
acertado no alvo. Se nada recebeu e nada pagou, então
a multa foi de:

1,40
3,50
5,00
8,75

Se de um retangulo de perímetro 4 e dimensões ``x`` e
``y`` , XY, retira-se um quadrado de lado ``x``,
então a área remanescente em função de ``x`` é:

1 – 2x
x – 2x^2
2x – 2x^2
2x – 4x^2

Uma das raízes da equação 2x^3 + x^2 – 7x – 6 = 0  é 
x1= 2. Pode-se afirmar que:

As outras raízes são imaginário puro
As outras raízes são –3 e –2
Só uma das outras raízes é real
As outras raízes estão entre –2 e 0


___
Conheça o novo Cadê? - Mais rápido, mais fácil e mais preciso.
Toda a web, 42 milhões de páginas brasileiras e nova busca por imagens!
http://www.cade.com.br
=
Instruções para entrar na lista, sair da lista e usar a lista em
http://www.mat.puc-rio.br/~nicolau/olimp/obm-l.html
=


[obm-l] ITA-96

2003-08-14 Por tôpico Jorge Paulino
Tô com dificuldades na resoluçao.
um abraço,
Jorge

___
Conheça o novo Cadê? - Mais rápido, mais fácil e mais preciso.
Toda a web, 42 milhões de páginas brasileiras e nova busca por imagens!
http://www.cade.com.br

ITA-ANEXO.zip
Description: ITA-ANEXO.zip


[obm-l] Re: [obm-l] Questes Esaex - RETA FINAL 4 e 5

2003-08-14 Por tôpico J Augusto Tavares





  
  1) 
  Seja:
  Z1 = 
  2ei/6 
  Z2 = (1 + i/1  i)15 
  ln Z3 = 1 + i(/3 + 2k)
  
  Ento o 
  valor de (Z1 + Z2)/e . Z3 :
  R: minha resposta deu diferente da do fabio ... como ninguem 
  falou nada ate agora ! a minha deve ta errada!
  
  
  4) NO CONSIGO 
  FATORAR A RAIZ CBICA!
  O valor do 
  limite quando x  0 de
   
  4(x+1) + 3(x+1) + (x+1)  
  3
   
  (x+1)  1
  
  
  Resposta: 
   Fazendo (x+1) = y^12 , como 
  x-0, y-1.
  (y^3 + y^4 + y^6 - 3)/(y^6 - 1) ,[(y^3 - 
  1) + (y^4 -1) + (y^6 - 1)]/[(y^3 + 1)(y^3 -1)]
  eleminando o fator (y-1),nao existira mais 
  a indeterminacao ! 
  
  
  
  5) 
  Determinar lim ln (1 + 
  _2_ )2x  1/3 
   
  n 
  x+3
  
  Resposta:
  n-
  ln[lim (1 + 2/(x+3))^(2x - 
  1/3)], lim (1 + 2/(x+3))^(x+3)*[(2x -1/3)/(x+3)  Elevando a 
  (x+3)/(x+3)
  temos o limite 
  fundamental{ n-} (1 + k/x)^x = e^k
  (e^2)^lim(2x -1/3)/(x+3) 
  = (e^2)^2, logo ln e^4 eh 4.
  
  
  


Re: [obm-l] Re: [obm-l]_Re:_[obm-l]_ENQUETE_-_BELEZA_MATEMÁTICA

2003-08-14 Por tôpico Johann Peter Gustav Lejeune Dirichlet
A demonstraçao que 3eu escrevi evitara este
mal-entendido.
Alias o Tengan me disse que este e um problema em
aberto muito chato e de que ninguem conseguiu uma
ideia muito esperançosa...

 --- Nicolau C. Saldanha
[EMAIL PROTECTED] escreveu:  On
Wed, Aug 13, 2003 at 09:39:49PM -0200,
 Claudio Buffara wrote:
  on 13.08.03 20:28, Henrique Patrício
 Sant'Anna Branco at
  [EMAIL PROTECTED] wrote:
  
   Cláudio,
   
   A classica prova de Euclides é aquela que
 diz:
   Sejam p1, p2, ..., pm todos os primos.
 Entao consideremos o número N = p1 *
   p2 * ... * pm + 1. Esse número não seria
 divisível por nenhum primo e,
   portanto, contradiz o Teorema Fundamental
 da Aritmetica?
   
   Abraços,
   Henrique.
  
  Eh isso ai mesmo.
 
 Por falar nisso, esta prova aparentemente induz
 um dos erros mais comuns.
 As pessoas incorretamente entendem que foi
 provado que 2*3*...*p + 1 é primo.
 Isto é falso mas o primeiro contraexemplo
 demora o suficiente para aparecer
 para convencer os mais afoitos de que sim,
 estes números são primos:
 2*3*5*7*11*13+1 = 30031 = 59*509
 2*3*5*7*11*13*17+1 = 510511 = 19*97*277
 2*3*5*7*11*13*17*19+1 = 9699691 = 347*27953
 2*3*5*7*11*13*17*19*23+1 = 223092871 =
 317*703763
 Também dá errado se trocarmos +1 por -1 no
 final:
 2*3*5*7-1 = 209 = 11*19
 2*3*5*7*11*13*17-1 = 510509 = 61*8369
 2*3*5*7*11*13*17*19-1 = 9699689 = 53*197*929
 2*3*5*7*11*13*17*19*23-1 = 223092869 =
 37*131*46027
 
 Desculpem, eu sei que ninguém perguntou, mas eu
 já ouvi este erro
 vezes demais.
 
 []s, N.
 
 

=
 Instruções para entrar na lista, sair da lista
 e usar a lista em

http://www.mat.puc-rio.br/~nicolau/olimp/obm-l.html

= 

___
Conheça o novo Cadê? - Mais rápido, mais fácil e mais preciso.
Toda a web, 42 milhões de páginas brasileiras e nova busca por imagens!
http://www.cade.com.br
=
Instruções para entrar na lista, sair da lista e usar a lista em
http://www.mat.puc-rio.br/~nicolau/olimp/obm-l.html
=


Re: [obm-l] Re: not subject

2003-08-14 Por tôpico Nicolau C. Saldanha
On Wed, Aug 13, 2003 at 07:06:06PM -0300, Eduardo Casagrande Stabel wrote:
 Olá Dirichlet,
 
 eu também pensei sobre o problema: demonstrar que não existe uma função nos
 reais contínua nos racionais e somente neles. Sequer tenho alguma estratégia
 ou alguma idéia de como atacar o problema. Será que alguém pode dar uma
 sugestão? O único progresso que fiz - que nem sei se está certo - é intuir
 que os racionais não são um conjunto tão especial neste enunciado, eu
 suspeito que podemos substituir por enumeraveis densos nos reais.
 
 Quem quiser fazer comentários, sinta-se à vontade.

Isto é essencialmente um corolário do teorema de Baire.

Se a função é descontínua em um ponto x então existe um n tal que para todo
delta  0 exitem x1, x2, |x - x1|  delta, |x - x2|  delta,
com |f(x1) - f(x2)| = 1/n. Seja Xn o conjunto dos x que satisfazem esta
condição (para n dado). Prove que Xn é fechado. Se f é contínua nos racionais
prove que Xn tem interior vazio. A união de todos os Xn e de todos os conjuntos
da forma {x} com x racional não pode ser igual a R.

A sua intuição está certíssima. Não apenas a mesma prova se aplica mas
dados dois subconjuntos Y1 e Y2 enumeráveis densos de R, existe uma
bijeção crescente (portanto contínua e com inversa contínua) g com g(Y1) = Y2.

[]s, N.
=
Instruções para entrar na lista, sair da lista e usar a lista em
http://www.mat.puc-rio.br/~nicolau/olimp/obm-l.html
=


Re: [obm-l] Re:_[obm-l]_ENQUETE_-_BELEZA_MATEMÁTICA

2003-08-14 Por tôpico Johann Peter Gustav Lejeune Dirichlet
Na verdade esta prova nao e a original de
Euclides mas e muito proxima no sentido de uma
demonstraçao facil e bonita para um bom compilado
de matematica antiga.
Dado um conjunto qualquer finito de primos e
possivel achar um primo que nao esteja no
conjunto.
Sejam p1,p2,p3,...,pn os tais primos do conjunto
dado.Considere o numero 1+p1*p2*p3*...*pn.Se este
cara for primo acabou.Caso contrario ele pode ser
fatorado. Esse número nao e
divisivel por nenhum primo deste conjunto,
portanto,seus divisores primos estao fora do
conjunto.E fim!

 --- Henrique_Patrício_Sant'Anna_Branco
[EMAIL PROTECTED] escreveu:  Cláudio,
 
 A classica prova de Euclides é aquela que diz:
 Sejam p1, p2, ..., pm todos os primos. Entao
 consideremos o número N = p1 *
 p2 * ... * pm + 1. Esse número não seria
 divisível por nenhum primo e,
 portanto, contradiz o Teorema Fundamental da
 Aritmetica?
 
 Abraços,
 Henrique.
 
 - Original Message - 
 From: Claudio Buffara
 [EMAIL PROTECTED]
 To: [EMAIL PROTECTED]
 Sent: Wednesday, August 13, 2003 6:08 PM
 Subject: Re: [obm-l] Re: [obm-l] ENQUETE -
 BELEZA MATEMÁTICA
 
 
 Eu vou dar mais um tempinho pra ver se alguem
 mais quer enviar sua lista e
 ai publico os enunciados, talvez ateh separados
 por assunto, como no Proofs
 from the Book. Jah as demonstracoes sao outra
 historia - e voluntarios serao
 muito bem-vindos. Eu me disponho a provar que
 ha infinitos primos (ou pelo
 menos mais de 200, o que jah eh um belo
 resultado parcial). Que tal?
 

=
 Instruções para entrar na lista, sair da lista
 e usar a lista em

http://www.mat.puc-rio.br/~nicolau/olimp/obm-l.html

===
 

___
Conheça o novo Cadê? - Mais rápido, mais fácil e mais preciso.
Toda a web, 42 milhões de páginas brasileiras e nova busca por imagens!
http://www.cade.com.br
=
Instruções para entrar na lista, sair da lista e usar a lista em
http://www.mat.puc-rio.br/~nicolau/olimp/obm-l.html
=


Re: [obm-l]

2003-08-14 Por tôpico Nicolau C. Saldanha
On Thu, Aug 14, 2003 at 03:08:34AM -0300, [EMAIL PROTECTED] wrote:
 OLÁ AMIGOS. URGENTEE 
 TEM UM TAL CARA COM O E-MAIL DE (CLAUDIO PRATICA) QUE ESTAR MADANDO VÍRUS 
 PARA A LISTA DE MATEMATICA. ESSE ÍNDIVIDUO JÁ MANDOU 2 VEZES PARA O MEU 
 PC! 

O Claudio já foi avisado. Por favor SEMPRE tratem este tipo de assunto
por fora da lista.

[]s, N.
=
Instruções para entrar na lista, sair da lista e usar a lista em
http://www.mat.puc-rio.br/~nicolau/olimp/obm-l.html
=


[obm-l] [univ] Teoria dos Grupos

2003-08-14 Por tôpico Eduardo Casagrande Stabel
Olá pessoal!

[Agradeço ao Nicolau pela solução enviada... teorema de Baire era o mais
natural...]

Uma questão de álgebra que não estou conseguindo resolver, do livro de
introdução a álgebra do Hernstein.

QUESTÃO. Um grupo abeliano finito possui dois subgrupos, um de ordem N e
outro de ordem M. Mostre que ele trambém possui um subgrupo de ordem MMC{ M,
N }.

Eu consigui resolver a questão no caso particular em que o grupo é cíclico.
No caso geral, eu pensei em usar o produto de subgrupos MN, mas a ordem pode
ser maior que MMC{ M, N }. O interessante é que unindo esta questão ao
teorema de Silow para grupos abelianos, acho que se demonstra a existência
de subgrupos de qualquer ordem divisora da ordem do grupo original. Este
resultado não é forte demais?

Agradeço pela ajuda!
Abração,
Duda.


=
Instruções para entrar na lista, sair da lista e usar a lista em
http://www.mat.puc-rio.br/~nicolau/olimp/obm-l.html
=


[obm-l] Produto de Cossenos

2003-08-14 Por tôpico Claudio Buffara
Thyago:

Para um produto de senos de numeros em PA, eu acho que a sua solucao eh a
melhor.

No entanto, se o produto for de cossenos de numeros em PG da razao 2, ai a
coisa muda de figura...

P = cos(a)cos(2a)cos(4a)...cos(2^na) ==

sen(a)P = sen(a)cos(a)cos(2a)cos(4a)...cos(2^na) =

= (1/2)sen(2a)cos(2a)cos(4a)...cos(2^na) =

= (1/4)sen(4a)cos(4a)...cos(2^na) =

= (1/8)sen(8a)cos(8a)...cos(2^na) =

...

= (1/2^n)sen(2^na)cos(2^na) =

= (1/2^(n+1))sen(2^(n+1)a)

Logo: P = sen(2^(n+1)a)/(2^(n+1)sen(a))


Serah que era esse o problema do Lidski que voce procurava?


Um abraco,
Claudio.

on 12.08.03 21:07, Thyago at [EMAIL PROTECTED] wrote:

 Olá Claudio e companheiros da lista
 
 Bom, sabe que estou me convencendo mesmo que esta solução é prática :-)
 
 O que eu estava querendo inicialmente é uma solução que nem a da questão
 abaixo, veja só:
 
 S = sen(a) + sen(2a) + sen(3a) + ... + sen(na)
 
 Em que a solução consiste em multiplicar ambos os lados da igualdade pelo
 seno da metade da razão da PA, e após efetuar a prostaférese e sair
 cortando. Sem muitas delongas!
 ...
 
 Já ouvi dizer que a resolução que procuro existe, e está escrita em um tal
 livro russo chamado Lidski, problemas de PA, ou algo do gênero... mas
 nunca tive o privilégio de ter algum contato com essa obra. Alguém já ouviu
 falar?
 
 Atenciosamente
 ¡Thyago!
 
 
 
 - Original Message -
 From: Claudio Buffara [EMAIL PROTECTED]
 To: [EMAIL PROTECTED]
 Sent: Tuesday, August 12, 2003 9:58 AM
 Subject: Re: [obm-l] Ajuda
 
 
 Oi, Thyago:
 
 Vou te confessar uma coisa: usando a identidade 1 - cis(a) =
 -2isen(a/2)cis(a/2) e mais esse problema do IME, que alias eh uma
 propriedade classica (e, como voce mostrou, util!) das raizes n-esimas da
 unidade, voce chegou a uma solucao mais curta e elegante do que a que eu
 tinha em mente. Parabens!
 
 A minha ideia era separar os casos n par e n impar e fatorar x^n - 1 de
 duas
 maneiras diferentes:
 Primeiro:
 x^(2m) - 1 = (x^2 - 1)*(x^(2m-2) + x^(2m-4) + ... + x^4 + x^2 + 1)
 x^(2m+1) - 1 = (x - 1)*(x^(2m) + x^(2m-1) + ... + x^2 + x + 1)
 
 Depois:
 x^(2m) - 1 = (x^2 - 1)*PRODUTO(1=k=m-1)(x^2 - 2xcos(kpi/m)x + 1)
 x^(2m+1) - 1 = (x - 1)*PRODUTO(1=k=m)(x^2 - 2xcos(2kpi/(2m+1)) + 1)
 
 E depois, fazer x = 1 e igualar as expressoes obtidas, mas a sua solucao
 eh
 mais simples e, portanto, melhor.
 
 O passo que faltou na sua solucao foi mostrar explicitamente que
 (-i)^(m-1)*cis(pi/n)*cis(2pi/n)*...*cis((n-1)pi/n) = 1
 mas isso eh bem facil (apesar de nao ser evidente).
 
 Um abraco,
 Claudio.
 
 PS: Se essa sua solucao nao eh pratica, entao eu nao sei o que eh.
 Repare:
 voce tem um produto de senos de numeros em PA. Como voce propoe
 calcula-los?
 Puramento por meio de identidades trigonometricas, sem usar complexos? Boa
 sorte...
 
 on 12.08.03 00:45, Thyago at [EMAIL PROTECTED] wrote:
 
 Olá Cláudio,
 
 Obrigado pelas dicas  :-)
 
 Mas a resolução que eu fiz não foi nada prática não.
 Eu já utilizei todas estas propriedades e não consegui chegar em nada.
 Bom, só para esclarecer um pouco mais... vou colocar o exercício que
 gerou
 tal questão:
 
 
 (IME) Sejam 1, X2, X3, ..., Xn as raízes de x^n=1. Calcule: P = (1 -
 x2)(1-x3)...(1-xn).
 
 Fazendo uso de Briot-Rufini e fatoração de polinômios, conseguimos
 chegar
 facilmente na resposta P = n.
 Mas, utilizando o tratamento vetorial de números complexos com a fórmula
 1-cis(a) = -2isen(a/2)cis(a/2) chegamos em
 
 P = 2^(n-1) . S
 
 Onde S = sen(pi/n) . sen(2pi/n) . sen(3pi/n) . ... . sen[(n-1)pi/n]
 
 Daí, utilizando a resposta da primeira resolução com a resposta da
 segunda
 resolução temos que S = n/[2^(n-1) ]
 Dá para ver que esta demonstração para S não é nada prática.
 
 Você citou uma solução padrão para este tipo de problema. Qual seria?
 
 Aguardo resposta
 
 Atenciosamente
 ¡Thyago!
 
 - Original Message -
 From: Cláudio (Prática) [EMAIL PROTECTED]
 To: [EMAIL PROTECTED]
 Sent: Monday, August 11, 2003 2:19 PM
 Subject: Re: [obm-l] Ajuda
 
 
 Oi, Thyago:
 
 A solução padrão pra esse tipo de problema realmente envolve
 complexos e
 polinômios.
 
 Tentando resolver outros problemas similares, você vai perceber que
 complexos e polinômios são uma forma de resolução bastante natural.
 
 Os resultados básicos são os seguintes:
 1) Todo número complexo pode ser representado na forma R*(cos(a) +
 i*sen(a)), onde R é um real não negativo e a é um real qualquer
 (mas
 normalmente limitado ao intervalo [0, 2pi) ou então (-pi,pi]);
 2) e^(i*a) = cos(a) + i*sen(a): essa é a definição da função
 exponencial
 complexa, que permite, por exemplo, que você transforme sequências de
 senos
 e cossenos de números reais em PA em sequências de complexos em PG, que
 as
 vezes são mais fáceis de manipular;
 3) Um polinômio com coeficientes reais pode ser expresso como o produto
 de
 binômios da forma (x - b) e/ou trinômios da forma (x^2 - 2*R*cos(a)*x +
 R^2), onde a e b são números reais quaisquer e R é um real positivo.
 
 Um abraço,
 Claudio.
 
 
 

[obm-l] triangulo ortico

2003-08-14 Por tôpico Johann Peter Gustav Lejeune Dirichlet
Oi turma,alguem consegue dar uma demonstraçao de
que o triangulo pedal do ortocentro e o triangulo
inscrito de menor perimetro possivel?

___
Conheça o novo Cadê? - Mais rápido, mais fácil e mais preciso.
Toda a web, 42 milhões de páginas brasileiras e nova busca por imagens!
http://www.cade.com.br
=
Instruções para entrar na lista, sair da lista e usar a lista em
http://www.mat.puc-rio.br/~nicolau/olimp/obm-l.html
=


Re: [obm-l] ENQUETE - BELEZA MATEMATICA

2003-08-14 Por tôpico Nicolau C. Saldanha
On Mon, Aug 11, 2003 at 07:53:06PM -0300, Claudio Buffara wrote:
 Por mais que eu ache pedante e ridiculo alguem se vangloriar de ter o QI
 mais alto do mundo, nesse caso acho que a Marilyn estah certa. Voce deve
 trocar de porta.
 
 Desculpem a minha ignorancia, mas o que ha de errado com o argumento de 1
 milhao de portas? Me parece que, nesse caso, a probabilidade de voce ter
 escolhido a porta certa de primeira eh apenas de 1/1.000.000. Logo, a
 probabilidade da outra porta ter o premio eh de 999.999/1.000.000. Ou nao?

A Marilyn está certíssima e sua argumentação é perfeita (neste caso).
Você também está certíssimo. Eu já escrevi um artigo na Eureka sobre
este problema (entre outros). 

[]s, N.
=
Instruções para entrar na lista, sair da lista e usar a lista em
http://www.mat.puc-rio.br/~nicolau/olimp/obm-l.html
=


Re: [obm-l] Re: [obm-l] ENQUETE - BELEZA MATEM ÁTICA

2003-08-14 Por tôpico Johann Peter Gustav Lejeune Dirichlet
Eiu vou colocar a minha demonstraçao preferida da
infinitude dos primos:

Seja F(t)-1=2^2^t,na notaçao de Knuth para
potenciaçao.
Seja 1d um certo natural. 
Se d divide F(a) e d divide F(b) com ab,entao
2^2^a=-1(d).Elevando a 2^(b-a),que e par pois   
b-a=1, obtemos 2^2^b=1(d).Mas 2^2^b=-1(d),e
portanto d divide 2.Mas d nao pode ser 1 pele
restriçao inicial,e nao pode ser 2 pois 2 nao
divide F(x) pois F e impar.
Logo dois numeros de Fermat sao primos entre si
se e somente se forem diferentes.
E ai e facil ver que os primos sao
infinitos:fatore todos os possiveis numeros de
Fermat.Os primos nao devem se repetir,pelo que
acabamos de provar.E como a sequencia e infinita
existem infinitos primos.
Ou melhor:escolha um divisor primo de cada numero
de Fermat.

Ate,ass.:Johann
 --- Claudio Buffara
[EMAIL PROTECTED] escreveu:  Eu vou
dar mais um tempinho pra ver se alguem
 mais quer enviar sua lista e
 ai publico os enunciados, talvez ateh separados
 por assunto, como no Proofs
 from the Book. Jah as demonstracoes sao outra
 historia - e voluntarios serao
 muito bem-vindos. Eu me disponho a provar que
 ha infinitos primos (ou pelo
 menos mais de 200, o que jah eh um belo
 resultado parcial). Que tal?
 
 Um abraco,
 Claudio.
 
 on 13.08.03 15:58, Domingos Jr. at
 [EMAIL PROTECTED] wrote:
 
 E então, quando veremos todos esses belos
 teoremas/problemas listados e
 demonstrados/resolvidos?
 Seria muito legal ter tudo isso reunido em
 formato eletrônico...
 
 [ ]'s
 
 - Original Message -
 From: Cláudio (Prática)
 mailto:[EMAIL PROTECTED]
 To: [EMAIL PROTECTED]
 Sent: Wednesday, August 13, 2003 3:05 PM
 Subject: [obm-l] ENQUETE - BELEZA MATEMÁTICA
 [EMAIL PROTECTED];s+
 [EMAIL PROTECTED],Ak9Yt{T~tkTtO'T~UI/'E4YDwl
 Ea2c/G
 xYR +c%w= [EMAIL PROTECTED],ucz]*sYXF]fcuXGuc)Y2

]ffu]2*uc/uc+]cW]2T]l+]f1ucvuX]WFu+]X*u*]W/]WcY+A]c*YWvY!Y2,ucbucwY*F]fcu*{Y
 Wb]2+]2]2TY]WZYcYWcY*2]*u,Y*+]X*W]fZ]WF]2
 uXcu*]WZY*W]c+Y],ucz]*sYXF]fcuXGuc)Y2

]ffu]2*uc/uc+]cW]2T]l+]f1ucvuX]WFu+]X*u*]W/]WcY+A]c+YW,Y*z:[EMAIL PROTECTED]@[EMAIL 
PROTECTED]
 +1gAAA==^#~@ 
 
 
 
  

___
Conheça o novo Cadê? - Mais rápido, mais fácil e mais preciso.
Toda a web, 42 milhões de páginas brasileiras e nova busca por imagens!
http://www.cade.com.br
=
Instruções para entrar na lista, sair da lista e usar a lista em
http://www.mat.puc-rio.br/~nicolau/olimp/obm-l.html
=


[obm-l] Re: [obm-l] Opção_de_Compra

2003-08-14 Por tôpico Cláudio \(Prática\)

 Olá Claudio!

 Interessante isso! :)
 Se o comprador pega a ações emprestado, ele terá que
 devolver daqui um ano a ações mais 100*a*0,2 (20% do
 valor do empréstimo), ou seja, a ações + 20a

Na verdade, 20% é a taxa de juros para empréstimos e aplicações de reais e
não de ações (desculpe-me se eu não deixei isso claro antes). Para este
problema, vamos assumir que você toma ações emprestadas a 0% aa.

 se ele vender essa ação emprestada, ele embolsará
 100*a reais
 então, supondo que x  100a:
 comprando a opção, sobrará 100a-x, o que ele poderá
 investir, obtendo (100a-x)*1,2 em 1 ano.
 ai temos dois casos:

 (i) a ação está valendo R$50
 ele deverá comprar a ações por 50*a, para devolver a
 ações + 20a para o cara que emprestou as ações.
 nesse caso, ele gastará R$ 70a
 o que ele deve ter, para não ter prejuíso, então:
 (100a-x)*1,2 = 70a = 100a-x = 84a = 16a = x =
 a = x/16

 (ii) a ação está valendo R$ 200
 ele exerce a opção e compra 1 ação por R$ 110,
 mas ele tem que devolver a ações, e ainda pagar 20a
 reais.
 então lhe sobra 200 - 200a - 20a = 200-220a
 para ele não ter prejuíso: 200-220a = 0 =
 200 = 220a = 20 = 22a = 10/11 = a
 então:

 x/16 = a = 10/11
 mas para o vendedor não ter prejuíso, x = 40
 será que o vendedor também deve pegar emprestado a
 ação, em vez de emprestar dinheiro do banco, para
 depois comprar a ação?

 Abraços,
 Hélder

Fora o detalhe do custo do aluguel de ações, eu diria que você pegou o
espírito da coisa.
Tanto que eu vou dar a minha solução pro problema.

Vamos assumir o ponto de vista do vendedor da opção:

Na data inicial ele:
- Vende 1 opção == Recebe  x
- Compra a ações == Paga 100a
- Toma o saldo emprestado == Recebe (100a - x)
Saldo Líquido = Zero

Dentro de 1 ano, teremos 2 situações possíveis:
1) Ação vale R$ 200:
- Repaga empréstimo == Paga (100a - x)*1,2
- Compra (1-a) ações == Paga 200*(1 - a)
- Vende 1 ação contra exercício da opção == Recebe 110
Saldo Líquido: Recebe 110 - 200*(1 - a) - (100a - x)*1,2

2) Ação vale R$ 50:
- Repaga empréstimo == Paga (100a - x)*1,2
- Vende a ações na bolsa == Recebe 50a
Saldo Líquido: Recebe 50a - (100a - x)*1,2

Como na data inicial, ele não pagou nem recebeu nada (saldo = zero), a fim
de evitar a possibilidade de uma arbitragem (lucro garantido com risco
zero), dentro de 1 ano ele também não deverá pagar ou receber nada.
Ou seja, em ambos os cenários o seu saldo deverá ser Zero. Isso implica que:

110 - 200*(1 - a) - (100a - x)*1,2 = 0
50a - (100a - x)*1,2 = 0 ==

80a + 1,2x = 90
-70a + 1,2x = 0  ==

a = 3/5  e  x = 35

Ou seja, o preço justo da opção é R$ 35,00 e a fim de se proteger o vendedor
deve, após vendê-la, comprar 0,6 ações.

Repare que o preço justo da opção independe da distribuição de probabilidade
dos preços futuros da ação. Essa é a beleza da sacada do Black e do
Scholes: existe uma quantidade de ações (o a acima) que o vendedor da
opção pode comprar a fim de exatamente neutralizar o risco da opção.

Um abraço,
Claudio.




=
Instruções para entrar na lista, sair da lista e usar a lista em
http://www.mat.puc-rio.br/~nicolau/olimp/obm-l.html
=


Re: [obm-l] BELEZA: belgas e pontos.

2003-08-14 Por tôpico Eduardo Wagner


--
From: Nicolau C. Saldanha [EMAIL PROTECTED]
To: [EMAIL PROTECTED]
Subject: Re: [obm-l]  BELEZA: belgas e pontos.
Date: Sun, Aug 10, 2003, 8:23 AM


 On Sun, Aug 10, 2003 at 02:08:56AM -0300, Eduardo Wagner wrote:
 Nao ha duvida sobre o que esta escrito acima. Entretanto, ha um pedaco de
 frase assim: e ja sabemos o que estas curvas sao.
 Sim! Nos sabemos, mas os alunos provavelmente nao.

 Como não? No ensino médio eles sabem. Eu estranho a idéia de estudar
 cônicas de forma puramente grega, sem mencionar que estas são as curvas
 de grau 2.

Eu nao disse isto. Os alunos conhecem as curvas do segundo grau mas
nao sao capazes de identificar as secoes no cone com elas uma vez que
a geometria analitica no espaco nao faz parte do programa de matematica
do ensino medio do nosso pais. Coordenadas no R3 so existe aqui no Rio de
Janeiro e, ainda assim, de forma muito elementar; equacao da reta e do
plano e nada mais.

 Ja que fui citado (ou provocado) em mensagem anterior, quero dizer que
 os alunos podem perfeitamente conhecer as conicas muito antes de estarem
 familiarizados com a geometria analitica no espaco, translacoes e rotacoes.
 E isto eh muito bom. Conhecer desde cedo as curvas e suas diversas formas,
 definidas por um unico numero chamado excentricidade.
 A demonstracao legal que usa as esferas eh totalmente elementar e permite
 obter um resultado surpreendente que vai agora para a beleza matematica
 da lista.

 Só para pacificar um pouco, eu também gosto muito da demonstração com
 as esferas, não estou de forma alguma querendo sugerir que a demonstração
 que apresentei (de natureza algébrica) deva eliminar a demonstração de
 natureza mais geométrica. Mas eu me lembro de, ao descobrir a demonstração
 algébrica, ter tido aquela sensação de isto é tão simples, pq me esconderam
 isso durante tanto tempo?...

Para pacificar totalmente eu talvez diria que voce deve ter tido bons
professores. Eles tiveram a nocao do que um aluno do ensino medio pode
compreender com as ferramentas que possuem na ocasiao, e deixar um mundo
de decobertas surpreendentes para depois, quando tiveram mais maturidade e
conhecimento.
E isto foi muito bom.

Abraco,

Wagner.


 []s, N.
 =
 Instruções para entrar na lista, sair da lista e usar a lista em
 http://www.mat.puc-rio.br/~nicolau/olimp/obm-l.html
 =
=
Instruções para entrar na lista, sair da lista e usar a lista em
http://www.mat.puc-rio.br/~nicolau/olimp/obm-l.html
=


Re: [obm-l] BELEZA: belgas e pontos.

2003-08-14 Por tôpico Eduardo Wagner


--
From: Nicolau C. Saldanha [EMAIL PROTECTED]
To: [EMAIL PROTECTED]
Subject: Re: [obm-l]  BELEZA: belgas e pontos.
Date: Sat, Aug 9, 2003, 10:37 PM


 On Sat, Aug 09, 2003 at 08:42:04PM -0300, Augusto Cesar de Oliveira Morgado
 wrote:
 Os teoremas a respeito de as seçoes do cone por planos que nao contem o
 vertice serem elipses, parabolas ou hiperboles foram demonstrados por dois
 belgas, Quetelet e Dandelin, e sao conhecidos por muitos como os teoremas
 belgas. As demonstraçoes sao particularmente elegantes e surpreendentemente
 simples, principalmente se expostas no quadro-negro pelo Wagner.

 Ok. A demonstração do Wagner deve ser uma que envolve desenhar umas esferas.
 É legal, mas não é minha demonstração favorita deste fato.
 Por mim a demonstração certa consiste em observar que o cone tem equação
 de grau 2 (x^2 + y^2 = z^2) e rodar ou transladar não altera o grau.
 Tomar a interseção com um plano, digamos o plano z=0, já que rodamos,
 também não altera o grau (só estamos eliminando os termos que envolvem z).
 Logo a interseção é uma curva de grau 2 e já sabemos o que estas curvas são.

 Observe que assim também demonstramos que a interseção de um parabolóide
 ou um hiperbolóide com um plano também é uma cônica. Em particular,
 a interseção de um hiperbolóide de revolução de uma folha (x^2 + y^2 = 1 +
z^2)
 com um plano tangente tem um ponto duplo, logo é um par de retas.

 []s, N.

Nao ha duvida sobre o que esta escrito acima. Entretanto, ha um pedaco de
frase assim: e ja sabemos o que estas curvas sao.
Sim! Nos sabemos, mas os alunos provavelmente nao.

Ja que fui citado (ou provocado) em mensagem anterior, quero dizer que
os alunos podem perfeitamente conhecer as conicas muito antes de estarem
familiarizados com a geometria analitica no espaco, translacoes e rotacoes.
E isto eh muito bom. Conhecer desde cedo as curvas e suas diversas formas,
definidas por um unico numero chamado excentricidade.
A demonstracao legal que usa as esferas eh totalmente elementar e permite
obter um resultado surpreendente que vai agora para a beleza matematica
da lista.

Em um cone (duplo) as geratrizes fazem angulo X com o eixo do cone e um
plano corta esse cone fazendo angulo Y com o eixo. Entao a secao do plano
com o cone eh uma curva cuja excentricidade eh cosY/cosX.

Abracos.

E. Wagner.









 =
 Instruções para entrar na lista, sair da lista e usar a lista em
 http://www.mat.puc-rio.br/~nicolau/olimp/obm-l.html
 =
=
Instruções para entrar na lista, sair da lista e usar a lista em
http://www.mat.puc-rio.br/~nicolau/olimp/obm-l.html
=


[obm-l] Questão de Geometria

2003-08-14 Por tôpico Eduardo Casagrande Stabel
Olá a todos!

Considere um quadrado ABCD de lado unitário. Trace quatro circunferências de
raios unitários centradas em A, B, C e D. No centro do quadrado, forma-se
uma região limitada pelos quatro círculos. A pergunta que faço é: como
calcular a área dessa figura?

Um modo de fazer é encontrar funções cujos gráficos sejam a borda das
circunferências e uttilizando-se uma integral calcular a área compreendida
entre as curvas. Deste modo, eu chegei à área Pi/3 + 1 - sqrt(3). Eu
gostaria de saber se existe uma solução usando somente dos recursos da
geometria euclidiana, sem usar integrais.

Abraço aos que leram!
Duda.

=
Instruções para entrar na lista, sair da lista e usar a lista em
http://www.mat.puc-rio.br/~nicolau/olimp/obm-l.html
=


Re: [obm-l] EsSA

2003-08-14 Por tôpico Cleber da Silva Alves




Caros companheiros da 
lista:
Para o problema da 
fábrica:
tomando 'x' como o valor a ser pago 
por cada trabalhador e 't' como o número de trabalhadores a contribuir, 
temos:
no primeiro caso: 900/x = t 
(i)
no segundo caso: 888/x = t-2 = 
888/x + 2 = t (ii)
igualando o primeiro membro de (i) 
e (ii):
900/x = 888/x +2 = x = 
6
Logo cada um dos participantes 
contribuiu com R$ 6,00
(Obs: no primeiro caso haviam 150 
trabalhadores e no segundo caso 148.)

Para o segundo 
problema:
(x+y)^2 + x^2 - y^2 + 2x = 

x^2 + 2xy + y^2 + x^2 - y^2 + 2x 
=
2x^2 + 2x +2xy 
=
2x(x +1 +y) =
2 . 19 . 101 (todos 
primos)
Portanto a expressão é divisível 
por 2, 19 e 101.

Cleber da Silva AlvesICQ # 
198583572MSN [EMAIL PROTECTED]


Re: [obm-l] ENQUETE - BELEZA MATEMATICA

2003-08-14 Por tôpico Fabio Henrique
A tal beleza começou pela idéia. 
Parabéns. 


Em 09 Aug 2003, [EMAIL PROTECTED] escreveu: 

Caros colegas da lista: 
 
Gostaria de contar com sua participacao numa enquete sobre beleza 
matematica. 
 
O que eu precisao eh que cada um de voces me envie uma lista contendo algo 
como 5 a 10 problemas/teoremas que voces consideram os mais bonitos e cujas 
solucoes/demonstracoes sao as mais elegantes e/ou inusitadas e/ou 
engenhosas. Nao precisa incluir a solucao/demonstracao, apenas o enunciado. 
No entanto, se voce tiver em mente uma solucao/demonstracao especifica 
(entre varias existentes) nao deixe de mencionar pelo menos o metodo 
utilizado. 
 
A unica restricao eh que estes resultados devem ser de um nivel acessivel a 
um aluno normal de 2o. grau (ou seja, o Ultimo Teorema de Fermat e o 
Porisma 
de Poncelet estao fora, mas o caso n = 4 do UTF e a versao para triangulos 
do Porisma poderiam ser incluidos). 
 
Importante: os resultados devem ser acessiveis a um aluno normal de 2o. 
grau, mas nao necessariamente fazer parte do curriculo normal do 2o. grau. 
 
Tambem nao precisa responder hoje ou amanha ou mesmo na semana que vem. 
Acho 
que vale a pena pensar por um tempo e consultar a literatura - as vezes 
pode 
ter um resultado belissimo do qual voce simplesmente se esqueceu por nao 
encontra-lo ha muito tempo. As Eurekas sao uma otima referencia. O Proofs 
from the Book tambem, apesar de nem tudo lah ter nivel de 2o. grau. 
 
Se houver um numero suficiente de respostas, eu me comprometo a publicar 
uma 
compilacao dos problemas e teoremas mais votados. 
 
Desde jah a gradeco o interesse de quem quiser participar. 
 
Um abraco, 
Claudio. 
 
= 
Instruções para entrar na lista, sair da lista e usar a lista em 
http://www.mat.puc-rio.br/~nicolau/olimp/obm-l.html 
= 
 
-- 

_
Voce quer um iGMail protegido contra vírus e spams? 
Clique aqui: http://www.igmailseguro.ig.com.br
Ofertas imperdíveis! Link: http://www.americanas.com.br/ig/
Ofertas imperdíveis!

=
Instruções para entrar na lista, sair da lista e usar a lista em
http://www.mat.puc-rio.br/~nicolau/olimp/obm-l.html
=


Re: [obm-l] Problemas em Aberto - Algarismos

2003-08-14 Por tôpico Aleandre Augusto da Rocha
Nao seria 3*10^(k+1) + 6*10^k?

-Auggy

- Original Message -
From: Johann Peter Gustav Lejeune Dirichlet
[EMAIL PROTECTED]
To: [EMAIL PROTECTED]
Sent: Tuesday, August 05, 2003 12:42 PM
Subject: Re: [obm-l] Problemas em Aberto - Algarismos


 Retorno do Abertos da lista?
 Que tal a gente achar quadrados perfeitos do tipo
 3*10^k+6*10^l?
 O tres nao pode vir no final.Talvez
 modulo...Depois eu penso...
  --- Claudio Buffara
 [EMAIL PROTECTED] escreveu:  Caros
 colegas:
 
  Aqui vao dois problemas que ainda estao em
  aberto na lista. O primeiro foi
  enviado pelo Duda Stabel. O segundo eh da
  olimpiada iraniana, se nao me
  engano.
 
  1) Determinar o conjunto de números inteiros
  positivos que satisfazem à duas
  condições: (i) todo número possui exatamente
  dois algarismos não-nulos,
  sendo um deles o três(3), (ii) todo número é
  quadrado perfeito.
 
  2) Prove ou disprove: existe uma potencia de 2
  tal que ao se permutar os
  algarismos de sua representacao decimal
  obtem-se uma outra potencia de 2.
 
  Esse segundo tem uma solucao aparentemente
  simples, mas esta solucao exclui
  o caso de potencias de 2 com algarismos 0
  internos (ou seja, numeros do
  tipo abcdefg).
 
  Um abraco,
  Claudio.
 
 
 

 ___
 Conheça o novo Cadê? - Mais rápido, mais fácil e mais preciso.
 Toda a web, 42 milhões de páginas brasileiras e nova busca por imagens!
 http://www.cade.com.br
 =
 Instruções para entrar na lista, sair da lista e usar a lista em
 http://www.mat.puc-rio.br/~nicolau/olimp/obm-l.html
 =



=
Instruções para entrar na lista, sair da lista e usar a lista em
http://www.mat.puc-rio.br/~nicolau/olimp/obm-l.html
=


[obm-l] Re: [obm-l] Problemas em Aberto - Algarismos

2003-08-14 Por tôpico yurigomes
 Esse segundo problema caiu na OBM 2000, numa versão mais fácil.
 Acho que foi essa versão a que vc resolveu, jah que ele dizia que as duas
potências têm que ter o mesmo número de algarismos, de modo que os zeros
não modificavam a quantidade de algarismos.
 Ateh mais, 
 Yuri
  
-- Mensagem original --

Caros colegas:

Aqui vao dois problemas que ainda estao em aberto na lista. O primeiro
foi
enviado pelo Duda Stabel. O segundo eh da olimpiada iraniana, se nao me
engano.

1) Determinar o conjunto de números inteiros positivos que satisfazem à
duas
condições: (i) todo número possui exatamente dois algarismos não-nulos,
sendo um deles o três(3), (ii) todo número é quadrado perfeito.

2) Prove ou disprove: existe uma potencia de 2 tal que ao se permutar os
algarismos de sua representacao decimal obtem-se uma outra potencia de
2.

Esse segundo tem uma solucao aparentemente simples, mas esta solucao exclui
o caso de potencias de 2 com algarismos 0 internos (ou seja, numeros
do
tipo abcdefg).

Um abraco,
Claudio.




[]'s, Yuri
ICQ: 64992515


--
Use o melhor sistema de busca da Internet
Radar UOL - http://www.radaruol.com.br



=
Instruções para entrar na lista, sair da lista e usar a lista em
http://www.mat.puc-rio.br/~nicolau/olimp/obm-l.html
=


Re: [obm-l] EsSA

2003-08-14 Por tôpico Bernardo Vieira Emerick



From: [EMAIL PROTECTED]
Reply-To: [EMAIL PROTECTED]
To: [EMAIL PROTECTED]
Subject: Re: [obm-l] EsSA
Date: Sun, 10 Aug 2003 11:29:15 EDT
Resolvi alguns,

Em uma mensagem de 10/8/2003 11:37:02 Hora padrão leste da Am. Sul,
[EMAIL PROTECTED] escreveu:
 2 – um festival de música lotou uma praça semicircular

 de 200m de diâmetro. Admitindo-se uma ocupação média

 de 3 pessoas por m^2, qual o número mais aproximado de

 pessoas presentes?

 (Adote pi=3,14)



 Achei: 47.100



 3 – Sendo x= 19 e y= 81, então a expressão (x+y)^2 +

 x^2 – y^2 + 2x é divisível por:



 a)2,19 e 81
 b)2,19 e 101
 c)2,81 e 100
 d)19,100 e 101
 e)81,100 e 101

 achei a letra B


(x+y)^2 + x^2 – y^2 + 2x

100^2 +((x+y)*(x-y)) + 38

1 + (100*(-62)) + 38

=3838 que eh dividsivel por 101, mas nao por 2,19...2,81...100...

O que fiz de errado ?






 Um triângulo ABC tem área 75m^2. os pontos D,E,F e G
 dividem o lado AC em 5 partes congruentes:
 AD=DE=EF=FG=GC. Desse modo, a área do triangulo BDF é:

 20
 30
 40
 50
 55
Esse nao consegui.


 Numa fábrica, trabalhadores reuniram-se para
 presentear um amigo que iria casar. O presente
 escolhido foi a quantia de 900,00, que seria dividida
 igualmente entre eles. Por razões particulares, dois
 daqueles trabalhadores tiraram seus nomes da lista e,
 por isso, decidiu-se diminuir a quantia para 888,00,
 de modo que na nova divisão coubesse a cada
 participante a mesma cota de antes da saída dos dois
 colegas. Com isso, coube a cada um dos participantes a
 quantia de :

 4,00
 6,00
 9,00
 10,00
 12,00



x= numero de pessoas
y=qtia paga por cada um
x*y = 900 (i)
(x-2)*y= 888 (ii)
De (ii) xy - 2y = 888
900 - 2y = 888
y=6
_
MSN Messenger: converse com os seus amigos online.  
http://messenger.msn.com.br

=
Instruções para entrar na lista, sair da lista e usar a lista em
http://www.mat.puc-rio.br/~nicolau/olimp/obm-l.html
=


Re: [obm-l] Re:_[obm-l]_Número_de_soluções_de_sistema_linear_-_Correção

2003-08-14 Por tôpico Johann Peter Gustav Lejeune Dirichlet
Nao necessariamenteOs caras nao precisam ser
maximos.O e=23 serve...
 --- Aleandre Augusto da Rocha
[EMAIL PROTECTED] escreveu:  
 Uma maneira de resolver apenas com
 conhecimentos de 5a serie e por conversao
 de base...
 note que 23(base10) = abcde(base2)
 como 23(base10) = 10111(base2)
 a=c=d=e=1, b=0
 
 -Auggy
 
 - Original Message -
 From: Alexandre Daibert
 [EMAIL PROTECTED]
 To: [EMAIL PROTECTED]
 Sent: Thursday, August 07, 2003 12:38 AM
 Subject: [obm-l] Número de soluções de sistema
 linear - Correção
 
 
  Desculpem-me pelo meu erro. O problema é
 determinar o número de soluções
  inteiras não negativas...
  Sendo assim como posso resolver? (nível de
 segundo grau se possível)
 
  Claudio Buffara escreveu:
 
  on 06.08.03 02:15, Alexandre Daibert at
 [EMAIL PROTECTED]
 wrote:
  
  
  
  Gostaria de ajuda para o seguinte problema:
  
 
  Calcular o número de soluções do sistema:
  16a + 8b + 4c + 2d + e = 23
  sendo a, b, c, d, e inteiros positivos.
  se possível usar somente conhecimentos de
 ensino médio, se isto não for
  possível, pelo tente explicar mais ou menos
 o q está fazendo para q um
  ignorante aluno q ainda não entrou em um
 curso superior possa entender
 :-)
  
  Alexandre Daibert - Juiz de Fora -
 [EMAIL PROTECTED]
  
  
 

=
  Instruções para entrar na lista, sair da
 lista e usar a lista em
 

http://www.mat.puc-rio.br/~nicolau/olimp/obm-l.html
 

=
  
  
  
  Oi, Alexandre:
  
  Se a, b, c, d, e sao inteiros positivos,
 entao o menor valor possivel
 para:
  16a + 8b + 4c + 2d + e  eh igual a  16*1 +
 8*1 + 4*1 + 2*1 + 1 = 31  23.
  Logo, o sistema dado (composto duma unica
 equacao) nao tem solucao em
  inteiros positivos, ou seja, o numero de
 solucoes pedido eh zero.
  
  Provavelmente, o enunciado nao eh bem esse.
 De uma conferida.
  
  Um abraco,
  Claudio.
  
 

=
  Instruções para entrar na lista, sair da
 lista e usar a lista em
 

http://www.mat.puc-rio.br/~nicolau/olimp/obm-l.html
 

=
  
  
  
  
 
 
 
 

=
  Instruções para entrar na lista, sair da
 lista e usar a lista em
 

http://www.mat.puc-rio.br/~nicolau/olimp/obm-l.html
 

=
 
 
 

=
 Instruções para entrar na lista, sair da lista
 e usar a lista em

http://www.mat.puc-rio.br/~nicolau/olimp/obm-l.html

= 

___
Conheça o novo Cadê? - Mais rápido, mais fácil e mais preciso.
Toda a web, 42 milhões de páginas brasileiras e nova busca por imagens!
http://www.cade.com.br
=
Instruções para entrar na lista, sair da lista e usar a lista em
http://www.mat.puc-rio.br/~nicolau/olimp/obm-l.html
=


RE: [obm-l] ITA-96

2003-08-14 Por tôpico Leandro Lacorte Recôva
Jorge,

Eu pensei nessa questao e tenho uma observacao. Na 2a equacao do
somatorio eles colocam 

X_i + 1 o que acarreta (x1 + 1)(x2+1) , etc... 

Mas eu considerei que o termo +1 estava somando o indice i e nao a x_i,
e dai a questao saiu. Caso nao seja isso, eu nao sei como a questao sai.


Use os fatos:

f(x^n)=f(x.x^n-1) = f(x) + f(x^n-1) = nf(x) 
f(x1.x2.x3.x4.x5)=f(x1)+f(x2)+f(x3)+f(x4)+f(x5) 

___

Do primeiro somatorio, voce tem 

Sum(i=1,5) f(xi) = 13f(2) + 2f(x1). Usando o fato de f(x.y)=f(x)+f(y)
podemos escreve-lo como 

F(x1.x2.x3.x4.x5) = f(2^13.x1^2)(1) 

Como x1,x2,x3,x4,x5 formam uma PG entao temos 

f(x1^5.q^10)=f(2^13.x1^2). Mas f e injetora, logo, 

x1^5.q^10 = 2^13.x1^2 = x1^3.q^4 = 2^13  (*)


Agora, o segundo somatorio (quebrei a cabeca ate ver a observacao acima)
e acho que essa notacao induziu as pessoas a terem um pensamento errado:



Sum(i=1,4) f(xi/x_i+1) = -2f(2x1). Usando a 1a propriedade de f podemos
escrever o somatorio da forma 

f(x1.x2.x3.x4/x2.x3.x4.x5) = -2f(2x1)  (2)

Note que f(1)=0 = f(x.1/x) = f(x) + f(1/x) = 0 = f(x)=-f(1/x). Usando
esse fato em (2) temos 

f(x1/x5) = f(1/4x1^2) = Como f e injetora temos x1/x5 = 1/4x1^2. Usando
o fato de que x5=x1q^4 entao,

q=(2x1)^(1/2). Substituindo esse valor em (*) entao teremos 

x1^3.q^10 = 2^13 = x1=2 e q=2. 


Se tiver algum erro de conta, corrijam por favor, mas o raciocinio era
esse. 

Regards,

Leandro. 
Los Angeles,CA








-Original Message-
From: [EMAIL PROTECTED]
[mailto:[EMAIL PROTECTED] On Behalf Of Jorge Paulino
Sent: Thursday, August 07, 2003 9:21 AM
To: [EMAIL PROTECTED]
Subject: [obm-l] ITA-96

Tô com dificuldades na resoluçao.
um abraço,
Jorge

___
Conheça o novo Cadê? - Mais rápido, mais fácil e mais preciso.
Toda a web, 42 milhões de páginas brasileiras e nova busca por imagens!
http://www.cade.com.br
=
Instruções para entrar na lista, sair da lista e usar a lista em
http://www.mat.puc-rio.br/~nicolau/olimp/obm-l.html
=


[obm-l] x^x = 2^(-raiz(x))

2003-08-14 Por tôpico Claudio Buffara
on 01.08.03 15:10, Paulo Santa Rita at [EMAIL PROTECTED] wrote:

 Ola Pessoal !
 
 Alguem me propos a questao ( que compartilho com voces ) :
 
 Quantas solucoes reais tem X^X = 2^(- RAIZ_2(X)), onde RAIZ_2(X) e a raiz
 quadrada de X.
 
 Regra : Nao vale usar calculo !
 Dica : X=1/e pode ser um ponto importante ...
 
 Um Abraco
 Paulo Santa Rita
 6,1508,010803
 
Oi, Paulo:

O universo de x tem que ser o conjunto dos reais positivos.

x^x = 2^(-raiz(x)) ==

(x^raiz(x))^raiz(x) = (1/2)^raiz(x) ==

x^raiz(x) = 1/2

Vamos supor que x = 1/2^n. Nesse caso:

x^raiz(x) = (1/2^n)^(1/2^(n/2)) = (1/2)^(n/2^(n/2)) = 1/2 ==
n/2^(n/2) = 1 ==
n = 2^(n/2) ==
n^2 = 2^n ==
n = 2  ou  n = 4  ou  n = -a,
onde a eh um numero real positivo menor do que 1 e tal que a^2 = 2^(-a)
(repare que os graficos de y = x^2 e y = 2^x se intersectam num ponto de
abscissa negativa igual a -a. Nao faco a menor ideia se a eh racional ou
irracional ou mesmo transcendente, mas apostaria nessa ultima alternativa)

n  4 == 2^n  n^2 == n = 4 eh a maior solucao de n^2 = 2^n

Portanto:
n = 2 == x = 1/4
Testando: 
x^x = (1/4)^(1/4) = (1/2)^(1/2) = 1/raiz(2)
2^(-raiz(x)) = 2^(-raiz(1/4)) = 2^(-1/2) = 1/raiz(2) ==
x = 1/4 eh raiz

n = 4 == x = 1/16
Testando:
x^x = (1/16)^(1/16) = (1/2)^(4/16) = 1/2^(1/4)
2^(-raiz(x)) = 2^(-raiz(1/16)) = 1/2^(1/4) ==
x = 1/16 eh raiz

n = -a == x = 2^a
Testando:
x^x = (2^a)^(2^a) = 2^(a*2^a) = 2^(a/a^2) = 2^(1/a)
2^(-raiz(x)) = 2^(-raiz(2^a)) = 2^(-raiz(1/a^2)) = 2^(-1/a) ==
x = 2^a nao eh raiz

Assim, a equacao original tem 2 solucoes: x = 1/4 e x = 1/16.


Um abraco,
Claudio.

 

=
Instruções para entrar na lista, sair da lista e usar a lista em
http://www.mat.puc-rio.br/~nicolau/olimp/obm-l.html
=


Re: [obm-l] Problemas em Aberto - Algarismos

2003-08-14 Por tôpico Cláudio \(Prática\)
Vou mais longe:

Os candidatos são os quadrados da forma:
(3*10^m + A)*10^(2n)
onde A pertence a {1,4,6} e m e n são inteiros não negativos.

Até agora, só encontrei números do tipo:
36, 3600, 36, ..., 36*10^(2n), ...
mas não consegui provar que são os únicos.

Um abraço,
Claudio.

- Original Message -
From: Johann Peter Gustav Lejeune Dirichlet
[EMAIL PROTECTED]
To: [EMAIL PROTECTED]
Sent: Tuesday, August 05, 2003 1:42 PM
Subject: Re: [obm-l] Problemas em Aberto - Algarismos


 Retorno do Abertos da lista?
 Que tal a gente achar quadrados perfeitos do tipo
 3*10^k+6*10^l?
 O tres nao pode vir no final.Talvez
 modulo...Depois eu penso...
  --- Claudio Buffara
 [EMAIL PROTECTED] escreveu:  Caros
 colegas:
 
  Aqui vao dois problemas que ainda estao em
  aberto na lista. O primeiro foi
  enviado pelo Duda Stabel. O segundo eh da
  olimpiada iraniana, se nao me
  engano.
 
  1) Determinar o conjunto de números inteiros
  positivos que satisfazem à duas
  condições: (i) todo número possui exatamente
  dois algarismos não-nulos,
  sendo um deles o três(3), (ii) todo número é
  quadrado perfeito.
 
  2) Prove ou disprove: existe uma potencia de 2
  tal que ao se permutar os
  algarismos de sua representacao decimal
  obtem-se uma outra potencia de 2.
 
  Esse segundo tem uma solucao aparentemente
  simples, mas esta solucao exclui
  o caso de potencias de 2 com algarismos 0
  internos (ou seja, numeros do
  tipo abcdefg).
 
  Um abraco,
  Claudio.
 
 
 

 ___
 Conheça o novo Cadê? - Mais rápido, mais fácil e mais preciso.
 Toda a web, 42 milhões de páginas brasileiras e nova busca por imagens!
 http://www.cade.com.br
 =
 Instruções para entrar na lista, sair da lista e usar a lista em
 http://www.mat.puc-rio.br/~nicolau/olimp/obm-l.html
 =

=
Instruções para entrar na lista, sair da lista e usar a lista em
http://www.mat.puc-rio.br/~nicolau/olimp/obm-l.html
=


[obm-l] PREFERÊNCIA SEXUAL

2003-08-14 Por tôpico jorgeluis
Olá! meus amigos,

Talvez eu não tenha mais vida longa na lista, isto se não for apedrejado por 
tocar num assunto tão delicado, mas é que, opção sexual, cada qual administra 
da melhor maneira possível. É bom deixar claro que em momento algum 
tive intenções de subestimar o prof. Morgado, jamais faria isto, apenas 
ratifiquei que o autor Jonofon Sérates participou magistralmente nos dois 
programas. Agora, não vamos misturar as coisas, pois quem de nós não tem um 
caso lamentável nas nossas famílias. O problema é que a verdade, muitas vezes é 
desconfortável. Sôbre a ocorrência policial, talvez tenha sido um dos primeiros 
a saber através da minha querida filha Dândara de 14 anos que adora o seu 
método CUCA LEGAL e vinha trocando e-mail's com o autor. Todos ficamos 
estarrecidos, mas a vida segue o seu curso e oremos! para que nossos filhos não 
sejam vítimas deste mundo insãno. Agora, vamos ao que interessa, pois os 
problemas do caramujo e dos dicionários, já estão dando o que falar, isto sem 
falar na pegadinha do Pereira e do viajante dorminhoco, que vocês não deram a 
mínima atenção. Avante!

Um abraço!


WebMail UNIFOR - http://www.unifor.br
=
Instruções para entrar na lista, sair da lista e usar a lista em
http://www.mat.puc-rio.br/~nicolau/olimp/obm-l.html
=


[obm-l] Raizes Cubicas - em aberto

2003-08-14 Por tôpico Claudio Buffara
Oi, pessoal:

Achei mais um problema em aberto, este submetido pelo Crom ha algumas
semanas:

Determine todos os inteiros positivos A e B tais que:

(A^(1/3) + B^(1/3) - 1)^2 = 49 + 20*6^(1/3)


Um abraco,
Claudio.


=
Instruções para entrar na lista, sair da lista e usar a lista em
http://www.mat.puc-rio.br/~nicolau/olimp/obm-l.html
=


[obm-l] Re: [obm-l] Número de soluções de sistemalinear

2003-08-14 Por tôpico Paulo Santa Rita
Ola Daibert e demais colegas
desta lista ... OBM-L,
Conforme outros membros desta lista ja observaram, a questao que voce propos 
nao tem solucao. Todavia, se voce aceitar que as solucoes sejam formadas por 
INTEIROS NAO NEGATIVOS, ela tem solucao. Alias, uma generalizacao natural 
PODE SER :

(Generalizacao) Discutir as solucoes formadas por INTEIROS NAO-NEGATIVOS da 
equacao :

A1*X1 + A2*X2 + ... + An*Xn = B

onde os Ai e o B sao inteiros positivos.

Um abraco
Paulo Santa Rita
4,1113,060803
on 06.08.03 02:15, Alexandre Daibert at [EMAIL PROTECTED] wrote:

Gostaria de ajuda para o seguinte problema:

Calcular o número de soluções do sistema:
16a + 8b + 4c + 2d + e = 23
sendo a, b, c, d, e inteiros positivos.
se possível usar somente conhecimentos de ensino médio, se isto não for
possível, pelo tente explicar mais ou menos o q está fazendo para q um
ignorante aluno q ainda não entrou em um curso superior possa entender :-)
Alexandre Daibert - Juiz de Fora - [EMAIL PROTECTED]
_
MSN Messenger: converse com os seus amigos online.  
http://messenger.msn.com.br

=
Instruções para entrar na lista, sair da lista e usar a lista em
http://www.mat.puc-rio.br/~nicolau/olimp/obm-l.html
=


Re: [obm-l] ajuda

2003-08-14 Por tôpico Fabio Bernardo
O conjunto solução dessa equação é vazio.

 (x²+1)²0 para todo x pertencente aos reais e (x²+3x-17)²=0,
logo,(x²+1)²+(x²+3x-17)² não pode ser igual a zero para nenhum x pertencente
aos reais.

Resposta: letra A

- Original Message -
From: Daniel Pini [EMAIL PROTECTED]
To: [EMAIL PROTECTED]
Sent: Saturday, August 09, 2003 6:29 PM
Subject: [obm-l] ajuda


Eu estou com duvida na seguinte questão:
dada  a equação (x²+1)²+(x²+3x-17)²=0, pode-se afirmar que no universo dos
números reais, o seu conjunto solução:
a) é vazio
b) tem apenas dois elementos
Obs: essa é uma questão da prova do CN e segundo os cursinhos preparatorios
a resposta correta é a) mas a marinha divulgou com  correta a alternativa b)



Esta mensagem continha vírus e foi descontaminada pelo E-mail Protegido
Terra.
Para impedir a contaminação do seu computador o E-mail Protegido Terra
remove
qualquer arquivo anexado que possua vírus.
Por este motivo, a mensagem pode estar mencionando um arquivo anexado que
foi
removido por segurança.
Veja abaixo o relatório do E-mail Protegido Terra

Mensagem: [obm-l] ajuda
Remetente: Daniel Pini [EMAIL PROTECTED]
Data de envio: 09/08/2003 18:29'16
Arquivo contaminado: (t/033e.js/033e.js)
Nome do vírus: JS/[EMAIL PROTECTED]

(t/033e.js/033e.js) (JS/[EMAIL PROTECTED])
Estes anexos foram removidos




=
Instruções para entrar na lista, sair da lista e usar a lista em
http://www.mat.puc-rio.br/~nicolau/olimp/obm-l.html
=


Re: [obm-l] Ajuda

2003-08-14 Por tôpico Thyago
Olá Claudio e companheiros da lista

Bom, sabe que estou me convencendo mesmo que esta solução é prática :-)

O que eu estava querendo inicialmente é uma solução que nem a da questão
abaixo, veja só:

S = sen(a) + sen(2a) + sen(3a) + ... + sen(na)

Em que a solução consiste em multiplicar ambos os lados da igualdade pelo
seno da metade da razão da PA, e após efetuar a prostaférese e sair
cortando. Sem muitas delongas!
...

Já ouvi dizer que a resolução que procuro existe, e está escrita em um tal
livro russo chamado Lidski, problemas de PA, ou algo do gênero... mas
nunca tive o privilégio de ter algum contato com essa obra. Alguém já ouviu
falar?

Atenciosamente
¡Thyago!



- Original Message -
From: Claudio Buffara [EMAIL PROTECTED]
To: [EMAIL PROTECTED]
Sent: Tuesday, August 12, 2003 9:58 AM
Subject: Re: [obm-l] Ajuda


 Oi, Thyago:

 Vou te confessar uma coisa: usando a identidade 1 - cis(a) =
 -2isen(a/2)cis(a/2) e mais esse problema do IME, que alias eh uma
 propriedade classica (e, como voce mostrou, util!) das raizes n-esimas da
 unidade, voce chegou a uma solucao mais curta e elegante do que a que eu
 tinha em mente. Parabens!

 A minha ideia era separar os casos n par e n impar e fatorar x^n - 1 de
duas
 maneiras diferentes:
 Primeiro:
 x^(2m) - 1 = (x^2 - 1)*(x^(2m-2) + x^(2m-4) + ... + x^4 + x^2 + 1)
 x^(2m+1) - 1 = (x - 1)*(x^(2m) + x^(2m-1) + ... + x^2 + x + 1)

 Depois:
 x^(2m) - 1 = (x^2 - 1)*PRODUTO(1=k=m-1)(x^2 - 2xcos(kpi/m)x + 1)
 x^(2m+1) - 1 = (x - 1)*PRODUTO(1=k=m)(x^2 - 2xcos(2kpi/(2m+1)) + 1)

 E depois, fazer x = 1 e igualar as expressoes obtidas, mas a sua solucao
eh
 mais simples e, portanto, melhor.

 O passo que faltou na sua solucao foi mostrar explicitamente que
 (-i)^(m-1)*cis(pi/n)*cis(2pi/n)*...*cis((n-1)pi/n) = 1
 mas isso eh bem facil (apesar de nao ser evidente).

 Um abraco,
 Claudio.

 PS: Se essa sua solucao nao eh pratica, entao eu nao sei o que eh.
Repare:
 voce tem um produto de senos de numeros em PA. Como voce propoe
calcula-los?
 Puramento por meio de identidades trigonometricas, sem usar complexos? Boa
 sorte...

 on 12.08.03 00:45, Thyago at [EMAIL PROTECTED] wrote:

  Olá Cláudio,
 
  Obrigado pelas dicas  :-)
 
  Mas a resolução que eu fiz não foi nada prática não.
  Eu já utilizei todas estas propriedades e não consegui chegar em nada.
  Bom, só para esclarecer um pouco mais... vou colocar o exercício que
gerou
  tal questão:
 
 
  (IME) Sejam 1, X2, X3, ..., Xn as raízes de x^n=1. Calcule: P = (1 -
  x2)(1-x3)...(1-xn).
 
  Fazendo uso de Briot-Rufini e fatoração de polinômios, conseguimos
chegar
  facilmente na resposta P = n.
  Mas, utilizando o tratamento vetorial de números complexos com a fórmula
  1-cis(a) = -2isen(a/2)cis(a/2) chegamos em
 
  P = 2^(n-1) . S
 
  Onde S = sen(pi/n) . sen(2pi/n) . sen(3pi/n) . ... . sen[(n-1)pi/n]
 
  Daí, utilizando a resposta da primeira resolução com a resposta da
segunda
  resolução temos que S = n/[2^(n-1) ]
  Dá para ver que esta demonstração para S não é nada prática.
 
  Você citou uma solução padrão para este tipo de problema. Qual seria?
 
  Aguardo resposta
 
  Atenciosamente
  ¡Thyago!
 
  - Original Message -
  From: Cláudio (Prática) [EMAIL PROTECTED]
  To: [EMAIL PROTECTED]
  Sent: Monday, August 11, 2003 2:19 PM
  Subject: Re: [obm-l] Ajuda
 
 
  Oi, Thyago:
 
  A solução padrão pra esse tipo de problema realmente envolve
complexos e
  polinômios.
 
  Tentando resolver outros problemas similares, você vai perceber que
  complexos e polinômios são uma forma de resolução bastante natural.
 
  Os resultados básicos são os seguintes:
  1) Todo número complexo pode ser representado na forma R*(cos(a) +
  i*sen(a)), onde R é um real não negativo e a é um real qualquer
(mas
  normalmente limitado ao intervalo [0, 2pi) ou então (-pi,pi]);
  2) e^(i*a) = cos(a) + i*sen(a): essa é a definição da função
exponencial
  complexa, que permite, por exemplo, que você transforme sequências de
  senos
  e cossenos de números reais em PA em sequências de complexos em PG, que
as
  vezes são mais fáceis de manipular;
  3) Um polinômio com coeficientes reais pode ser expresso como o produto
de
  binômios da forma (x - b) e/ou trinômios da forma (x^2 - 2*R*cos(a)*x +
  R^2), onde a e b são números reais quaisquer e R é um real positivo.
 
  Um abraço,
  Claudio.
 
 
  - Original Message -
  From: dex [EMAIL PROTECTED]
  To: [EMAIL PROTECTED]
  Sent: Monday, August 11, 2003 11:05 AM
  Subject: [obm-l] Ajuda
 
 
  Olá pessoal
 
  Gostaria de saber uma boa demonstração para o exercício abaixo
 
  P = sen(pi/n) . sen(2pi/n) . sen(3pi/n) . ... . sen[(n-1)pi/n]
  com n Inteiro positivo
 
  A resposta é P = n/[2^(n-1)], mas cheguei até este resultado de uma
  maneira
  muito pouco prática, nada natural para uma questão de matemática (de
  vestibular). Consegui prová-la utilizando o resultado de uma outra
  questão,
  que versava sobre polinômios e complexos. Ou seja, se eu não tivesse
  visto
  esta outra 

Re: [obm-l] ENQUETE - BELEZA MATEMATICA

2003-08-14 Por tôpico Frederico Reis Marques de Brito
Exatamente Cláudio, o Princípio de Dirichlet tb é conhecido como Princípio 
da Casa dos Pombos ou das gavetas.

O exemplo do monge é muito bom. Coloquei-o certa vez numa prova de cálculo 
I. Os alunos acharam bacana. E quanto ao TNP a prova não é simples 
realmente, mas a tentação de mostrar aos alunos a relação entre ln  e os nos 
primos, destas relações absolutaamente inesperadas, é forte.

Aproveito para colocar mais alguns resultados e dizer que, a medida que leio 
as respostas dos nossos colegas a sua enquete, fico cada vez mais perplexo, 
pois raramente discordo de algum, o que me alegra por demonstrar que, 
convenhamos, a Matemática é linda demais.

(6) Esse é simples e bonitinho demais: Existem desertos de primos tão 
grandes qto se queira, isto é, formalmente: dado N natural, existe uma 
sequência de N inteiros consecutivos compostos.

(7) A demonstração de que os números transcendentes são não-enumeráveis.

(8) A solução da eq: 2^x = x^2  . ( acho que se encontra isto em : Meu 
Professor de Matemática... , do Elon. SBM )

(9) A demonstração de que  C  não é um corpo ordenado, pela simplicidade. ( 
Aqui vale dizer que não é necessário o emprego dos termos técnicos, como 
corpo, por exemplo... )

(10) A relação de Euler para poliedros, que, pecaminosamente havia me 
esquecido.

Abraços,

Frederico.

From: Claudio Buffara [EMAIL PROTECTED]
Reply-To: [EMAIL PROTECTED]
To: [EMAIL PROTECTED]
Subject: Re: [obm-l] ENQUETE - BELEZA MATEMATICA
Date: Sat, 09 Aug 2003 20:34:04 -0300
on 09.08.03 18:39, Frederico Reis Marques de Brito at 
[EMAIL PROTECTED]
wrote:

 1) Acho que esse será praticamente unânime: Teorema de Euclides sobre a
 exist~encia de  infinitos primos.

 2) Teorema de Bezout sobre MDC: O máximo dvisor comum de dois inteiros é 
uma
 comb. linear inteira ( em realidade a menor positiva ) desses números ,
 pelas várias aplicações deste na Teoria dos Números.

 3) O Princípio de Dirichlet, pela potência .

 4) O posto-linha = posto-coluna. Não sei mais sempre achei este 
resultado
 muito inusitado, já que uma matriz nada mais é que um amontoado de
 números...

 5) Teorema do Valor Intermediário ( Acho que podemos abordar este tema 
no
 ensino médio... )

 Como o Morgado, pensarei um pouco mais antes de enviar outros 5. (  A
 propósito é tentador citar o Teorema dos Números Primos, mas acho que 
esse
 tema não seria acessível. Fica um voto de louvor então!)

 Frederico.

Oi, Frederico.

O principio de Dirichlet a que voce se refere eh o das casas de pombos?

O TNP eh um pouco avancado demais (mesmo com uma demonstracao elementar),
mas aquelas desigualdades de Chebichev sao aceitaveis, assim como o
postulado de Bertrand, que penso seriamente em botar na minha lista.
Pra mim, o TVI (e qualquer resultado que dependa do axioma do supremo) foi
um caso mais dificil de decidir, mas como tem aquele probleminha do monge
subindo e descendo a montanha, acho que ele tambem eh aceitavel. E, afinal
de contas, tem um volume da colecao do Iezzi que trata de limites, 
derivadas
e integrais...

Um abraco,
Claudio.
=
Instruções para entrar na lista, sair da lista e usar a lista em
http://www.mat.puc-rio.br/~nicolau/olimp/obm-l.html
=
_
MSN Hotmail, o maior webmail do Brasil.  http://www.hotmail.com
=
Instruções para entrar na lista, sair da lista e usar a lista em
http://www.mat.puc-rio.br/~nicolau/olimp/obm-l.html
=


[obm-l] Soma de Senos

2003-08-14 Por tôpico Claudio Buffara
Oi, Thyago:

Uma solucao 100% trigonometrica pra essa soma de senos voce encontra no
livro do Luis Lopes - Manual de Trigonometria, ou entao, voce pode usar
sen(x) = (e^(ix) - e^(-ix))/(2i) e transformar a soma em duas PGs complexas.

Os dois jeitos sao um pouco bracais.

Um abraco,
Claudio.

on 12.08.03 21:07, Thyago at [EMAIL PROTECTED] wrote:

 Olá Claudio e companheiros da lista
 
 Bom, sabe que estou me convencendo mesmo que esta solução é prática :-)
 
 O que eu estava querendo inicialmente é uma solução que nem a da questão
 abaixo, veja só:
 
 S = sen(a) + sen(2a) + sen(3a) + ... + sen(na)
 
 Em que a solução consiste em multiplicar ambos os lados da igualdade pelo
 seno da metade da razão da PA, e após efetuar a prostaférese e sair
 cortando. Sem muitas delongas!
 ...
 
 Já ouvi dizer que a resolução que procuro existe, e está escrita em um tal
 livro russo chamado Lidski, problemas de PA, ou algo do gênero... mas
 nunca tive o privilégio de ter algum contato com essa obra. Alguém já ouviu
 falar?
 
 Atenciosamente
 ¡Thyago!
 
 
 
 - Original Message -
 From: Claudio Buffara [EMAIL PROTECTED]
 To: [EMAIL PROTECTED]
 Sent: Tuesday, August 12, 2003 9:58 AM
 Subject: Re: [obm-l] Ajuda
 
 
 Oi, Thyago:
 
 Vou te confessar uma coisa: usando a identidade 1 - cis(a) =
 -2isen(a/2)cis(a/2) e mais esse problema do IME, que alias eh uma
 propriedade classica (e, como voce mostrou, util!) das raizes n-esimas da
 unidade, voce chegou a uma solucao mais curta e elegante do que a que eu
 tinha em mente. Parabens!
 
 A minha ideia era separar os casos n par e n impar e fatorar x^n - 1 de
 duas
 maneiras diferentes:
 Primeiro:
 x^(2m) - 1 = (x^2 - 1)*(x^(2m-2) + x^(2m-4) + ... + x^4 + x^2 + 1)
 x^(2m+1) - 1 = (x - 1)*(x^(2m) + x^(2m-1) + ... + x^2 + x + 1)
 
 Depois:
 x^(2m) - 1 = (x^2 - 1)*PRODUTO(1=k=m-1)(x^2 - 2xcos(kpi/m)x + 1)
 x^(2m+1) - 1 = (x - 1)*PRODUTO(1=k=m)(x^2 - 2xcos(2kpi/(2m+1)) + 1)
 
 E depois, fazer x = 1 e igualar as expressoes obtidas, mas a sua solucao
 eh
 mais simples e, portanto, melhor.
 
 O passo que faltou na sua solucao foi mostrar explicitamente que
 (-i)^(m-1)*cis(pi/n)*cis(2pi/n)*...*cis((n-1)pi/n) = 1
 mas isso eh bem facil (apesar de nao ser evidente).
 
 Um abraco,
 Claudio.
 
 PS: Se essa sua solucao nao eh pratica, entao eu nao sei o que eh.
 Repare:
 voce tem um produto de senos de numeros em PA. Como voce propoe
 calcula-los?
 Puramento por meio de identidades trigonometricas, sem usar complexos? Boa
 sorte...
 
 on 12.08.03 00:45, Thyago at [EMAIL PROTECTED] wrote:
 
 Olá Cláudio,
 
 Obrigado pelas dicas  :-)
 
 Mas a resolução que eu fiz não foi nada prática não.
 Eu já utilizei todas estas propriedades e não consegui chegar em nada.
 Bom, só para esclarecer um pouco mais... vou colocar o exercício que
 gerou
 tal questão:
 
 
 (IME) Sejam 1, X2, X3, ..., Xn as raízes de x^n=1. Calcule: P = (1 -
 x2)(1-x3)...(1-xn).
 
 Fazendo uso de Briot-Rufini e fatoração de polinômios, conseguimos
 chegar
 facilmente na resposta P = n.
 Mas, utilizando o tratamento vetorial de números complexos com a fórmula
 1-cis(a) = -2isen(a/2)cis(a/2) chegamos em
 
 P = 2^(n-1) . S
 
 Onde S = sen(pi/n) . sen(2pi/n) . sen(3pi/n) . ... . sen[(n-1)pi/n]
 
 Daí, utilizando a resposta da primeira resolução com a resposta da
 segunda
 resolução temos que S = n/[2^(n-1) ]
 Dá para ver que esta demonstração para S não é nada prática.
 
 Você citou uma solução padrão para este tipo de problema. Qual seria?
 
 Aguardo resposta
 
 Atenciosamente
 ¡Thyago!
 
 - Original Message -
 From: Cláudio (Prática) [EMAIL PROTECTED]
 To: [EMAIL PROTECTED]
 Sent: Monday, August 11, 2003 2:19 PM
 Subject: Re: [obm-l] Ajuda
 
 
 Oi, Thyago:
 
 A solução padrão pra esse tipo de problema realmente envolve
 complexos e
 polinômios.
 
 Tentando resolver outros problemas similares, você vai perceber que
 complexos e polinômios são uma forma de resolução bastante natural.
 
 Os resultados básicos são os seguintes:
 1) Todo número complexo pode ser representado na forma R*(cos(a) +
 i*sen(a)), onde R é um real não negativo e a é um real qualquer
 (mas
 normalmente limitado ao intervalo [0, 2pi) ou então (-pi,pi]);
 2) e^(i*a) = cos(a) + i*sen(a): essa é a definição da função
 exponencial
 complexa, que permite, por exemplo, que você transforme sequências de
 senos
 e cossenos de números reais em PA em sequências de complexos em PG, que
 as
 vezes são mais fáceis de manipular;
 3) Um polinômio com coeficientes reais pode ser expresso como o produto
 de
 binômios da forma (x - b) e/ou trinômios da forma (x^2 - 2*R*cos(a)*x +
 R^2), onde a e b são números reais quaisquer e R é um real positivo.
 
 Um abraço,
 Claudio.
 
 
 - Original Message -
 From: dex [EMAIL PROTECTED]
 To: [EMAIL PROTECTED]
 Sent: Monday, August 11, 2003 11:05 AM
 Subject: [obm-l] Ajuda
 
 
 Olá pessoal
 
 Gostaria de saber uma boa demonstração para o exercício abaixo
 
 P = sen(pi/n) . sen(2pi/n) . sen(3pi/n) . ... . sen[(n-1)pi/n]
 

Re: [obm-l] MMC

2003-08-14 Por tôpico Domingos Jr.



126000 = 2^4 * 3² * 5³ * 7

sejam A, B inteiros, se mmc(A, B) = 126.000
então
A= 2^r1 * 3^r2 * 5^r3 * 7^r4
B= 2^s1 * 3^s2 * 5^s3 * 7^s4
se definirmos t[i] = max(r[i], s[i]), i = 1..4
então
mmc(A, B) = 2^t1 * 3^t2 * 5^t3 * 7^t4
t1 = 4, t2 = 2, t3 = 3, t4 = 1

vamos contar os pares que satisfazem isso:
max(r1, s1) = 4, existem9 possibilidades
max(r2, s2) = 2, existem 5 possibilidades
max(r3, s3) = 3, existem 7 possibilidades
max(r4, s4) = 1, existem 3 possibilidades
isso daria 9*5*7*3 maneiras, no entanto, estamos contando o par (126000, 
126000) uma única vez e todos os demais pares (A, B) duas vezes... ou seja, o 
total de pares é (9*5*7*3 + 1)/2 =472 pares...

[ ]'s

  - Original Message - 
  From: 
  Pedro Costa 
  To: [EMAIL PROTECTED] 
  Sent: Wednesday, August 13, 2003 9:49 
  AM
  Subject: [obm-l] MMC
  
  
  Quantos pares de 
  inteiros positivos A e B existem cujo mínimo múltiplo comum é 126000?
   ( Considere o par (A,B) como sendo o 
  mesmo que (B,A)
  -- Esta mensagem foi verificada pelo sistema de antivírus e 
  acredita-se estar livre de perigo. 
  


[obm-l] Re: [obm-l] OUTRAS Questes Esaex

2003-08-14 Por tôpico J Augusto Tavares




numerador--[(y^3 - 1) + (y^4 -1) + (y^6 - 1)]
denominador--(y^6 - 1)
vou fatorar 
(y^3 - 1) = (y -1)(y^2 + y + 1)
(y^4 -1) = (y -1)(y^3 +y^2 + y + 1)
(y^6 - 1) =(y^3 + 1)(y^3 -1)=(y^3+1)(y -1)(y^2 + y + 1)
agora:
numerador( vou colar (y-1) em evidencia): (y-1)[(y^2 + y + 1) + (y^3 
+y^2 + y + 1) + (y^3+1)(y^2 + y + 1)]
denominador: (y^3+1)(y -1)(y^2 + y + 1),
eliminando y-1 de ambos e substituindo y por 1,
numerador: 13, denominador:6, ou seja, 13/6 esse limite!
se eu 
nao errei nada, eh claro! eheheh 
 
Abracao 
 
Guto.
obs.: 
eu so nao tinha efetuado as contas, mas daria um resultado sim ...!





  
  Resposta: 
   Fazendo (x+1) = y^12 , como 
  x-0, y-1.
  (y^3 + y^4 + y^6 - 3)/(y^6 - 1) ,[(y^3 - 
  1) + (y^4 -1) + (y^6 - 1)]/[(y^3 + 1)(y^3 -1)]
  eleminando o fator (y-1),nao existira mais 
  a indeterminacao ! 
  
  Esta fatorao vai te levar novamente a 
  (y^2 + 1)(y + 1)(y - 1) / (y + 1)(y2 -y + 1)(y - 1)(y^2 + y + 
  1)
  Faremos um bocado de conta e o resultado 
  no bate
  
  


[obm-l] OUTRAS Questes Esaex

2003-08-14 Por tôpico Joo



4) NO CONSIGO 
FATORAR A RAIZ CBICA! 

O valor do 
limite quando x  0 de
 
4(x+1) + 3(x+1) + (x+1)  
3
 
(x+1)  1


Resposta: 
 Fazendo (x+1) = y^12 , como 
x-0, y-1.
(y^3 + y^4 + y^6 - 3)/(y^6 - 1) ,[(y^3 - 1) 
+ (y^4 -1) + (y^6 - 1)]/[(y^3 + 1)(y^3 -1)]
eleminando o fator (y-1),nao existira mais a 
indeterminacao ! 

Esta fatorao vai te levar novamente a 
(y^2 + 1)(y + 1)(y - 1) / (y + 1)(y2 -y + 1)(y - 1)(y^2 + y + 
1)
Faremos um bocado de conta e o resultado 
no bate


QUESTO NOVA:

De quantas maneiras se pode colocar 3 anis em 5 
dedos?


Re: [obm-l] Livro da OBM

2003-08-14 Por tôpico Anderson Sales Pereira
Ola Nicolau,

Você sabe detalhes a respeito de valor e formato do livro?

Anderson

At 16:38 7/8/2003 -0300, Nicolau C. Saldanha wrote:
On Fri, Aug 08, 2003 at 03:01:12AM -0700, Olimpiada Brasileira de 
Matematica wrote:
 Livro Olimpiadas Brasileiras de Matematica
 9a. a 16a. , problemas e resolucoes.

 Organizadores: Carlos Moreira (Gugu), Edmilson Motta,
 Eduardo Tengan, Luiz Amancio, Nicolau Saldanha,
 Paulo Rodrigues.

 Uma publicacao de:
 Olimpiada Brasileira de Matematica
 Sociedade Brasileira de Matematica

 172 paginas

 Vendas e distribuicao:
 Sociedade Brasileira de Matematica
 Tel: (21)25295073 - (21)25295072
 e-mail: [EMAIL PROTECTED]

A SBM também tem uma home page, www.sbm.org.br,
mas não garanto que o livro já esteja lá.
[]s, N.
=
Instruções para entrar na lista, sair da lista e usar a lista em
http://www.mat.puc-rio.br/~nicolau/olimp/obm-l.html
=
Esta mensagem foi verificada pelo E-mail Protegido Terra.
Scan engine: VirusScan / Atualizado em 06/08/2003 / Versão: 1.3.13
Proteja o seu e-mail Terra: http://www.emailprotegido.terra.com.br/


=
Instruções para entrar na lista, sair da lista e usar a lista em
http://www.mat.puc-rio.br/~nicolau/olimp/obm-l.html
=


Re: [obm-l] Livro da OBM

2003-08-14 Por tôpico Nicolau C. Saldanha
On Fri, Aug 08, 2003 at 03:01:12AM -0700, Olimpiada Brasileira de Matematica wrote:
 Livro Olimpiadas Brasileiras de Matematica
 9a. a 16a. , problemas e resolucoes.
 
 Organizadores: Carlos Moreira (Gugu), Edmilson Motta,
 Eduardo Tengan, Luiz Amancio, Nicolau Saldanha,
 Paulo Rodrigues.
 
 Uma publicacao de:
 Olimpiada Brasileira de Matematica
 Sociedade Brasileira de Matematica
 
 172 paginas
 
 Vendas e distribuicao:
 Sociedade Brasileira de Matematica
 Tel: (21)25295073 - (21)25295072
 e-mail: [EMAIL PROTECTED]

A SBM também tem uma home page, www.sbm.org.br,
mas não garanto que o livro já esteja lá.

[]s, N.
=
Instruções para entrar na lista, sair da lista e usar a lista em
http://www.mat.puc-rio.br/~nicolau/olimp/obm-l.html
=


Re: [obm-l] Opção_de_Compra

2003-08-14 Por tôpico Helder Suzuki
 --- Claudio Buffara [EMAIL PROTECTED]
escreveu:
  
  Oi Claudio :)
  
  Tem razão!
  
  Vamos ver,
  Se eu compro a ação de hoje por R$ 100 e guardo
 até o
  ano que vem e o preço da opção é x:
  terei que pegar emprestado R$ (100-x), já que x
 deve
  ser menor que R$ 75.
  
  então daqui um ano pode acontecer:
  (i) a ação valer R$ 200
  nesse caso, o comprador da opção irá comprar a
 ação
  por R$ 110, que deverei usar para pagar o
 empréstimo
  
  (ii) a ação valer R$ 50
  nesse caso, o comprador da opção não irá comprar a
  ação, e eu ainda terei que pagar o empréstimo,
  então eu vendo a ação por R$ 50 e pago o
 empréstimo.
  logo, para não ter prejuíso:
  (100-x)*1,2 = 50 = 100-x = 60 = x = R$ 40
  
  então 40 = x = 75
  agora voltando ao caso (i):
  receberei R$ 110 reais, e o meu lucro será:
  l = 110 - (100-x)*1,2 = 110 - 120 + 1,2x = 1,2x -
 10
  
  se x for R$ 40, l = R$ 8
  o que é excelente, pois nesse caso o comprador
 pode
  lucrar 200-110-40 = R$ 50 fazendo com que a opção
 não
  seja tão ruim de se comprar caso p seja grande, e
 eu
  ainda poderei ter um lucro de R$ 8 reais, sendo
 que no
  começo da operação eu não tinha nenhum
 tostão(peguei
  tudo emprestado do banco).
  e o melhor de tudo, é garantido que eu não terei
  nenhum prejuíso!
  
  Ainda poderia escolher valores maiores de x,
  dependendo de p.
  
  []'s,
  Hélder T. Suzuki
  
 Oi, Helder:
 
 Tudo bem pro vendedor da opcao!
 
 Mas pense no caso do comprador da opcao. Se a acao
 cair pra R$ 50, ele terah
 perdido todo o premio que pagou (premio eh o
 jargao do mercado para o
 preco de uma opcao). Assim, um premio entre R$ 40 e
 R$ 75 nao seria justo.
 
 Mas voce estah no caminho certo.
 
 A fim de se calcular o preco justo da opcao - isto
 eh, o preco no qual tanto
 o comprador quanto o vendedor saem no zero-a-zero -
 voce tem que fazer duas
 suposicoes:
 1) A fim de se proteger contra uma alta no preco da
 acao, o vendedor da
 opcao tem de comprar a acao. Isso voce fez.
 2) A fim de se proteger contra uma queda no preco da
 acao, o comprador da
 opcao precisa vender a acao. No mercado financeiro
 isso ocorreria da
 seguinte forma: ele tomaria a acao emprestada e a
 venderia no mercado,
 aplicando os reais provenientes da venda a juros. Ao
 fim de um ano, ele
 precisaria recomprar a acao para devolver ao dono
 (que alugou a acao pra
 ele).
 
 O truque eh determinar exatamente quantas acoes
 precisam ser compradas ou
 vendidas pro resultado financeiro no fim de 1 ano
 ser zero em qualquer
 hipotese (acao a R$ 200 ou acao a R$ 50). Da forma
 que o enunciado estah
 escrito, pode parecer que a acao eh um objeto
 indivisivel, mas este nao eh o
 caso. Ou seja, pra esse problema suponha que voce
 pode comprar ou vender a
 acoes, onde a eh um numero real qualquer entre 0 e
 1.
 
 No mundo real, uma operacao envolveria tipicamente
 10.000 ou mesmo 100.000
 acoes == o montante financeiro equivalente seria de
 R$ 1 milhao ou R$ 10
 milhoes. No entanto, para uma acao de R$100, o lote
 padrao para negociacao
 seria de 50 ou 100 acoes - R$ 5.000 a R$ 10.000.
 
 Um abraco,
 Claudio.

Olá Claudio!

Interessante isso! :)
Se o comprador pega a ações emprestado, ele terá que
devolver daqui um ano a ações mais 100*a*0,2 (20% do
valor do empréstimo), ou seja, a ações + 20a
se ele vender essa ação emprestada, ele embolsará
100*a reais
então, supondo que x  100a:
comprando a opção, sobrará 100a-x, o que ele poderá
investir, obtendo (100a-x)*1,2 em 1 ano.
ai temos dois casos:

(i) a ação está valendo R$50
ele deverá comprar a ações por 50*a, para devolver a
ações + 20a para o cara que emprestou as ações.
nesse caso, ele gastará R$ 70a
o que ele deve ter, para não ter prejuíso, então:
(100a-x)*1,2 = 70a = 100a-x = 84a = 16a = x =
a = x/16

(ii) a ação está valendo R$ 200
ele exerce a opção e compra 1 ação por R$ 110,
mas ele tem que devolver a ações, e ainda pagar 20a
reais.
então lhe sobra 200 - 200a - 20a = 200-220a
para ele não ter prejuíso: 200-220a = 0 =
200 = 220a = 20 = 22a = 10/11 = a
então:

x/16 = a = 10/11
mas para o vendedor não ter prejuíso, x = 40
será que o vendedor também deve pegar emprestado a
ação, em vez de emprestar dinheiro do banco, para
depois comprar a ação?

Abraços,
Hélder

___
Conheça o novo Cadê? - Mais rápido, mais fácil e mais preciso.
Toda a web, 42 milhões de páginas brasileiras e nova busca por imagens!
http://www.cade.com.br
=
Instruções para entrar na lista, sair da lista e usar a lista em
http://www.mat.puc-rio.br/~nicolau/olimp/obm-l.html
=


[obm-l] Exemplo contra-intuitivo

2003-08-14 Por tôpico Claudio Buffara
on 10.08.03 00:50, Artur Costa Steiner at [EMAIL PROTECTED] wrote:


 Eu, por exemplo, acho um tanto contra intuitivo que o fato de f ser
 diferenciavel  em R e apresentar limite no infinito nao implique
 que f' apresente limite zero no infinito.
 Artur
 

Oi, Artur:

Quando li sua mensagem tambem fiquei com a mesma impressao. Entao, fui olhar
no livro Counterexamples in Analysis e achei um exemplo bem elementar:

f(x) = sen(x^2)/x  ==  f'(x) = 2*cos(x^2) - sen(x^2)/x^2

Naturalmente, lim(x - +inf) f(x) = 0 mas lim(x - +inf) f'(x) nao existe.

Acho que uma explicacao seria a seguinte: a derivada de uma funcao mede a
taxa de variacao dessa funcao em relacao ao seu argumento, certo? No caso de
f(x) acima, a variacao tem duas componentes: o decaimento da amplitude
(dado por 1/x) e o zig-zag (dado por sen(x^2)).

No caso de f(x), quando x - infinito, o crescimento da frequencia do
zig-zag eh mais rapido do que a reducao da amplitude (x^2 contra 1/x). Logo,
eh de se esperar que a derivada nao tenda a zero.

De fato, se tomarmos um exemplo mais vivido:

g(x) = sen(x^3)/x == g'(x) = 3*x*cos(x^3) - sen(x^3)/x^2.

veremos que, quando x cresce sem limite, a derivada nao soh nao se anula,
como tambem assume valores arbitrariamente grandes (positivos e negativos).

Nao sei se essa explicacao torna o fenomeno intuitivo, mas pelo menos pra
mim, joga o pepino pra definicao de derivada e pro vilao de sempre - o
infinito.

*

Serah que existe alguma f tal que:
f(x) tende a zero
e
f'(x) tende a L (diferente de zero)
quando x - infinito?

*

Acho que um sinal de que estamos fazendo progresso no nosso entendimento de
matematica (e de qualquer outro assunto) eh o fato de passarmos a achar
natural algo que antes parecia contra-intuitivo. Quanto maior esse
entendimento, menor o numero de fatos que permanecem contra-intuitivos.

O que eh meio chato eh eu ter levado uns 5 minutos pra pensar nos 10
exemplos da minha msg anterior, mas pelo menos eu tenho uma vaga ideia da
extensao da minha ignorancia.


Um abraco,
Claudio.

=
Instruções para entrar na lista, sair da lista e usar a lista em
http://www.mat.puc-rio.br/~nicolau/olimp/obm-l.html
=


Re: [obm-l] ENQUETE - BELEZA MATEMATICA

2003-08-14 Por tôpico A. C. Morgado
Minhas escolhas são, como pedido, bem em nível de ensino médio e revelam 
minha admiração pela simplicidade e pela surpresa.

1) O conjunto dos primos eh infinito. Incluído pela beleza da prova de 
Euclides.
2) Desigualdade das médias aritmética e geométrica. Incluída pela beleza 
da prova de Cauchy.
3) As alturas de um triângulo concorrem em um mesmo ponto. (Sei que vão 
achar surpreendente essa minha indicação, mas é um resultado que 
conhecemos desde pequenos e, em geral, não nos damos conta de quão 
surpreendente ele é, nem tampouco da engenhosidade da demonstração.)
4) O problema dos pontos. Pela beleza da solução de Fermat.
5) São apenas 5 os poliedros regulares. (Outro que, em geral, não nos 
damos conta de quão surpreendente ele é.)

Claudio Buffara wrote:

Caros colegas da lista:

Gostaria de contar com sua participacao numa enquete sobre beleza
matematica.
O que eu precisao eh que cada um de voces me envie uma lista contendo algo
como 5 a 10 problemas/teoremas que voces consideram os mais bonitos e cujas
solucoes/demonstracoes sao as mais elegantes e/ou inusitadas e/ou
engenhosas. Nao precisa incluir a solucao/demonstracao, apenas o enunciado.
No entanto, se voce tiver em mente uma solucao/demonstracao especifica
(entre varias existentes) nao deixe de mencionar pelo menos o metodo
utilizado. 

A unica restricao eh que estes resultados devem ser de um nivel acessivel a
um aluno normal de 2o. grau (ou seja, o Ultimo Teorema de Fermat e o Porisma
de Poncelet estao fora, mas o caso n = 4 do UTF e a versao para triangulos
do Porisma poderiam ser incluidos).
Importante: os resultados devem ser acessiveis a um aluno normal de 2o.
grau, mas nao necessariamente fazer parte do curriculo normal do 2o. grau.
Tambem nao precisa responder hoje ou amanha ou mesmo na semana que vem. Acho
que vale a pena pensar por um tempo e consultar a literatura - as vezes pode
ter um resultado belissimo do qual voce simplesmente se esqueceu por nao
encontra-lo ha muito tempo. As Eurekas sao uma otima referencia. O Proofs
from the Book tambem, apesar de nem tudo lah ter nivel de 2o. grau.
Se houver um numero suficiente de respostas, eu me comprometo a publicar uma
compilacao dos problemas e teoremas mais votados.
Desde jah a gradeco o interesse de quem quiser participar.

Um abraco,
Claudio.
=
Instruções para entrar na lista, sair da lista e usar a lista em
http://www.mat.puc-rio.br/~nicolau/olimp/obm-l.html
=
 

=
Instruções para entrar na lista, sair da lista e usar a lista em
http://www.mat.puc-rio.br/~nicolau/olimp/obm-l.html
=


Re: [obm-l] Problema das 3 portas

2003-08-14 Por tôpico Eduardo Casagrande Stabel
Oi Bernardo.

Por favor, leia a última mensagem enviado por Camilo Marcantonio Junior,
onde ele explica corretamente o problema. Há muitas pessoas que,  mesmo
depois de ler os argumentos que justificam que é melhor TROCAR DE PORTA, não
se convencem e continuam a insistir que tanto faz trocar ou não trocar de
porta. Posso lhe assegurar que a resposta correta (todos as pessoas sérias
dessa lista, grandes matemáticos: Nicolau, Gugu, Morgado, Luis Lopes, Shine,
Camilo, Paulo Santa Rita, etc. concordarão que o melhor é trocar de porta) é
esta.

Se você, ainda sim, não conseguir compreender o argumento, lhe sugiro para
fazer o seguinte experimento. Se você souber programar em computador, faça
um programa que escolha aleatoriamente uma dentre três opções (a premiada) e
lhe pede para decidir uma delas (1, 2 ou 3). Depois ele mostra que um dos
números que você escolheu não contém o prêmio. Por fim, ele diz se você
ganha permanecendo na mesma porta ou se trocando de porta. E ele faz uma
contagem. Repita este jogo, umas 100 vezes e você perceberá que em
aproximadamente 67 casos você teria ganho TROCANDO de porta e em
aproximadamente 33 casos você ganharia PERMANECENDO com a mesma porta. Isto
tem de lhe convencer.

Se você não souber programar, sugiro que pegue três copos (não
transparentes) e uma bolinha de papel que é o prêmio. Peça para alguém ter a
função do apresentador do programa, e vá você mesmo fazendo a contagem que
lhe sugeri. Repita umas 100 vezes o jogo, e constate a proporção
(aproximada) de 2/3 para 1/3.

Mas faça mesmo essa experiência, antes de enviar uma outra mensagem à lista,
ok?

Abraço,
Duda.

From: Bernardo Vieira Emerick [EMAIL PROTECTED]
 Claudio,
 Eu insisto que tanto faz trocar de porta. Pensemos no problema em duas
 etapas. Na primeira você escolhe entre três portas. Atrás de uma está o
 prêmio. A probabilidade de você ganhar será de 1/3, certo? Na segunda,
você
 tem que escolher entre duas portas. O prêmio está em uma delas. A sua
 probabilidade de ganhar será, portanto, 1/2 para as duas portas. Pouco
 importa o que você escolheu na primeira etapa. É como se fosse outro jogo,
 só que se tenha eliminada uma das opções erradas.


 From: Claudio Buffara [EMAIL PROTECTED]
 Reply-To: [EMAIL PROTECTED]
 To: [EMAIL PROTECTED]
 Subject: [obm-l] Problema das 3 portas
 Date: Tue, 12 Aug 2003 00:43:58 -0300
 
 Oi, Henrique:
 
 Eu insisto que a estrategia otima eh trocar de porta.
 
 Veja o meu raciocinio:
 
 Chame as 3 portas de A, B e C.
 Suponha s.p.d.g. que inicialmente voce escolhe a porta A.
 
 Temos 3 casos a considerar:
 1) O premio estah atras de A:
 Nesse caso, o apresentador abre B ou abre C (qualquer uma das duas
estarah
 vazia)
 Se voce trocar, voce estarah saindo da porta vencedora e indo para uma
das
 perdedoras (a que ele nao abriu) - voce perde se trocar.
 
 2) O premio estah atras de B:
 Nesse caso, o apresentador abre a porta C.
 Se voce trocar, voce estarah saindo de A e indo para B - a porta
vencedora.
 Ou seja, voce ganha se trocar.
 
 3) O premio estah atras de C:
 Nesse caso, o apresentador abre a porta B.
 Se voce trocar, voce estarah saindo de A e indo para C - a porta
vencedora.
 Ou seja, voce ganha se trocar.
 
 Assim, ao decidir trocar voce perde em um caso e ganha em 2. Supondo que
a
 probabilidade do premio estar atras de uma dada porta eh 1/3, a sua
 probabilidade de ganhar ao trocar eh igual a 2/3  1/2. Logo, voce deve
 trocar de porta.
 
 Com 1 milhao de portas, a decisao eh ainda mais obvia, pois se voce nao
 trocar, o que voce estarah dizendo eh que voce escolheu a porta certa de
 primeira, um evento que pra voce tem uma probabilidade de 1 em 10^6.
 
 Suponha que voce tenha escolhido inicialmente a porta no. 1, a qual tem,
 pra
 voce, probabilidade de 1/10^6 de conter o premio.
 Isso quer dizer que, pra voce, a probabilidade do premio estar atras de
uma
 das outras 999.999 portas eh de 999.999/10^6.
 
 Quando o apresentador abre 999.998 portas dentre as 999.999 que voce nao
 escolheu, ele estah colapsando a probabilidade de cada porta aberta para
0,
 e concentrando a probabilidade total de 999.999/10^6 numa unica porta,
que
 permanece fechada (estas probabilidades sao sempre do seu ponto de vista.
 Do
 ponto de vista do apresentador, que sabe qual a porta premiada, as
 probabilidades sao: 1 do premio estar atras da porta premiada e 0 de
estar
 atras de qualquer outra).
 
 Nesse caso, voce seria louco de nao trocar de porta.
 
 Um abraco,
 Claudio.
 
 on 11.08.03 23:27, Henrique Patrício Sant'Anna Branco at
 [EMAIL PROTECTED] wrote:
 
   Por mais que eu ache pedante e ridiculo alguem se vangloriar de ter o
 QI
   mais alto do mundo, nesse caso acho que a Marilyn estah certa. Voce
 deve
   trocar de porta.
  
   Desculpem a minha ignorancia, mas o que ha de errado com o argumento
de
 1
   milhao de portas? Me parece que, nesse caso, a probabilidade de voce
 ter
   escolhido a porta certa de primeira eh apenas de 1/1.000.000. Logo, a
   probabilidade da outra porta 

Re: [obm-l] ENQUETE - BELEZA MATEMATICA

2003-08-14 Por tôpico Domingos Jr.
O Noga Alon conta que fizeram esta pergunta para ele uma vez que ele
começou explicando a prova de Euclides de que há infinitos primos
em um programa de televisão, eu acho:

And today, are there still infinitely many primes?

E sem sair do clima, deem uma olhada em
http://qsilver.queensu.ca/~phil158d/intro/montyh3.htm

Eu deveria ter visto isso antes de escrever o meu artigo da Eureka!

--- x ---
Putz, essa mulher do QI mais alto do mundo (bullshit!) não concorda com o
Princípio da Indução Finita também! hehehe, o pior é que é sério!!!

=
Instruções para entrar na lista, sair da lista e usar a lista em
http://www.mat.puc-rio.br/~nicolau/olimp/obm-l.html
=


Re: [obm-l] Problemas em Aberto - Algarismos

2003-08-14 Por tôpico Claudio Buffara
Title: Re: [obm-l] Problemas em Aberto - Algarismos



on 05.08.03 19:03, Domingos Jr. at [EMAIL PROTECTED] wrote:

Uma idéia para o segundo:
Considere, SPG, j  i, tq:
2^j = a0 + a1*10 + ... + a[k]*10^k
e f uma permutação tq.
2^i = f(a0) + f(a1)*10 + ... + f(a[k])*10^k
 
então
2^j - 2^i = a0 - f(a0) + [a1 - f(a1)]*10 + ... + [a[k] - f(a[k])]*10^k
logo 
2^j - 2^i ~ a0 - f(a0) + ... + a[k] - f(a[k]) = 0 (mod 9)
2^i[2^(j-i) - 1] = 0 (mod 9) = j - i = 6k para algum k
 
será que sai alguma coisa a partir daqui?
o que fiz até aqui já mostra que a permutação tem que colocar pelo menos 1 zero a esquerda...

Pois eh. O problema eh justamente se:
2^(i+6k) = a b c d 0 0 0 e f g 
e
2^i = f g a b d c 
ou algo do genero.




2) Prove ou disprove: existe uma potencia de 2 tal que ao se permutar os algarismos de sua representacao decimal obtem-se uma outra potencia de 2.

Esse segundo tem uma solucao aparentemente simples, mas esta solucao exclui o caso de potencias de 2 com algarismos 0 internos (ou seja, numeros do tipo abcdefg). 

Um abraco,
Claudio.








Re: [obm-l] geometria

2003-08-14 Por tôpico Fábio Dias Moreira
-BEGIN PGP SIGNED MESSAGE-
Hash: SHA1

Em Wednesday 13 August 2003 12:13, Fábio Dias Moreira escreveu:
 [...]
 Note que isso também vale para tetraedros -- se 6s é a área superficial do
 tetraedro, e r é o raio da esfera inscrita, então [ABCD] = s * r.
 [...]

Correção: [...] se ***3s*** é a área superficial [...]

(Obrigado, Cláudio!)

[]s,

- -- 
Fábio ctg \pi Dias Moreira
-BEGIN PGP SIGNATURE-
Version: GnuPG v1.2.2 (GNU/Linux)

iD8DBQE/OwNQalOQFrvzGQoRAvZnAKDHvUIh5K3BbGl8QvIsyoZuAQS0UgCfdqDd
5QfZyJ9Xmb7bXlFXmLWyb5U=
=x416
-END PGP SIGNATURE-

=
Instruções para entrar na lista, sair da lista e usar a lista em
http://www.mat.puc-rio.br/~nicolau/olimp/obm-l.html
=


Re: [obm-l] Número de soluções de sistema linear - Correção

2003-08-14 Por tôpico Alexandre Daibert




Tive uma dvida nessa resoluo. Depois de tudo feito, faltando
calcular o coeficiente de x. supondo para um problema menor, como
calcularamos o coeficiente de x no grau 23 brao na expresso fatorada
do tipo
(1+x^16)*(x^24-1)^4/((x^8-1)*(x^4-1)*(x^2-1)*(x-1))

 q no podemos desmembr-la, pois voltaramos ao problema inicial.

obs: desculpe minha ignorncia, mas sou um mero pobre, ignorante e
humilde vestibulando...
 :-P 


Claudio Buffara escreveu:

  on 07.08.03 01:38, Alexandre Daibert at [EMAIL PROTECTED] wrote:

...
  
  
O problema  determinar o nmero de solues
inteiras no negativas do sistema:
16a + 8b + 4c + 2d + e = 23

  
  ...

Oi, Alexandre:

A solucao classica pra esse tipo de problema eh via series formais (vide
artigo do Eduardo Tengan na Eureka 11).

No caso, nem precisamos usar series infinitas, mas apenas polinomios.
Precisamente, voce estah interessado no coeficiente de x^23 do polinomio
formal:

f(x) = a(x)*b(x)*c(x)*d(x)*e(x), onde:

a(x) = 1 + x^16
b(x) = 1 + x^8 + x^16 = (x^24-1)/(x^8-1)
c(x) = 1 + x^4 + x^8 + x^12 + x^16 + x^20 = (x^24-1)/(x^4-1)
d(x) = 1 + x^2 + x^4 + x^6 + ... + x^20 + x^22 = (x^24-1)/(x^2-1)
e(x) = 1 + x + x^2 + x^3 + ... + x^21 + x^22 + x^23 = (x^24-1)/(x-1)

Voce consegue ver o porque disso?

Logo:
f(x) = (1+x^16)*(x^24-1)^4/((x^8-1)*(x^4-1)*(x^2-1)*(x-1))

Pondo esta expressao para f(x) (que de fato eh um polinomio de grau 97) para
ser avaliada pelo PARI-GP, eu achei que o coeficiente de x^23 eh igual a 74.

Logo, existm 74 solucoes inteiras nao-negativas para a sua equacao.

Naturalmente, Mathematica, Matlab ou Maple tambem podem ser usados. O que eu
nao recomendo eh fazer na mao. Nao soh ha uma grande chance de voce errar
alguma conta, mas tambem voce vai ficar de saco tao cheio que corre o risco
de comecar a odiar matematica e abondonar esta bela ciencia pela razao
errada.

O PARI-GP eh um software de matematica (especialmente teoria dos numeros)
que pode ser baixado gratuitamente da internet.
O site eh este aqui:
http://www.parigp-home.de/

Um abraco,
Claudio.

=
Instrues para entrar na lista, sair da lista e usar a lista em
http://www.mat.puc-rio.br/~nicolau/olimp/obm-l.html
=


  






[obm-l] Re: [obm-l] Re: [obm-l]_Re:_[obm-l]_ENQUETE_-_BELEZA_MATEMÁTICA

2003-08-14 Por tôpico Eduardo Casagrande Stabel
Oi Dirichlet,

o Nicolau não comentou sobre nenhum problema. De qual problema em aberto
você está falando?

Abraço,
Duda.

From: Johann Peter Gustav Lejeune Dirichlet
[EMAIL PROTECTED]
 A demonstraçao que 3eu escrevi evitara este
 mal-entendido.
 Alias o Tengan me disse que este e um problema em
 aberto muito chato e de que ninguem conseguiu uma
 ideia muito esperançosa...

  --- Nicolau C. Saldanha
 [EMAIL PROTECTED] escreveu:  On
 Wed, Aug 13, 2003 at 09:39:49PM -0200,
  Claudio Buffara wrote:
   on 13.08.03 20:28, Henrique Patrício
  Sant'Anna Branco at
   [EMAIL PROTECTED] wrote:
  
Cláudio,
   
A classica prova de Euclides é aquela que
  diz:
Sejam p1, p2, ..., pm todos os primos.
  Entao consideremos o número N = p1 *
p2 * ... * pm + 1. Esse número não seria
  divisível por nenhum primo e,
portanto, contradiz o Teorema Fundamental
  da Aritmetica?
   
Abraços,
Henrique.
   
   Eh isso ai mesmo.
 
  Por falar nisso, esta prova aparentemente induz
  um dos erros mais comuns.
  As pessoas incorretamente entendem que foi
  provado que 2*3*...*p + 1 é primo.
  Isto é falso mas o primeiro contraexemplo
  demora o suficiente para aparecer
  para convencer os mais afoitos de que sim,
  estes números são primos:
  2*3*5*7*11*13+1 = 30031 = 59*509
  2*3*5*7*11*13*17+1 = 510511 = 19*97*277
  2*3*5*7*11*13*17*19+1 = 9699691 = 347*27953
  2*3*5*7*11*13*17*19*23+1 = 223092871 =
  317*703763
  Também dá errado se trocarmos +1 por -1 no
  final:
  2*3*5*7-1 = 209 = 11*19
  2*3*5*7*11*13*17-1 = 510509 = 61*8369
  2*3*5*7*11*13*17*19-1 = 9699689 = 53*197*929
  2*3*5*7*11*13*17*19*23-1 = 223092869 =
  37*131*46027
 
  Desculpem, eu sei que ninguém perguntou, mas eu
  já ouvi este erro
  vezes demais.
 
  []s, N.
 
 
 
 =
  Instruções para entrar na lista, sair da lista
  e usar a lista em
 
 http://www.mat.puc-rio.br/~nicolau/olimp/obm-l.html
 
 =

 ___
 Conheça o novo Cadê? - Mais rápido, mais fácil e mais preciso.
 Toda a web, 42 milhões de páginas brasileiras e nova busca por imagens!
 http://www.cade.com.br
 =
 Instruções para entrar na lista, sair da lista e usar a lista em
 http://www.mat.puc-rio.br/~nicolau/olimp/obm-l.html
 =



=
Instruções para entrar na lista, sair da lista e usar a lista em
http://www.mat.puc-rio.br/~nicolau/olimp/obm-l.html
=


Re: [obm-l] ENQUETE - BELEZA MATEMATICA

2003-08-14 Por tôpico A. C. Morgado
Desculpe a ignorancia, poderia detalhar mais a segunda escolha?

Paulo Santa Rita wrote:

Ola Claudio !

Muito legal essa sua enquete. Bom, so pode entrar resultados 
elementares e/ou de facil compreensao, certo ? Entao me ocorre de 
imediato alguns resultados.

PRIMEIRO ( trivial, mas mercece um quadro na parece. Devido a Bernoulli )

1^P + 2^P + 3^P + ... + (N-1)^P + N^P = [(N+B)^P  -  B^P]/(P+1)
onde B^k deve ser interpretado como o K-esimo numero de bernoulli.
Alias, foi verificando as somas das potencias P-esimas dos numeros 
naturais que Bernoulli descobriu os fantasticos numeros que hoje levam 
o seu nome. Mais adiante, quando eu estiver mais tranquilo, vou 
escrever sobre este tema.

SEGUNDO ( Isso nao e um principio, e um Salmo do Profeta. Devido a 
Erdos )

Se em um conjunto de objeto, um objeto tem uma probabilidade menor 
que 1 de ter uma determinada propriedade, entao existe um objeto do 
conjunto com aquela propriedade

Esse principio, nao obstante muito contestado e criticado por alguns, 
e poderoso e acredito que abre novas e imensas possibilidades para o 
pensamente matematico.

TERCEIRO ( trivial, mas facilita a prova de muitas coisas. A 
desigualdade Eduardo Wagner )

Em todo triangulo, o semi-perimetro nunca e menor que a soma dos 
produtos de cada lado pelo cosseno do angulo oposto

p = a*cosA + b*cosB + c*cosC

Com a desigualdade acima da pra derivar quase todas as desigualdades 
complicadas da Geometria Elemntar.

Um Abraco
Paulo Santa Rita
7,1425,090803
EM TEMPO. Sobre a beleza matematica :

A Divina Proporcao
Um Ensaio sobre a beleza na Matematica
H. E. Huntley
Editora UnB
O autor mostra como o numero fi, ( 1 + raiz_quadrada(5) )/2, aparece 
nas mais diversas circunstancias e inesperadas circunstancias, sempre 
com um toque de inegavel beleza. Eu acredito que este numero 
contribuem pelo menos com um resultado :

A UNICA progressao geometrica de termos positivos que na qual An+1 = 
An + An-1 e a sequencia :
1, fi, fi^2, fi^3, fi^4, ...


From: Claudio Buffara [EMAIL PROTECTED]
Reply-To: [EMAIL PROTECTED]
To: Lista OBM [EMAIL PROTECTED]
CC: Claudio Buffara [EMAIL PROTECTED]
Subject: [obm-l] ENQUETE - BELEZA MATEMATICA
Date: Sat, 09 Aug 2003 10:24:26 -0300
Caros colegas da lista:

Gostaria de contar com sua participacao numa enquete sobre beleza
matematica.
O que eu precisao eh que cada um de voces me envie uma lista contendo 
algo
como 5 a 10 problemas/teoremas que voces consideram os mais bonitos e 
cujas
solucoes/demonstracoes sao as mais elegantes e/ou inusitadas e/ou
engenhosas. Nao precisa incluir a solucao/demonstracao, apenas o 
enunciado.
No entanto, se voce tiver em mente uma solucao/demonstracao especifica
(entre varias existentes) nao deixe de mencionar pelo menos o metodo
utilizado.

A unica restricao eh que estes resultados devem ser de um nivel 
acessivel a
um aluno normal de 2o. grau (ou seja, o Ultimo Teorema de Fermat e o 
Porisma
de Poncelet estao fora, mas o caso n = 4 do UTF e a versao para 
triangulos
do Porisma poderiam ser incluidos).

Importante: os resultados devem ser acessiveis a um aluno normal de 2o.
grau, mas nao necessariamente fazer parte do curriculo normal do 2o. 
grau.

Tambem nao precisa responder hoje ou amanha ou mesmo na semana que 
vem. Acho
que vale a pena pensar por um tempo e consultar a literatura - as 
vezes pode
ter um resultado belissimo do qual voce simplesmente se esqueceu por nao
encontra-lo ha muito tempo. As Eurekas sao uma otima referencia. O 
Proofs
from the Book tambem, apesar de nem tudo lah ter nivel de 2o. grau.

Se houver um numero suficiente de respostas, eu me comprometo a 
publicar uma
compilacao dos problemas e teoremas mais votados.

Desde jah a gradeco o interesse de quem quiser participar.

Um abraco,
Claudio.
= 

Instruções para entrar na lista, sair da lista e usar a lista em
http://www.mat.puc-rio.br/~nicolau/olimp/obm-l.html
= 



_
MSN Hotmail, o maior webmail do Brasil.  http://www.hotmail.com
=
Instruções para entrar na lista, sair da lista e usar a lista em
http://www.mat.puc-rio.br/~nicolau/olimp/obm-l.html
=

=
Instruções para entrar na lista, sair da lista e usar a lista em
http://www.mat.puc-rio.br/~nicolau/olimp/obm-l.html
=


Res: [obm-l] probabilidade

2003-08-14 Por tôpico Oswaldo Stanziola






{p1,p2,b1,b2,a1,a2,c1,c2,p'1,p'2,b'1,b'2,a'1,a'2,c'1,c'2}
total 16 elementos

---Mensagem original---


De: [EMAIL PROTECTED]
Data: quinta-feira, 7 de agosto de 2003 09:07:02
Para: [EMAIL PROTECTED]
Assunto: [obm-l] probabilidade

considerando o experimento aleatório o nascimento de 2
gatos, qual o número de elementos do espaço amostral
considerando que os gatos podem ser macho ou fêmea,
nas cores preto, branco, amarelo ou cinza.

a) n(U) = 8
b) n(U) = 16
c) n(U) = 12
d) n(U) = 14


___
Conheça o novo Cadê? - Mais rápido, mais fácil e mais preciso.
Toda a web, 42 milhões de páginas brasileiras e nova busca por imagens!
http://www.cade.com.br
=
Instruções para entrar na lista, sair da lista e usar a lista em
http://www.mat.puc-rio.br/~nicolau/olimp/obm-l.html
=

.







 IncrediMail - O mundo do correio eletrônico finalmente desenvolveu-se - Clique aqui

Re: [obm-l] EsSA

2003-08-14 Por tôpico Eduardo F. Botelho
Um tringulo ABC tem rea 75m^2. os pontos D,E,F e G
dividem o lado AC em 5 partes congruentes:
AD=DE=EF=FG=GC. Desse modo, a rea do triangulo BDF :
20
30
40
50
55
O tringulo considerado tem a mesma altura relativa ao vrtice B que o 
tringulo ABC com relao ao mesmo vrtice. Como podemos calcular a rea 
 por baseXaltura/2 , e a base do tringulo BDF  2AC/5 , ento a rea 
do BDF  2/5 da rea do ABC, ou seja 30.

Abrao
Eduardo
=
Instruções para entrar na lista, sair da lista e usar a lista em
http://www.mat.puc-rio.br/~nicolau/olimp/obm-l.html
=


[obm-l]

2003-08-14 Por tôpico tarciosd
OLÁ AMIGOS. URGENTEE 
TEM UM TAL CARA COM O E-MAIL DE (CLAUDIO PRATICA) QUE ESTAR MADANDO VÍRUS 
PARA A LISTA DE MATEMATICA. ESSE ÍNDIVIDUO JÁ MANDOU 2 VEZES PARA O MEU 
PC! 

_
Voce quer um iGMail protegido contra vírus e spams?
Clique aqui: http://www.igmailseguro.ig.com.br
Ofertas imperdíveis! Link: http://www.americanas.com.br/ig/

=
Instruções para entrar na lista, sair da lista e usar a lista em
http://www.mat.puc-rio.br/~nicolau/olimp/obm-l.html
=


Re: [obm-l] Trigonometria (ajuda)

2003-08-14 Por tôpico Ariel de Silvio



mas e o senx??
sen(-pi/3) = -sqrt(3)/2
o resultado de senx + sqrt(3).cosx seria ZERO

*** MENSAGEM ORIGINAL 
***As 17:47 de 9/8/2003 Nelson escreveu:

  Olá a todos, estou com uma dúvida muito fácil, mas que não consigo uma 
  explicação teórica.
  Para resolver equações trigonométricas do tipo a(senx) + b(cosx) = c, 
  onde a, b e c são números conhecidos, existem, basicamente, três métodos para 
  resolve-las.
  Por exemplo:
  
  Resolver a equação senx + raiz3(cosx) = raiz3 (obs: raiz quadrada)
  
  Utilizando o método que tenho dúvida, fica assim:
  
  Fazemos senx = y, cosx = z e resolvemos o sistema:
  y + (raiz3)z = raiz3
  y^2 + z^2 = 1
  
  (1) z = 1 = y = 0. Nesse caso, senx = 0 e cosx = 1; logo x= 
2kpi
  (2) z = 1/2 = y = raiz3/2. Nesse caso, senx = raiz3/2 e cosx = 1/2; 
  
  logo x = pi/3 + 2kpi
  
  Esse exemplo foi tirado do livro temas e metas, de Antônio dos Santos 
  Machado.O problema é que temos em cosx = 1/2, x =+ou- pi/3 + 2kpi, 
  então, qual éa explicação para descatarmos o negativo?:
  
  Desde já, agradeço.
  Nelson
  
  
  Conheça o 
  novo Cadê? - Mais rápido, mais fácil e mais preciso.Toda a web, 42 
  milhões de páginas brasileiras e nova busca por imagens! 




Re: [obm-l] ENQUETE - BELEZA MATEMATICA

2003-08-14 Por tôpico Domingos Jr.
Alguns que acho legais: --- não diria que são os 5 mais belos, mesmo pq tem
muita coisa que eu desconheço.
- Teorema de Euler (generalização do pequeno teorema de Fermat).
acredito que a demonstração poderia ser dada no ensino médio pois não é
muito difícil
- A dem. de que existem infinitos primos é sem dúvida um 'must' na sua
enquete
- Resolução de sistemas lineares através de escalonamento
acho que é o primeiro algoritmo um pouco mais avançado que se vê no
colegial!
- Teorema Binomial + Triângulo de Pascal
- O cálculo de 1² + 2² + ... + n² é sem dúvida muito interessante, existem
várias demonstrações, variando de uso de técnicas consagradas (usando
polinômios para definir a recorrência, ie p(n) - p(n-1) = n² e p(0) = 0) até
uso de blocos 1x1x1 para formar uma figura geométrica que possui na base n x
n blocos, sobre a base (n-1)x(n-1) blocos alinhados num dos quadrantes e por
aí vai... fazendo algumas manipulações com a figura vc chega na fórmula!
Isso tudo pode ser compreendido por um aluno do colegial e certamente
empolgaria muitos alunos.

Algumas outras que incluiria na lista apesar de serem mais avançadas:
http://mathworld.wolfram.com/LagrangesGroupTheorem.html
http://mathworld.wolfram.com/PrimeNumberTheorem.html
http://mathworld.wolfram.com/DirichletSeriesGeneratingFunction.html
essa última e sua relação com a teoria dos números é algo que me surpreendeu
muito... se alguém sabe onde encontro mais coisas a respeito da conexão
entre análise e teoria dos números por favor mande!

[ ]'s

=
Instruções para entrar na lista, sair da lista e usar a lista em
http://www.mat.puc-rio.br/~nicolau/olimp/obm-l.html
=


RES: RES: [obm-l] CUCA LEGAL

2003-08-14 Por tôpico Walter Gongora Junior
Tem razão, não pensei nisso :)

Desculpem-me

--
Walter Gongora Jr
[EMAIL PROTECTED] ** [EMAIL PROTECTED] #8368573

-Mensagem original-
De: [EMAIL PROTECTED]
[mailto:[EMAIL PROTECTED] Em nome de Eduardo Henrique
Leitner
Enviada em: terça-feira, 5 de agosto de 2003 00:05
Para: [EMAIL PROTECTED]
Assunto: Re: RES: [obm-l] CUCA LEGAL


está errado...

1o dia: sobe 3 metros - posição: 3metros
1a noite: desce 2 metros  - posição: 1metros
2o dia: sobe 3 metros - posição: 4metros
2a noite: desce 2 metros  - posição: 2metros
...
...
...
...
...
7a noite: desce 2 metros  - posição: 7metros
8o dia: sobre 3 metros- posição: 10metros
8a noite: desce 2 metros  - posição: 8metros
9o dia: sobre 3 metros- posição: 11metros
9a noite: desce 2 metros  - posição: 9metros
10o dia: sobre 3 metros   - posição: 12metros

portanto, 10o dia

On Mon, Aug 04, 2003 at 10:18:28PM -0300, Walter Gongora Junior wrote:
 Dois dicionários estão ordenadamente na estante da biblioteca, sendo 
 o
 primeiro
 com 900 páginas e o segundo com 600. Quantas páginas impressas estão
 entre a
 primeira página do primeiro dicionário e a primeira do segundo?
 
 Resp: 600 ou 599 páginas
 
 O problema talvez não esteja contando as capas, o que somaria mais 2 
 ao resultado final, mas creio que a resposta seja 899 paginas 
 impressas, caso as capas sejam contadas, 901.
 
 Um caramujo resolve subir um muro de 12 metros de altura da seguinte
 maneira:
 durante o dia ele sobe 3 metros e durante a noite desce 2 metros.
 Sabendo-se
 que iniciou a subida da base, ao amanhecer do primeiro dia, quantos
 dias
 gastará o caramujo para chegar ao topo?
 
 Resp: 09 dias e meio, 10 dias, no decorrer do décimo dia, etc..?
 
 Se durante o dia ele sobe 3, e desce 2 à noite, ele sobe 1 metro em um

 dia inteiro. Logo, ele levou 12 dias para subir o muro.
 
 Espero estar certo hehe
 
 Abraços!
 
 
 ==
 ===
 Instruções para entrar na lista, sair da lista e usar a lista em
 http://www.mat.puc-rio.br/~nicolau/olimp/obm-l.html


=

=
Instruções para entrar na lista, sair da lista e usar a lista em
http://www.mat.puc-rio.br/~nicolau/olimp/obm-l.html

=


=
Instruções para entrar na lista, sair da lista e usar a lista em
http://www.mat.puc-rio.br/~nicolau/olimp/obm-l.html
=


Re: [obm-l] ENQUETE - BELEZA MATEMATICA

2003-08-14 Por tôpico Johann Peter Gustav Lejeune Dirichlet
BoasVeja o livro do Gugu e do Yoshi de
Combinatoria Contemporanea.
 --- Fabio Henrique [EMAIL PROTECTED]
escreveu:  A tal beleza começou pela idéia. 
 Parabéns. 
 
 
 Em 09 Aug 2003, [EMAIL PROTECTED] escreveu: 
 
 Caros colegas da lista: 
  
 Gostaria de contar com sua participacao numa
 enquete sobre beleza 
 matematica. 
  
 O que eu precisao eh que cada um de voces me
 envie uma lista contendo algo 
 como 5 a 10 problemas/teoremas que voces
 consideram os mais bonitos e cujas 
 solucoes/demonstracoes sao as mais elegantes
 e/ou inusitadas e/ou 
 engenhosas. Nao precisa incluir a
 solucao/demonstracao, apenas o enunciado. 
 No entanto, se voce tiver em mente uma
 solucao/demonstracao especifica 
 (entre varias existentes) nao deixe de
 mencionar pelo menos o metodo 
 utilizado. 
  
 A unica restricao eh que estes resultados
 devem ser de um nivel acessivel a 
 um aluno normal de 2o. grau (ou seja, o Ultimo
 Teorema de Fermat e o 
 Porisma 
 de Poncelet estao fora, mas o caso n = 4 do
 UTF e a versao para triangulos 
 do Porisma poderiam ser incluidos). 
  
 Importante: os resultados devem ser acessiveis
 a um aluno normal de 2o. 
 grau, mas nao necessariamente fazer parte do
 curriculo normal do 2o. grau. 
  
 Tambem nao precisa responder hoje ou amanha ou
 mesmo na semana que vem. 
 Acho 
 que vale a pena pensar por um tempo e
 consultar a literatura - as vezes 
 pode 
 ter um resultado belissimo do qual voce
 simplesmente se esqueceu por nao 
 encontra-lo ha muito tempo. As Eurekas sao uma
 otima referencia. O Proofs 
 from the Book tambem, apesar de nem tudo lah
 ter nivel de 2o. grau. 
  
 Se houver um numero suficiente de respostas,
 eu me comprometo a publicar 
 uma 
 compilacao dos problemas e teoremas mais
 votados. 
  
 Desde jah a gradeco o interesse de quem quiser
 participar. 
  
 Um abraco, 
 Claudio. 
  

=
 
 Instruções para entrar na lista, sair da lista
 e usar a lista em 

http://www.mat.puc-rio.br/~nicolau/olimp/obm-l.html
 

=
 
  
 -- 
 

_
 Voce quer um iGMail protegido contra vírus e
 spams? 
 Clique aqui: http://www.igmailseguro.ig.com.br
 Ofertas imperdíveis! Link:
 http://www.americanas.com.br/ig/
 Ofertas imperdíveis!
 

=
 Instruções para entrar na lista, sair da lista
 e usar a lista em

http://www.mat.puc-rio.br/~nicolau/olimp/obm-l.html

= 

___
Conheça o novo Cadê? - Mais rápido, mais fácil e mais preciso.
Toda a web, 42 milhões de páginas brasileiras e nova busca por imagens!
http://www.cade.com.br
=
Instruções para entrar na lista, sair da lista e usar a lista em
http://www.mat.puc-rio.br/~nicolau/olimp/obm-l.html
=


[obm-l] EsSA

2003-08-14 Por tôpico elton francisco ferreira
Numa fábrica, trabalhadores reuniram-se para
presentear um amigo que iria casar. O presente
escolhido foi a quantia de 900,00, que seria dividida
igualmente entre eles. Por razões particulares, dois
daqueles trabalhadores tiraram seus nomes da lista e,
por isso, decidiu-se diminuir a quantia para 888,00,
de modo que na nova divisão coubesse a cada
participante a mesma cota de antes da saída dos dois
colegas. Com isso, coube a cada um dos participantes a
quantia de :

4,00
6,00
9,00
10,00
12,00

 Sendo x= 19 e y= 81, então a expressão (x+y)^2 + x^2
– y^2 + 2x é divisível por:

a)  2,19 e 81
b)  2,19 e 101
c)  2,81 e 100
d)  19,100 e 101
e)  81,100 e 101

achei a letra B

Um triângulo ABC tem área 75m^2. os pontos D,E,F e G
dividem o lado AC em 5 partes congruentes:
AD=DE=EF=FG=GC. Desse modo, a área do triangulo BDF é:

20
30
40
50
55

 um festival de música lotou uma praça semicircular de
200m de diâmetro. Admitindo-se uma ocupação média de 3
pessoas por m^2, qual o número mais aproximado de
pessoas presentes?
(Adote pi=3,14)

Achei: 47.100


___
Conheça o novo Cadê? - Mais rápido, mais fácil e mais preciso.
Toda a web, 42 milhões de páginas brasileiras e nova busca por imagens!
http://www.cade.com.br
=
Instruções para entrar na lista, sair da lista e usar a lista em
http://www.mat.puc-rio.br/~nicolau/olimp/obm-l.html
=


[obm-l] Algebra

2003-08-14 Por tôpico Marcos Neves
Será que alguem poderia me ajudar com
este problema de algebra?
Serei grato!!
Encontre uma série central para os
grupos D4 e S4

Marcos Neves

 
__
Acabe com aquelas janelinhas que pulam na sua tela.
AntiPop-up UOL - É grátis!
http://antipopup.uol.com.br/


=
Instruções para entrar na lista, sair da lista e usar a lista em
http://www.mat.puc-rio.br/~nicolau/olimp/obm-l.html
=


[obm-l] Re: [obm-l] ENQUETE - BELEZA MATEMTICA

2003-08-14 Por tôpico Domingos Jr.
Title: Help



E ento, quando veremos todos esses belos teoremas/problemas listados e 
demonstrados/resolvidos?
Seria muito legal ter tudo isso reunido em formato eletrnico...

[ ]'s


  - Original Message - 
  From: 
  Cludio (Prtica) 
  To: [EMAIL PROTECTED] 
  Sent: Wednesday, August 13, 2003 3:05 
  PM
  Subject: [obm-l] ENQUETE - BELEZA 
  MATEMTICA





Re: [obm-l] Problemas em Aberto - Algarismos

2003-08-14 Por tôpico e_lema
Nenhum nº qudrado perfeito termina em 3, logo o 3 deverá ser sempre o 1ºalg. 
da esq. p/ a dir.;já o seis é mais complicado. 
os nº serão da forma: 30000600...00=3*10^(f+2q+1)+6*10^(2q) 
onde onde f é o nºde zeros entre o 3 e o 6 e 2q é o nºde zeros depois do 6, 
f e q sendo inteiros não-negativos. 

Agora vamos mostrar que f só poderá ser 0(admitindo q=0): 
3*10^(f+1)+6=30*10^(f)+6=k^2 ; k inteiro positivo 
k^2=6*(5*10^(f)+1) :. 6*a=k*k ; k=a=6 ou (a=6c e 6c=q^2)  {a,c,q}C(Z*+), 
c=1 :.  5*10^(n)+1=6*c :. c=(5*10^(f)+1)/6 

6*c deverá ser múltiplo de 6, logo deverá ser múltiplo de 2 e de 3 ao mesmo 
tempo, assim a soma dos alg. de c deve ser múltiplo de 3(o que é fácil de 
observar que sempre ocorre) e c deverá ser par. 
temos duas hipóteses, 1- n0  ou  2- n=0 
1-se f0 então 6*c=50...001, isto é 6*c nunca será par. 
2-se f=0 então c=6, que é par CONCLUSÃO: 6*c=6 

Se c=1 então  a=6=k, logo 3*10^(f+1)+6=36 = f=0 

Logo o conjunto pedido será o dos números da forma: 

 36000...0; com nº par de zeros, ou: 3,6*10^(2q+1) 




Em 5 Aug 2003, [EMAIL PROTECTED] escreveu: 

Vou mais longe: 
 
Os candidatos são os quadrados da forma: 
(3*10^m + A)*10^(2n) 
onde A pertence a {1,4,6} e m e n são inteiros não negativos. 
 
Até agora, só encontrei números do tipo: 
36, 3600, 36, ..., 36*10^(2n), ... 
mas não consegui provar que são os únicos. 
 
Um abraço, 
Claudio. 
 
- Original Message - 
From: Johann Peter Gustav Lejeune Dirichlet 
 
To: 
Sent: Tuesday, August 05, 2003 1:42 PM 
Subject: Re: [obm-l] Problemas em Aberto - Algarismos 
 
 Retorno do Abertos da lista? 
 Que tal a gente achar quadrados perfeitos do tipo 
 3*10^k+6*10^l? 
 O tres nao pode vir no final.Talvez 
 modulo...Depois eu penso... 
 --- Claudio Buffara 
 escreveu:  Caros 
 colegas: 
  
  Aqui vao dois problemas que ainda estao em 
  aberto na lista. O primeiro foi 
  enviado pelo Duda Stabel. O segundo eh da 
  olimpiada iraniana, se nao me 
  engano. 
  
  1) Determinar o conjunto de números inteiros 
  positivos que satisfazem à duas 
  condições: (i) todo número possui exatamente 
  dois algarismos não-nulos, 
  sendo um deles o três(3), (ii) todo número é 
  quadrado perfeito. 
  
  2) Prove ou disprove: existe uma potencia de 2 
  tal que ao se permutar os 
  algarismos de sua representacao decimal 
  obtem-se uma outra potencia de 2. 
  
  Esse segundo tem uma solucao aparentemente 
  simples, mas esta solucao exclui 
  o caso de potencias de 2 com algarismos 0 
  internos (ou seja, numeros do 
  tipo abcdefg). 
  
  Um abraco, 
  Claudio. 
  
  
  
 
 ___ 
 Conheça o novo Cadê? - Mais rápido, mais fácil e mais preciso. 
 Toda a web, 42 milhões de páginas brasileiras e nova busca por imagens! 
 http://www.cade.com.br 
 = 
 Instruções para entrar na lista, sair da lista e usar a lista em 
 http://www.mat.puc-rio.br/~nicolau/olimp/obm-l.html 
 = 
 
= 
Instruções para entrar na lista, sair da lista e usar a lista em 
http://www.mat.puc-rio.br/~nicolau/olimp/obm-l.html 
= 
 
-- 

_
Voce quer um iGMail protegido contra vírus e spams? 
Clique aqui: http://www.igmailseguro.ig.com.br
Ofertas imperdíveis! Link: http://www.americanas.com.br/ig/
Ofertas imperdíveis!

=
Instruções para entrar na lista, sair da lista e usar a lista em
http://www.mat.puc-rio.br/~nicolau/olimp/obm-l.html
=


Re: [obm-l] geometria....

2003-08-14 Por tôpico Claudio Buffara
on 07.08.03 22:59, guilherme S. at [EMAIL PROTECTED] wrote:

 Por favor gostaria da ajuda de vcs para a soluçao
 destes dois exercicios:
 
 Mostre que se P_1,P_2,...,P_n sao vertices de um
 poligono regular
 de n lados inscrito em um circulo de raio 1, entao :
 (P_1P_2)*(P_1P_3)*(P_1P_4)*.*(P_1P_n) =n.
 
Este primeiro sai por complexos (mais especificamente, raizes n-esimas da
unidade) - veja artigo do Edmilson Motta sobre aplicacoes dos complexos em
geometria na Eureka no. 6.


 Prove que ,se uma secante a dois circulos ortogonais
 passa pelo centro
 de um deles ,os quatro pontos de intersecçao formam
 uma divisao 
 harmonica.
 
Sejam O e O' os centros, a e b as medidas dos raios e P e Q os pontos de
de interseccao das circunferencias.
Por hipotese, os triangulos OO'P e OO'Q sao retangulos (em P e Q), ja que as
circunferencias sao ortogonais.

Vamos considear primeiro o caso particular em que a secante passa pelos dois
centros. Sejam A, B, C e D os pontos de interseccao (em ordem, digamos, da
esquerda para a direita - supondo a secante horizontal).

Seja x a medida de BC.

OO' = OB + BC + BO' = (a - x) + x + (b - x) = a + b - x

Alem disso, OO' = raiz(a^2 + b^2)  (Pitagoras em OO'P)

Logo, x = a + b - raiz(a^2+b^2).

Com isso, teremos:
AB = 2a - x = raiz(a^2+b^2) + a - b
BC = x = a + b - raiz(a^2 + b^2)
CD = 2b - x = raiz(a^2+b^2) - a + b
AD = 2a + 2b - x = a + b + raiz(a^2+b^2)

Logo:
BC*AD = [a+b-raiz(a^2+b^2)]*[a+b+raiz(a^2+b^2)] = (a+b)^2 - (a^2+b^2) = 2ab

AB*CD = [raiz(a^2+b^2)+a-b]*[raiz(a^2+b^2)-a+b] = (a^2+b^2) - (a-b)^2 = 2ab

Ou seja, BC*AD = AB*CD ==
BC/AB = CD/AD == 
A, B, C, D formam uma divisao harmonica

*

Agora, tome uma secante generica passando por O e que intercepte cada
circunferencia em 2 pontos.

Sejam X, Y, Z e W os pontos de interseccao tais que O estah entre X e Y, Y
entre O e Z e Z entre Y e W.

Inicialmente teremos:
OX = OZ = OA = a

Alem disso (potencia de O em relacao a outra circunferencia):
OY*OW = OB*OD = (a - x)*(2b + a - x) = a^2 (fazendo as contas)

XY = OX + OY = a + OY
YZ = OZ - OY = a - OY
ZW = OW - OZ = OW - a
XW = OX + OW = a + OW

Assim:
YZ*XW = (a - OY)*(a + OW) = a^2 + (OW - OY)*a - OY*OW

XY*ZW = (a + OY)*(OW - a) = -a^2 + (OW - OY)*a + OY*OW

Portanto:
YZ*XW - XY*ZW = 2*(a^2 - OY*OW) = 0 ==

YZ*XW = XY*ZW == X, Y, Z, W formam uma divisao harmonica


Um abraco,
Claudio.

=
Instruções para entrar na lista, sair da lista e usar a lista em
http://www.mat.puc-rio.br/~nicolau/olimp/obm-l.html
=


[obm-l] Problemas em Aberto - Algarismos

2003-08-14 Por tôpico Claudio Buffara
Title: Problemas em Aberto - Algarismos



Caros colegas:

Aqui vao dois problemas que ainda estao em aberto na lista. O primeiro foi enviado pelo Duda Stabel. O segundo eh da olimpiada iraniana, se nao me engano.
 
1) Determinar o conjunto de números inteiros positivos que satisfazem à duas condições: (i) todo número possui exatamente dois algarismos não-nulos, sendo um deles o três(3), (ii) todo número é quadrado perfeito.
 
2) Prove ou disprove: existe uma potencia de 2 tal que ao se permutar os algarismos de sua representacao decimal obtem-se uma outra potencia de 2.

Esse segundo tem uma solucao aparentemente simples, mas esta solucao exclui o caso de potencias de 2 com algarismos 0 internos (ou seja, numeros do tipo abcdefg). 

Um abraco,
Claudio.






[obm-l]

2003-08-14 Por tôpico tarciosd
PODERIA AJUDAM POR FAVOR.. ESTOU ENROLADO NAS QUESTÕES ;; 
01) O valor de  lim-x-0 (sen²x)/sen²x é: 
a) –1   b) 0c) 1d) 2e) + infinito 

02)Considere o triângulo ABC de área S, baricentro G e medianas CM e BN. A 
área do quadrilátero AMGN é igual a: 
a) S/2  b) 2S/3 c) S/3  d) S/4  e) 3S/4 

03)) A relação entre os coeficientes b e c para que a equação x³ + bx + x = 
0 possua duas raízes iguais é: 
a) 4b³ + 27c² = 0   b) b³ + c² = 0 
c) 2b³ + 3c² = 0d) b³ + c² = 0 
e) 3b = c 

04)Considere a proposição: “Se x  5 então y = 6”. A proposição equivalente 
é: 
a) “Se x  5 então y 6” b) “Se y 6 então x  5” 
c) “Se y  5 então x = 5”   d) “Se y diferente 6 então x  = 5” 
e) “Se x =5 então y diferente 6” 

_
Voce quer um iGMail protegido contra vírus e spams? 
Clique aqui: http://www.igmailseguro.ig.com.br
Ofertas imperdíveis! Link: http://www.americanas.com.br/ig/

=
Instruções para entrar na lista, sair da lista e usar a lista em
http://www.mat.puc-rio.br/~nicolau/olimp/obm-l.html
=


[obm-l] Questes Esaex - RETA FINAL

2003-08-14 Por tôpico Joo



Pessoal, faltam 31 dias para a prova e eu ainda no 
consigo fazer vrias questes. Gostaria mesmo de ter aprendido de verdade! Mas 
no d pra saber tudo! Por favor, quem quiser ajudar, sinta-se  vontade. No 
precisa resolver as questes se no quiser,mas mandem dicas. Acho tambm 
que algumas so muito pesadas pra esta lista ( no para os Medalhistas hehehe! 
).Ento quem quiser passarpro meu e-mail diminuindo o trfego da 
lista, agradeo tambm!
Professor Morgado! Na Geometria Espacial aqui t 
brabssimo!!

FORTE ABRAO A TODOS

CLEBER


1) Seja:
Z1 = 
2ei/6 
Z2 = (1 + i/1  i)15 
ln Z3 = 1 + i(/3 + 2k)

Ento o 
valor de (Z1 + Z2)/e . Z3 :


2) QUESTO 
DESGRAADA!
Num 
determinado pas, em um ano de transio, a sua moeda foi desvalorizada frente 
ao dlar em 10% ao ms nos primeiros 6 meses e nos 6 meses seguintes em 6% ao 
ms. Neste ano a desvalorizao mdia mensal foi de:  
 
GABARITO: 7,98% am

3) O 
intervalo de convergncia da srie
ax + 1 
+ (x+1)2/2 + (x+1)3/3 + ... + (x+1)n/n + ... 
  
GABARITO = -2 x  0

4) NO 
CONSIGO FATORAR A RAIZ CBICA!
O valor do 
limite quando x  0 de
 
4(x+1) + 3(x+1) + (x+1)  
3
 
(x+1)  1

5) 
Determinar lim ln (1 + _2_ )2x  1/3 
 
n 
x+3 
GABARITO = 4

6) Expandir 
a funo f(x) = 1 / (1 + x2) em potncias de x-2, determinando o 
intervalo de convergncia da srie obtida.

7) Sejam a,b 
 0 
x1 = (a + b)/2 e 
y1 = ab,  
assim
 xn+1 = (xn + 
yn)/2 e yn = (xn + 
yn) , n = 1, 2, 3, ...
Mostre 
que (xn) e 
(yn) so seqncias montonas e convergem para o mesmo limite:

8) Mostre 
que a soma de TODAS as razes da equao Zn  1 = 0, no conjunto dos 
nmeros complexos  zero.

9) Dado um 
semi-crculo MN de raio R e um ponto P no prolongamento do dimetro, calcular a 
distncia do centro do semi-crculo ao ponto N, de modo que a tangente PQ possa 
gerar em torno do dimetro uma rea igual  gerada pelo arco MQ em torno deste 
mesmo dimetro.

10) A 
transformao do plano no plano que representa uma CONTRAO (???) de 1/2 
seguida de uma rotao horria de 45o  :

11) NO 
ENTENDI NADA!
Acumula-se 
areia num monte de forma cnica  razo de 10 dm3/min. Se a altura do 
monte  sempre igual ao dobro do raio da base, a razo na qual varia (???) a 
altura do monte quando esta (???)  de 8 dm  igual a:

12) A 
equao da curva que passa no ponto (1,1) e corta a famlia de hiprboles 
x2-y2 = a2 em ngulos retos para TODOS os 
valores de a : GABARITO 
x.y=1

13) A 
famlia de todas as curvas que tm o comprimento do segmento da normal constante 
: GABARITO 
(x-0)2 + y2 = k2

14) A rea 
do tringulo cujos vrtices so, o centro da esfera 
3x2+3y2+3z2  6x+12y+3 = 0, a origem do espao 
e o ponto (1,0,0)  ( COMO ACHO O CENTRO DA ESFERA???)

15) O 
coeficiente angular da reta perpendicular  reta tangente ao grfico de y = (x) 
+ 1 no ponto de abcissa h : 




Re: [obm-l] Problema 6 do dia 1 da IMC-rascunhos

2003-08-14 Por tôpico Johann Peter Gustav Lejeune Dirichlet
Ah,eu nao li sua soluçao ainda mas me lembrei do
artigo.Va no link 

http://www.unl.edu/amc/a-activities/a4-for-students/

Ai estara um artigo do Kiran sobre desigualdades.
Talvez depois eu acabe o problema...



 
--- Claudio Buffara
[EMAIL PROTECTED] escreveu:  on
08.08.03 17:16, Johann Peter Gustav Lejeune
 Dirichlet at
 [EMAIL PROTECTED] wrote:
 
  Oi turmaOntem imprimi a prova da
 decima
  IMC.Eu nao consegui fazer muita coisa,afinal
  ainda estou nmo ensino me dio.Mas parei pra
  pensar no problema seis no onibus enquanto
  voltava a casa.Aqui vai um resumo.
  
  Seja f(x)=soma de 1 ate n de a_n*x^n um
  polinomio em R[x].Se f(t)=0 acarreta Re(t)0
  entao a_k*a_(k+3)a_(k+2)*a_(k+3)
  Podemos supor que o polinomio e monico.
  Consegui resolver isto no caso n=3:
  Caso 1:as raizes sao reais.Logo o polinomio
 pode
  ser escrito na forma (t+t1)(t+t2)(t+t3) com
 os ts
  positivos.Assim basta abrir e conferir a
  desigualdade automaticamente!
  Caso 2:complexos nas raizes.Basta fazer
  Re(f)=Im(f)=0,escrever e usar o caso
 anterior!
  
  Pro caso geral tive uma ideia bem
 legal.Tentem
  ver se este artigo ainda esta online,senao...
  
  Parece Parece que na pagina da AMS tem algo
 do
  Kiran Kedlaya sobre desigualdades.Tem um
 truque
  util que deve dar para o gasto nesse
 problema...
  Favor procurar em www.ams.org algo
 parecido...
  
 Oi, Dirichlet:
 
 O enunciado na verdade diz que:
 a(k)*a(k+3)  a(k+1)*a(k+2) para k = 0,1,..,
 n-3
  
 *
 
 Esse deu um certo trabalho...
 
 Na minha opiniao, a observacao mais importante
 para esse problema eh a
 seguinte:
 Se todas as raizes de um dado polinomio real
 p(x) tem parte real negativa,
 entao os fatores irredutiveis de p(x) serao da
 forma:
 (x + a), onde a eh um real positivo
 ou
 (x^2 + bx + c), onde b e c sao reais positivos.
 Assim, se p(x) = a(0) + a(1)*x + ... +
 a(n)*x^n, entao todos os a(i) terao o
 mesmo sinal (em particular, se p(x) for monico,
 entao todos os coeficientes
 serao positivos)
 
 Isso quer dizer que, para 0 = i = j = n,
 teremos a(i)*a(j)  0.
 
 *
   
 Os casos onde grau(p(x)) = 1, 2 e 3 podem ser
 verificados por inspecao, como
 voce disse.
 
 Hipotese de inducao:
 Suponha que o resultado seja verdadeiro para
 polinomios de grau = n.
 
 Seja p(x) = b(0) + b(1)*x + ... +
 b(n+1)*x^(n+1) um polinomio de grau n+1
 cujas raizes tem parte real negativa.
 
 Precisamos considerar apenas dois casos:
 
 CASO 1: Todas as raizes de p(x) sao reais (e
 negativas).
 Nesse caso, podemos escolher duas dessas raizes
 e chamar a sua soma -t e seu
 produto de u (u, t: reais positivos).
 
 CASO 2: p(x) possui (pelo menos) um par de
 raizes complexas conjugadas.
 Nesse caso, podemos chamar as suas partes reais
 (que sao iguais) de -t/2 e
 seus modulos (tambem iguais) de raiz(u) (u, t:
 reais positivos).
 
 Em ambos os casos podemos escrever:
 p(x) = (x^2 + tx + u)*q(x), onde q(x) eh um
 polinomio de grau n-1 cujas
 raizes tem parte real negativa.
 
 *
 
 q(x) = a(0) + a(1)*x + ... + a(n-1)*x^(n-1) ==
 
 Para 0 = k = n+1, a relacao entre os
 coeficientes de p(x) e q(x) eh:
 
 b(k) = u*a(k) + t*a(k-1) + a(k-2)
 
 onde convencionamos que:
 a(-2) = a(-1) = a(n) = a(n+1) = 0.
 
 *
 
 Seja k um inteiro tal que 0 = k = n-2.
 
 b(k+1)*b(k+2) - b(k)*b(k+3) =
 
 [u*a(k+1) + t*a(k) + a(k-1)]*[u*a(k+2) +
 t*a(k+1) + a(k)] -
 [u*a(k) + t*a(k-1) + a(k-2)]*[u*a(k+3) +
 t*a(k+2) + a(k+1)] =
 
 [a(k+1)a(k+2) - a(k)a(k+3)]*u^2
 + [a(k+1)^2 - a(k-1)a(k+3)]*u*t
 + [a(k)a(k+1) - a(k-1)a(k+2)]*t^2
 + [a(k-1)a(k+2) - a(k-2)a(k+3)]*u
 + [a(k)^2 - a(k-2)a(k+2)]*t
 + [a(k-1)a(k) - a(k-2)a(k+1)] =
 
 A*u^2 + B*u*t + C*t^2 + D*u + E*t + F, onde,
 pela hipotese de inducao
 aplicada a q(x), os coeficientes A, C, D e F
 sao positivos.

 a(k-1)a(k)  a(k-2)a(k+1)  0  (vide observacao
 acima)
 a(k)a(k+1)  a(k-1)a(k+2)  0 ==
 
 Multiplicando ambas as desigualdades, obtemos:
 a(k-1)a(k)^2a(k+1)  a(k-2)a(k-1)a(k+1)a(k+2)
 
 Dividindo ambos os membros por a(k-1)a(k+1)
 (que eh uma quantidade
 positiva), obtemos:
 a(k)^2  a(k-2)a(k+2) ==
 
 E = a(k)^2 - a(k-2)a(k+2)  0
 
 Analogamente, podemos provar que B  0.
 
 Ou seja, t, u, A, B, C, D, E, F sao todos
 positivos ==
 
 b(k+1)b(k+2) - b(k)b(k+3)  0 e acabou!!!
 
 Um abraco,
 Claudio.
 
 

=
 Instruções para entrar na lista, sair da lista
 e usar a lista em

http://www.mat.puc-rio.br/~nicolau/olimp/obm-l.html

= 

___
Conheça o novo Cadê? - Mais rápido, mais fácil e mais preciso.
Toda a web, 42 milhões de páginas brasileiras e nova busca por imagens!
http://www.cade.com.br
=
Instruções para entrar na lista, sair da lista e usar a lista em
http://www.mat.puc-rio.br/~nicolau/olimp/obm-l.html

Re: [obm-l] Teorema_das_raízes_racionais

2003-08-14 Por tôpico Johann Peter Gustav Lejeune Dirichlet
Isto nao e tao dificil mas e trabalhoso...A ideia
e substituir x=p/q e abrir tudo com muita
vontade.Va ao artigo do Shine da ultima semana
olimpica sobre polinomios.
Dificil mesmo e generalizar...

 --- Henrique_Patrício_Sant'Anna_Branco
[EMAIL PROTECTED] escreveu:  Caros,
 Recentemente foi usado na lista o teorema das
 raízes racionais, que segue:
 Se um polinômio f(x) = a_n*x^n +
 a_(n-1)*x^(n-1) + ... + a_0 tiver raízes
 racionais, estas serão da forma p/q com p
 divisor de a_0 e q divisor de a_n.
 Todo mundo aprende isso no ensino médio, mas é
 raro ver a demonstração.
 Pesquisando na Internet, nao achei nada
 também...
 Alguém saberia me dar uma demonstração desse
 teorema?
 
 Grato,
 Henrique.
 

=
 Instruções para entrar na lista, sair da lista
 e usar a lista em

http://www.mat.puc-rio.br/~nicolau/olimp/obm-l.html

= 

___
Conheça o novo Cadê? - Mais rápido, mais fácil e mais preciso.
Toda a web, 42 milhões de páginas brasileiras e nova busca por imagens!
http://www.cade.com.br
=
Instruções para entrar na lista, sair da lista e usar a lista em
http://www.mat.puc-rio.br/~nicolau/olimp/obm-l.html
=


Re: [obm-l] Problema das 3 portas

2003-08-14 Por tôpico Bernardo Vieira Emerick
Claudio,
Eu insisto que tanto faz trocar de porta. Pensemos no problema em duas 
etapas. Na primeira você escolhe entre três portas. Atrás de uma está o 
prêmio. A probabilidade de você ganhar será de 1/3, certo? Na segunda, você 
tem que escolher entre duas portas. O prêmio está em uma delas. A sua 
probabilidade de ganhar será, portanto, 1/2 para as duas portas. Pouco 
importa o que você escolheu na primeira etapa. É como se fosse outro jogo, 
só que se tenha eliminada uma das opções erradas.


From: Claudio Buffara [EMAIL PROTECTED]
Reply-To: [EMAIL PROTECTED]
To: [EMAIL PROTECTED]
Subject: [obm-l] Problema das 3 portas
Date: Tue, 12 Aug 2003 00:43:58 -0300
Oi, Henrique:

Eu insisto que a estrategia otima eh trocar de porta.

Veja o meu raciocinio:

Chame as 3 portas de A, B e C.
Suponha s.p.d.g. que inicialmente voce escolhe a porta A.
Temos 3 casos a considerar:
1) O premio estah atras de A:
Nesse caso, o apresentador abre B ou abre C (qualquer uma das duas estarah
vazia)
Se voce trocar, voce estarah saindo da porta vencedora e indo para uma das
perdedoras (a que ele nao abriu) - voce perde se trocar.
2) O premio estah atras de B:
Nesse caso, o apresentador abre a porta C.
Se voce trocar, voce estarah saindo de A e indo para B - a porta vencedora.
Ou seja, voce ganha se trocar.
3) O premio estah atras de C:
Nesse caso, o apresentador abre a porta B.
Se voce trocar, voce estarah saindo de A e indo para C - a porta vencedora.
Ou seja, voce ganha se trocar.
Assim, ao decidir trocar voce perde em um caso e ganha em 2. Supondo que a
probabilidade do premio estar atras de uma dada porta eh 1/3, a sua
probabilidade de ganhar ao trocar eh igual a 2/3  1/2. Logo, voce deve
trocar de porta.
Com 1 milhao de portas, a decisao eh ainda mais obvia, pois se voce nao
trocar, o que voce estarah dizendo eh que voce escolheu a porta certa de
primeira, um evento que pra voce tem uma probabilidade de 1 em 10^6.
Suponha que voce tenha escolhido inicialmente a porta no. 1, a qual tem, 
pra
voce, probabilidade de 1/10^6 de conter o premio.
Isso quer dizer que, pra voce, a probabilidade do premio estar atras de uma
das outras 999.999 portas eh de 999.999/10^6.

Quando o apresentador abre 999.998 portas dentre as 999.999 que voce nao
escolheu, ele estah colapsando a probabilidade de cada porta aberta para 0,
e concentrando a probabilidade total de 999.999/10^6 numa unica porta, que
permanece fechada (estas probabilidades sao sempre do seu ponto de vista. 
Do
ponto de vista do apresentador, que sabe qual a porta premiada, as
probabilidades sao: 1 do premio estar atras da porta premiada e 0 de estar
atras de qualquer outra).

Nesse caso, voce seria louco de nao trocar de porta.

Um abraco,
Claudio.
on 11.08.03 23:27, Henrique Patrício Sant'Anna Branco at
[EMAIL PROTECTED] wrote:
 Por mais que eu ache pedante e ridiculo alguem se vangloriar de ter o 
QI
 mais alto do mundo, nesse caso acho que a Marilyn estah certa. Voce 
deve
 trocar de porta.

 Desculpem a minha ignorancia, mas o que ha de errado com o argumento de 
1
 milhao de portas? Me parece que, nesse caso, a probabilidade de voce 
ter
 escolhido a porta certa de primeira eh apenas de 1/1.000.000. Logo, a
 probabilidade da outra porta ter o premio eh de 999.999/1.000.000. Ou 
nao?

 Cláudio,

 No problema original, temos três portas, escolhemos uma e o apresentador
 logo em seguida abre outra que, com certeza, não tem o prêmio. 
Inicialmente,
 havia uma chance de 1/3 de uma determinada porta conter o prêmio. Ao ser
 aberta uma das portas e mostrar que ela não contém o prêmio, sobram 
apenas
 duas portas: a que você escolheu e uma outra. É como se a probabilidade
 tivesse sido atualizada pelo fato do apresentador mostrar uma porta 
que
 não contém o prêmio (isso é o Teorema de Bayes se não me engano). Agora 
que
 sobraram apenas duas portas, cada uma delas tem uma em duas chances 
(1/2) de
 ter o prêmio e, portanto, não há justificativa (matematica) para trocar 
de
 porta ou não. O fato do apresentador abrir uma das portas muda a
 probabilidade das DUAS portas e não apenas para uma, como a Sra. Marilyn
 quer nos fazer crer.

 Quanto ao argumento de 1 milhão de portas... Como você disse, a
 probabilidade de você ter escolhido a porta certa de primeira é de 
1/10^6
 que é a mesma probabilidade de cada uma das outras portas 
individualmente.
 Lembre-se que todas as probabilidades devem somar 1 = 10^6/10^6. O caso 
que
 você apontou (999.999/10^6) é a probabilidade combinada de todas as 
outras
 portas (cada uma entre as 10^6 portas têm probabilidade de 1/10^6) que 
você
 não escolheu de terem o prêmio e não de uma única porta das que você não
 escolheu. Se você simplesmente muda de porta, a probabilidade continua 
sendo
 a mesma... E, se ele abrir 777.777 portas sem o prêmio, a probabilidade 
de
 TODAS as portas fica em 1/222.223 e, novamente, não faz diferença mudar 
a
 porta...

 Espero ter sido claro.
 Abraço,
 Henrique.

 

[obm-l] Ajuda

2003-08-14 Por tôpico dex
Olá pessoal 

Gostaria de saber uma boa demonstração para o exercício abaixo 

P = sen(pi/n) . sen(2pi/n) . sen(3pi/n) . ... . sen[(n-1)pi/n]
com n Inteiro positivo 

A resposta é P = n/[2^(n-1)], mas cheguei até este resultado de uma maneira 
muito pouco prática, nada natural para uma questão de matemática (de 
vestibular). Consegui prová-la utilizando o resultado de uma outra questão, 
que versava sobre polinômios e complexos. Ou seja, se eu não tivesse visto 
esta outra questão não conseguiria provar nada! 

Atneciosamente
¡Thyago! 



--
POP. Nem parece internet grátis. 

Seja POP você também!
Acesse: http://www.pop.com.br/pop_discador.php e baixe o POPdiscador.
=
Instruções para entrar na lista, sair da lista e usar a lista em
http://www.mat.puc-rio.br/~nicolau/olimp/obm-l.html
=


Re: [obm-l] Questões da ESAEx Por favor!

2003-08-14 Por tôpico A. C. Morgado




Se n  1, o limite eh evidentemente infinito.
Se 0= n  1 o limite evidentemente vale 0.
Resta o caso n=1. Nesse caso temos [1 + 2/(x-1)]^x que tende a e^2.
Nao existe n para o qual o limite seja 9.

Joo wrote:

  
  
  
  
  Estou me matando e no consigo!
  
  
  8) Se lim [(nx + 1)/(x-1)]^x = 9,
qual o valor de n ?
   x- + oo
  
  
  QUEM POR FAVOR PUDER AJUDAR, ESTAS
QUESTES SO DE CONCURSOS PASSADOS DA ESAEX!
  O CONCURSO  EM SET E AINDA ESTOU
MUITO FRACO!
  
  forte abrao
  
  CLEBER
  





[obm-l] Problema dos algarismos do Duda Stabel

2003-08-14 Por tôpico Claudio Buffara

Problema original:

Achar todos os quadrados perfeitos que tenham apenas 2 algarismos
significativos sendo um deles o 3.

*

O Eduardo ([EMAIL PROTECTED]) apresentou uma demonstracao de que os unicos
numeros que satisfazem o enunciado sao os da forma 36*10^(2m), a qual eu
ainda nao examinei.

Entrementes, com algum esforco consegui encontrar demonstracoes (longas e
nao muito elegantes mas espero que corretas) de que nenhum numero da forma
3*10^(m+1) + 1  ou da forma  3*10^(m+1) + 4  eh quadrado perfeito.

Como estes eram os unicos numeros que ainda precisavam ser examinados (vide
minha mensagem anterior sobre o assunto), concluimos que os unicos numeros
que satisfazem o enunciado sao, de fato, os da forma 36*10^(2m) (m inteiro
nao negativo)

*

Suponhamos que 3*10^(m+1) + 1 = N^2, para algum natural N.

Como 34 e 304 nao sao quadrados, podemos supor que m = 2.

N^2 == 1 (mod 10) ==
N == 1  ou  N == -1 (mod 10) ==
N = 10a +ou- 1, para algum natural a ==
N^2 = 100a^2 +ou- 20a + 1 = 20a(5a +ou- 1) + 1 ==
3*10^(m+1) = 20a(5a +ou- 1) ==
3*2^(m-1)*5^m = a(5a +ou- 1)

5^m | a == 
a = b*5^m, para algum natural b ==
3*2^(m-1) = b*(b*5^(m+1) +ou- 1)

Caso 1: b eh par
b*5^(m-1) +ou- 1 eh impar ==
2^(m-1) | b == 
b = c*2^(m-1) para algum natural c ==
3 = c*(c*2^m*5^(m-1) +ou- 1)

Mas:
m = 2 == 
c*2^m*5^(m-1) +ou- 1 = 20*c +ou- 1  3 ==
contradicao

Caso 2: b eh impar
2^(m-1) | b*5^(m+1) +ou- 1 ==
b*5^(m+1) +ou- 1 = c*2^(m-1) para algum natural c ==
3 = b*c ==
b = 1 e c = 3   ou   b = 3 e c = 1

b = 1 e c = 3 == 
5^(m+1) +ou- 1 = 3*2^(m-1) ==
contradicao, pois para m = 2, 5^(m+1) - 1  3*2^(m-1) (inducao facil)

b = 3 e c = 1 ==
3*5^(m+1) +ou- 1 = 2^(m-1) ==
contradicao (idem)

Conclusao: nenhum numero da forma 3*10^(m+1) + 1 eh quadrado perfeito.

*

Suponhamos que 3*10^(m+1) + 4 = N^2, para algum natural N.

Como 34, 304 e 3004 nao sao quadrados, podemos supor que m = 3.

N^2 == 4 (mod 10) ==
N == 2 ou N == -2 (mod 10) ==
N = 10a +ou- 2, para algum natural a ==
N^2 = 100a^2 +ou- 40a + 4 = 20a(5a +ou- 2) + 4 ==
3*10^(m+1) = 20a(5a +ou- 2) ==
3*2^(m-1)*5^m = a(5a +ou- 2)

5^m | a == 
a = b*5^m para algum natural b ==
3*2^(m-1) = b(b*5^(m+1) +ou- 2)

Caso 1: b eh par
b = 2c, para algum natural c ==
3*2^(m-1) = 2c(2c*5^(m+1) +ou- 2) ==
3*2^(m-3) = c(c*5^(m+1) +ou- 1)

Caso 1.1: c eh par
c*5^(m-1) +ou- 1 eh impar ==
2^(m-3) | c ==
c = d*2^(m-3) para algum natural d ==
3 = d*(d*2^(m-3)*5^(m-1) +ou- 1) ==
contradicao, pois d*2^(m-3)*5^(m-1) +ou-1 = 24

Caso 1.2: c eh impar ==
c*5^(m-1) +ou- 1 eh par ==
c*5^(m-1) +ou- 1 = 2^(m-3)*d, para algum natural d ==
3 = c*d ==
c = 1, d = 3ouc = 3, d = 1
Ambos os casos resultam em contradicao, conforme visto acima

Caso 2: b eh impar
b(b*5^(m-1) +ou- 2) eh impar ==
3*2^(m-1) eh impar ==
m = 1 ==
contradicao, pois estamos supondo m = 3.

Conclusao: nenhum numero da forma 3*10^(m+1) + 4 eh quadrado perfeito

*

Um abraco,
Claudio.

=
Instruções para entrar na lista, sair da lista e usar a lista em
http://www.mat.puc-rio.br/~nicolau/olimp/obm-l.html
=


[obm-l] Re: [obm-l] Re:[obm-l] Re: [obm-l] Questões Esaex - RETA FINAL 4 e 5

2003-08-14 Por tôpico J Augusto Tavares
Obrigado... eu nao formatei o texto ... os simbolos q tao com problemas sao
os da msg original!!
eu copiei o simbolo de infinito pra colocar na minha msg, ele deu errado
vou colocar so as respostas da 4 e 5 cujo titulo eh
[obm-l] Questões Esaex - RETA FINAL  do Joao!

- Original Message -
From: amurpe [EMAIL PROTECTED]
To: [EMAIL PROTECTED]
Cc: [EMAIL PROTECTED]
Sent: Thursday, August 14, 2003 4:43 AM
Subject: [obm-l] Re:[obm-l] Re: [obm-l] Questões Esaex - RETA FINAL 4 e 5


 
 Os simbolos estão com problema, por favor reenvie a msg.

 Um abraço.Amurpe
4) NÃfO CONSIGO FATORAR A RAIZ CÃsBICA!

Resposta:
 
Fazendo  (x+1) = y^12 , como x-0  ,   y-1.
 
 (y^3 + y^4 + y^6 - 3)/(y^6 - 1) ,  [(y^3 -
  1) + (y^4 -1) + (y^6 - 1)]/[(y^3 + 1)(y^3 -1)]
 
 eleminando o fator (y- 1), nao existira mais  a indeterminacao !
 
 
 
 
 
 
 
5)

Resposta:
 
n-+infinito
 
ln[lim (1 + 2/(x+3))^(2x - 1/3)], lim (1 + 2/(x+3))^
 (x+3)*[(2x -1/3)/(x+3)  Elevando a (x+3)/(x+3)
 
temos o limite fundamental { n-+infinito } d  (1 + k/x)^x = e^k
 
(e^2)^lim(2x -1/3)/(x+3) = (e^2)^2 , logo ln e^4  eh 4.
 
 
 
 
 
 
 
 


 __
 Acabe com aquelas janelinhas que pulam na sua tela.
 AntiPop-up UOL - É grátis!
 http://antipopup.uol.com.br/


 =
 Instruções para entrar na lista, sair da lista e usar a lista em
 http://www.mat.puc-rio.br/~nicolau/olimp/obm-l.html
 =


=
Instruções para entrar na lista, sair da lista e usar a lista em
http://www.mat.puc-rio.br/~nicolau/olimp/obm-l.html
=


[obm-l] Teorema das raízes racionais

2003-08-14 Por tôpico Henrique Patrício Sant'Anna Branco
Pessoal,

Há um tempo atrás eu e o Fael lançamos uma pergunta sobre a demonstração
desse teorema (respondida pelo Morgado).

Procurando nos meus favoritos, achei um link interessante sobre Teoria da
Factorização (parece ser lusitano), que tem a demonstração na página 17.

Se interessar:
http://hermite.cii.fc.ul.pt/~pedro/fact.pdf

Abraços,
Henrique.

=
Instruções para entrar na lista, sair da lista e usar a lista em
http://www.mat.puc-rio.br/~nicolau/olimp/obm-l.html
=


Re: [obm-l] ENQUETE - BELEZA MATEMATICA

2003-08-14 Por tôpico Johann Peter Gustav Lejeune Dirichlet
Colegas,nao acreditem em testes de QI

 --- Bernardo Vieira Emerick
[EMAIL PROTECTED] escreveu:  Que
piada!!! Marylin vos Savant, tida como a
 pessoa com o maior QI do mundo 
 (concordo com o Domingos Jr.: bulsshit!)
 confundiu tudo. O problema era 
 assim: num jogo, a pessoa escolha uma entre
 três portas. O apresentador, 
 então, abra uma das portas. Como ele sabe qual
 é a porta que contém o 
 prêmio, ele abre uma que não o contém - já que
 o jogo dar-se-ia por 
 encerrado. A pergunta é: o jogador deveria
 trocar de porta?
 Segundo Marylin, sim!, porque a probabilidade
 da opção que ele teria 
 continuaria 1/3, enquanto a outra aumentaria
 para 2/3!!! Qual a razão disso? 
 A probabilidade da porta que ele escolheu não
 poderia subir subitamente para 
 1/2, como sugerem os matemáticos. Ora, como
 então a outra porta pode??? Isso 
 ela não explica.
 Ela aparentemente desconhece o conceito
 primeiro de probabilidade, que é a 
 chance de se acertar, e por isso está atrelada
 ao número de possibilidades 
 possíveis e o número de possibilidades
 requeridas para se acertar o 
 resultado. Então, a probabilidade será dada -
 como é de conhecimento geral, 
 exceto possivelmente de Marylin - pela divisão
 do número de possibilidades 
 requeridas pelo número total de
 possibilidades. Parece-me que ela acredita 
 que a única forma de se aumentar a
 probabilidade é aumentando o número de 
 possibilidades requeridas. Isso justificaria
 o we've learned nothing to 
 allow us to revise the chances on the shell
 under your finger que ela diz. 
 O que mudou, e que ela incrivelmente não
 percebeu, é o número total de 
 possibilidades. Simplificando para ela, o
 numerozinho de baixo diminuiu, 
 então o número do outro lado do sinal de
 igualdade aumentou, já que o 
 numerozinho de cima da fração permaneceu
 constante. Será que assim ela 
 entenderia???
 
 
 
 From: Domingos Jr. [EMAIL PROTECTED]
 Reply-To: [EMAIL PROTECTED]
 To: [EMAIL PROTECTED]
 Subject: Re: [obm-l] ENQUETE - BELEZA
 MATEMATICA
 Date: Mon, 11 Aug 2003 19:03:11 -0300
 
 O Noga Alon conta que fizeram esta pergunta
 para ele uma vez que ele
 começou explicando a prova de Euclides de que
 há infinitos primos
 em um programa de televisão, eu acho:
 
 And today, are there still infinitely many
 primes?
 
 E sem sair do clima, deem uma olhada em

http://qsilver.queensu.ca/~phil158d/intro/montyh3.htm
 
 Eu deveria ter visto isso antes de escrever o
 meu artigo da Eureka!
 
 --- x ---
 Putz, essa mulher do QI mais alto do mundo
 (bullshit!) não concorda com o
 Princípio da Indução Finita também! hehehe, o
 pior é que é sério!!!
 

=
 Instruções para entrar na lista, sair da lista
 e usar a lista em

http://www.mat.puc-rio.br/~nicolau/olimp/obm-l.html

=
 

_
 MSN Hotmail, o maior webmail do Brasil. 
 http://www.hotmail.com
 

=
 Instruções para entrar na lista, sair da lista
 e usar a lista em

http://www.mat.puc-rio.br/~nicolau/olimp/obm-l.html

= 

___
Conheça o novo Cadê? - Mais rápido, mais fácil e mais preciso.
Toda a web, 42 milhões de páginas brasileiras e nova busca por imagens!
http://www.cade.com.br
=
Instruções para entrar na lista, sair da lista e usar a lista em
http://www.mat.puc-rio.br/~nicolau/olimp/obm-l.html
=


[obm-l] Re: [obm-l] Livro da OBM

2003-08-14 Por tôpico ciceroth
Esse livro da OBM é igual ao livro escrito somente pelo Luiz Amâncio que
foi publicado pela Editora da UFC a alguns anos?
Cícero Thiago



--
Use o melhor sistema de busca da Internet
Radar UOL - http://www.radaruol.com.br



=
Instruções para entrar na lista, sair da lista e usar a lista em
http://www.mat.puc-rio.br/~nicolau/olimp/obm-l.html
=


Re: [obm-l] Cochilo na aula de algebra

2003-08-14 Por tôpico Henrique Patrício Sant'Anna Branco
Você não pode usar isso assim, pois nada assegura que todos os divisores de
1024 são raízes da equação.
De fato, o teorema nos diz que, SE um polinômio f(x) = a_n*x^n +
a_(n-1)*x^(n-1) + ... + a_0 tiver raízes racionais, estas serão da forma p/q
com p divisor de a_0 e q divisor de a_n. No problema, só temos o dado que as
raízes são todas reais e positivas e, logo, pode haver raizes irracionais.

Abraço,
Henrique.

- Original Message - 
From: [EMAIL PROTECTED]
To: [EMAIL PROTECTED]
Sent: Saturday, August 09, 2003 10:18 PM
Subject: Re: [obm-l] Cochilo na aula de algebra


Pelo teorema das raizes racionais temos as raizes p/q da eq. de modo que
a[n]
eh divisor de q e a[0] eh divisor de p.
No problema abaixo temos a[n] = 1 e a[0] = 1024. Com a[n] =1 e as raizes sao
reais e positivas, temos que as raizes da equacao abaixo eh o conjunto dos
divisores positivos de 1024 (=2^10) que sao obtidas tomando-se o produto
2^10
dois a dois  {2,4,8,16,32,64,128,256,512,1024}
Se cometi algum deslize, por favor Claudio (ou qualquer outro participante)
me corrija.

=
Instruções para entrar na lista, sair da lista e usar a lista em
http://www.mat.puc-rio.br/~nicolau/olimp/obm-l.html
=


[obm-l] ENQUETE - BELEZA MATEMATICA

2003-08-14 Por tôpico Claudio Buffara
Caros colegas:

Estou extremamente decepcionado com as listas de problemas supostamente
bonitos que foram enviadas pra lista ateh o presente momento. Imaginem soh -
teorema do valor intermediario, secoes conicas, poliedros regulares,
conjuntos enumeraveis. Onde voces estao com a cabeca? Isso tudo eh
matematica do tempo da carochinha. Infinitude dos primos? Isso eh tao velho
que ja devia ter sido revogado!

Pessoal, vamos olhar pra frente, prestigiar a modernidade e, mais
importante, os talentos matematicos nacionais. Com isso em mente, elaborei
uma lista que deixaria o David Hilbert morrendo de inveja. E o que eh
melhor, com problemas propostos pelas mentes mais privilegiadas da nossa
lista - aqueles que certamente vao cobrir o Brasil de gloria com medalhas
Fields, premios Abel e ainda fazer os suecos criarem o premio Nobel de
matematica. Sendo assim, aqui vao os problemas que pautarao o
desenvolvimento da matematica nos seculos vindouros.

As 7 primeiras perolas (perolas? brilhantes, isso sim!) sao de autoria do
nobre sabio Jorge Luiz de Fortaleza. As 5 restantes - puro genio - do nosso
querido e muitas vezes incompreendido mestre Dirichlet.

Problemas e teoremas do Jorge Luiz:

1) A pegadinha no Pereira.

2) O caramujo.

3) A margem de erro real.

4) A compra, venda, recompra e revenda.

5) Apostar ou jogar uma moeda.

6) As páginas dos dicionários.

7) O método cuca legal de multiplicação.


Problemas e teoremas do Dirichlet:

8) O da sequencia que sai facil por Kronecker - eh soh pegar os numeros das
pontas e das extremidades e ir multiplicando de fora pra dentro;

9) O do triangulo que voce expressa o seno em funcao da tangente da metade
do angulo externo do triangulo da pagina ao lado e cai num polinomio que os
fatores se cancelam e se nao se cancelarem eh que voce errou alguma conta
mas tudo bem que tambem tem a solucao cearense.

10) Aquele que voce aplica Schur 3 vezes e depois divide o intervalo em 4
sub-intervalos e aplica Schur mais umas 3 ou 4 vezes e ai fica trivial -
basta usar Fermat, ou Cauchy, ou Euler, ou um cara desses ai.

11) Tem o outro que sai pelo teorema de Turan-Erdos-Von Neumann e mais
outros dois ou tres hungaros que eu nao lembro o nome e ai com 4n^2+3  2n+1
 7n^3+9  111  112  2m eh soh achar os dois grafos sem triangulos no meio do
emaranhado de pontinhos.

12) Finalmente, tem aquele da soma dos senos que eh soh ver a Eureka e - ah!
meu! Esse ai eh muito trivial - depois eu mando pra lista.


Um abraco,
Claudio.

=
Instruções para entrar na lista, sair da lista e usar a lista em
http://www.mat.puc-rio.br/~nicolau/olimp/obm-l.html
=


Re: [obm-l] EsSA

2003-08-14 Por tôpico Henrique Patrício Sant'Anna Branco
  Sendo x= 19 e y= 81, então a expressão (x+y)^2 + x^2
 - y^2 + 2x é divisível por:

Desenvolvendo, temos:
x^2 + 2xy + y^2 + x^2 - y^2 + 2x = 2x^2+2xy+2x
Essa expressão é (claramente) divisível por x (logo, por 19).
Dividindo chegamos a 2x + 2y + 2, divisível por 2. Assim, (2x + 2y + 2)/2 =
x + y + 1. Divisível por ela mesma, ou seja, 101.
Logo, letra b) 2, 19, 101.

Abraços,
Henrique.

=
Instruções para entrar na lista, sair da lista e usar a lista em
http://www.mat.puc-rio.br/~nicolau/olimp/obm-l.html
=


Re: [obm-l] Ajuda - pequena obs.

2003-08-14 Por tôpico J Augusto Tavares
Sendo n inteiro positivo,
o caso n=1 teria que dar 1 segundo o resultado... so que P sera nulo! hehe

- Original Message -
From: dex [EMAIL PROTECTED]
To: [EMAIL PROTECTED]
Sent: Monday, August 11, 2003 7:05 AM
Subject: [obm-l] Ajuda


 Olá pessoal

 Gostaria de saber uma boa demonstração para o exercício abaixo

 P = sen(pi/n) . sen(2pi/n) . sen(3pi/n) . ... . sen[(n-1)pi/n]
 com n Inteiro positivo

 A resposta é P = n/[2^(n-1)], mas cheguei até este resultado de uma
maneira
 muito pouco prática, nada natural para uma questão de matemática (de
 vestibular). Consegui prová-la utilizando o resultado de uma outra
questão,
 que versava sobre polinômios e complexos. Ou seja, se eu não tivesse visto
 esta outra questão não conseguiria provar nada!

 Atneciosamente
 ¡Thyago!



 --
 POP. Nem parece internet grátis.

 Seja POP você também!
 Acesse: http://www.pop.com.br/pop_discador.php e baixe o POPdiscador.
 =
 Instruções para entrar na lista, sair da lista e usar a lista em
 http://www.mat.puc-rio.br/~nicolau/olimp/obm-l.html
 =


=
Instruções para entrar na lista, sair da lista e usar a lista em
http://www.mat.puc-rio.br/~nicolau/olimp/obm-l.html
=


[obm-l] Resultado IX Olimpiada de Maio.

2003-08-14 Por tôpico Olimpiada Brasileira de Matematica
Caros(as) Professores(as)

Ja' esta' publicado no nosso site o resultado
oficial da IX Olimpiada de Maio.
http://www.obm.org.br/maio.htm

Abracos, Nelly.

=
Instruções para entrar na lista, sair da lista e usar a lista em
http://www.mat.puc-rio.br/~nicolau/olimp/obm-l.html
=


[obm-l] Problema das 3 portas

2003-08-14 Por tôpico Claudio Buffara
Oi, Henrique:

Eu insisto que a estrategia otima eh trocar de porta.

Veja o meu raciocinio:

Chame as 3 portas de A, B e C.
Suponha s.p.d.g. que inicialmente voce escolhe a porta A.

Temos 3 casos a considerar:
1) O premio estah atras de A:
Nesse caso, o apresentador abre B ou abre C (qualquer uma das duas estarah
vazia)
Se voce trocar, voce estarah saindo da porta vencedora e indo para uma das
perdedoras (a que ele nao abriu) - voce perde se trocar.

2) O premio estah atras de B:
Nesse caso, o apresentador abre a porta C.
Se voce trocar, voce estarah saindo de A e indo para B - a porta vencedora.
Ou seja, voce ganha se trocar.

3) O premio estah atras de C:
Nesse caso, o apresentador abre a porta B.
Se voce trocar, voce estarah saindo de A e indo para C - a porta vencedora.
Ou seja, voce ganha se trocar.

Assim, ao decidir trocar voce perde em um caso e ganha em 2. Supondo que a
probabilidade do premio estar atras de uma dada porta eh 1/3, a sua
probabilidade de ganhar ao trocar eh igual a 2/3  1/2. Logo, voce deve
trocar de porta.

Com 1 milhao de portas, a decisao eh ainda mais obvia, pois se voce nao
trocar, o que voce estarah dizendo eh que voce escolheu a porta certa de
primeira, um evento que pra voce tem uma probabilidade de 1 em 10^6.

Suponha que voce tenha escolhido inicialmente a porta no. 1, a qual tem, pra
voce, probabilidade de 1/10^6 de conter o premio.
Isso quer dizer que, pra voce, a probabilidade do premio estar atras de uma
das outras 999.999 portas eh de 999.999/10^6.

Quando o apresentador abre 999.998 portas dentre as 999.999 que voce nao
escolheu, ele estah colapsando a probabilidade de cada porta aberta para 0,
e concentrando a probabilidade total de 999.999/10^6 numa unica porta, que
permanece fechada (estas probabilidades sao sempre do seu ponto de vista. Do
ponto de vista do apresentador, que sabe qual a porta premiada, as
probabilidades sao: 1 do premio estar atras da porta premiada e 0 de estar
atras de qualquer outra).

Nesse caso, voce seria louco de nao trocar de porta.

Um abraco,
Claudio.

on 11.08.03 23:27, Henrique Patrício Sant'Anna Branco at
[EMAIL PROTECTED] wrote:

 Por mais que eu ache pedante e ridiculo alguem se vangloriar de ter o QI
 mais alto do mundo, nesse caso acho que a Marilyn estah certa. Voce deve
 trocar de porta.
 
 Desculpem a minha ignorancia, mas o que ha de errado com o argumento de 1
 milhao de portas? Me parece que, nesse caso, a probabilidade de voce ter
 escolhido a porta certa de primeira eh apenas de 1/1.000.000. Logo, a
 probabilidade da outra porta ter o premio eh de 999.999/1.000.000. Ou nao?
 
 Cláudio,
 
 No problema original, temos três portas, escolhemos uma e o apresentador
 logo em seguida abre outra que, com certeza, não tem o prêmio. Inicialmente,
 havia uma chance de 1/3 de uma determinada porta conter o prêmio. Ao ser
 aberta uma das portas e mostrar que ela não contém o prêmio, sobram apenas
 duas portas: a que você escolheu e uma outra. É como se a probabilidade
 tivesse sido atualizada pelo fato do apresentador mostrar uma porta que
 não contém o prêmio (isso é o Teorema de Bayes se não me engano). Agora que
 sobraram apenas duas portas, cada uma delas tem uma em duas chances (1/2) de
 ter o prêmio e, portanto, não há justificativa (matematica) para trocar de
 porta ou não. O fato do apresentador abrir uma das portas muda a
 probabilidade das DUAS portas e não apenas para uma, como a Sra. Marilyn
 quer nos fazer crer.
 
 Quanto ao argumento de 1 milhão de portas... Como você disse, a
 probabilidade de você ter escolhido a porta certa de primeira é de 1/10^6
 que é a mesma probabilidade de cada uma das outras portas individualmente.
 Lembre-se que todas as probabilidades devem somar 1 = 10^6/10^6. O caso que
 você apontou (999.999/10^6) é a probabilidade combinada de todas as outras
 portas (cada uma entre as 10^6 portas têm probabilidade de 1/10^6) que você
 não escolheu de terem o prêmio e não de uma única porta das que você não
 escolheu. Se você simplesmente muda de porta, a probabilidade continua sendo
 a mesma... E, se ele abrir 777.777 portas sem o prêmio, a probabilidade de
 TODAS as portas fica em 1/222.223 e, novamente, não faz diferença mudar a
 porta...
 
 Espero ter sido claro.
 Abraço,
 Henrique.
 
 =
 Instruções para entrar na lista, sair da lista e usar a lista em
 http://www.mat.puc-rio.br/~nicolau/olimp/obm-l.html
 =
 

=
Instruções para entrar na lista, sair da lista e usar a lista em
http://www.mat.puc-rio.br/~nicolau/olimp/obm-l.html
=


[obm-l] Opo de Compra

2003-08-14 Por tôpico Cludio \(Prtica\)
Title: Help



Caros colegas:

No sei se algum vai se interessar pelo problema abaixo mas, dentro do 
mercado financeiro, a rea que mais utiliza mtodos matemticos avanados  a de 
avaliao de instrumentos derivativos, dos quais existem inmeros tipos, entre 
os quais as opes de compra - objeto do problema

*

O mercado de opes (e, mais geralmente, o de instrumentos derivativos) 
movimenta todos os dias bilhes de dlares em todo o mundo.A 
definiodesse instrumento  bem simples:
Uma Opo de Compra  um contrato entre duas partes que d a uma delas (o 
titular da opo) o direito, mas no a obrigao, de comprar um dado bem, por um 
preo fixo, numa certa data futura.

Por exemplo,suponhamos que um investidor compre uma opo de compra 
sobre uma certa ao que lhe d o direito de, dentro de 1 ano, comprar esta ao 
por R$ 100,00.

Ao fim de 1 ano, temos dois casos a considerar:
Caso 1: A ao est sendo cotada em bolsa a um preo superior a R$ 
100.
Nesse caso, o investidor deve exercer sua opo de comprar a ao, uma vez 
que ao fazer isso, ele estar adquirindo esta ao a um preo inferior quele 
disponvel na bolsa.
Por exemplo, se a ao estiver sendo cotada em bolsa a R$ 130, ao exercer a 
opo e adquirir a ao por apenas R$ 100, este investidor estar auferindo um 
lucro de R$ 30.

Caso 2: A ao est sendo cotada em bolsa a um preo superior a R$ 
100.
Nesse caso, o investidor deve abandonar a sua opo (abrir mo do seu 
direito) pois, se quiser, poder adquirir a ao por um preo menor diretamente 
na bolsa.
Por exemplo, se a ao estiver sendo cotada em bolsa a R$ 80, por que o 
investidor ir pagar R$ 100 por ela?

*

Feito esse prembulo explicativo, passemos ao problema matemtico de se 
avaliar uma opo de compra.

Vamos supor o caso simples (e irreal) de uma ao cujo preo hoje  R$ 100, 
e cujo preo, daqui a 1 ano,tem a seguinte distribuio 
probabilstica:
R$ 200, com probabilidade = p
e
R$ 50, com probabilidade = 1 - p.
Alm disso, vamos supor (tambm irrealmente) que seja possvel aplicar 
dinheiro e/ou tomar dinheiro emprestado a uma mesma taxa de juros, digamos de 
20% ao ano.

Pergunta: Quanto deve valer uma opo de compra que d o direito de se 
comprar esta ao daqui a 1 ano por R$ 110?

*

OBS1: Repare que, se daqui a 1 ano, a ao estiver valendo R$ 200, a opo 
permitir que o titular tenha um lucro de R$ 90 (200 - 110). Por outro lado, se 
a ao estiver valendo R$ 50, o titular dever simplesmente abandon-la. 
Assim,na pior das hipteses, o titular no perde nem ganha, e na melhor, 
ganha R$ 90. Logo,  razovel se esperar que esta opo valha alguma coisa 
hoje.

OBS2: Esse caso parece (de fato ) excesivamente simples e irreal. No 
entanto,  a base para o chamado modelo binomial de avaliao de opes, um dos 
modelos que  usado no mundo inteiro por investidores e instituies financeiras 
para avaliar opes de verdade.

OBS3: Opes tem sido negociadas desde a Renascena (e talvez antes disso), 
mas foi apenas em 1973 que dois acadmicos (Fisher Black e Myron Scholes - este 
ltimo ganhador do prmio Nobel de Economia em 1997) tiveram a "sacada" que 
revolucionou as tcnicas de avaliao e, consequentemente, o mercado mundial de 
derivativos.

Um abrao,
Claudio.




Re: [obm-l] ENQUETE - BELEZA MATEMATICA

2003-08-14 Por tôpico Nicolau C. Saldanha
On Sat, Aug 09, 2003 at 12:27:25PM -0300, A. C. Morgado wrote:
 8) Os teoremas belgas a respeito das seções cônicas.

Outro que eu não sei o que é.

[]s, N.
=
Instruções para entrar na lista, sair da lista e usar a lista em
http://www.mat.puc-rio.br/~nicolau/olimp/obm-l.html
=


Re: [obm-l] continuidade

2003-08-14 Por tôpico edalbuquerque
Oi Cláudio,

Talvez vc naum tenha observado que a função é f(x)=1/x²,não 
f(x)=1/x.De qualquer maneira,a resolução abaixo deuma 
encaminhada boa e acho que consegui terminar o problema.

Brigadão,

Eder

 on 10.08.03 20:58, edalbuquerque at [EMAIL PROTECTED]
 wrote:
 
  Como eu provo que f(x)=1/x² é contínua?
Melhor,como determinar
  o delta apropriado?
  
  Grato por qualquer ajuda.
  
  Eder
  
 Oi, Eder:
 
 Devemos ter cuidado pra definir f, pois seu dominio nao cont
em x = 0.
 
 Seja  a  0. Temos que provar que lim(x - a) 1/x = 1/a.
 
 
 Seja eps  0.
 
 Como a  0, teremos |a|  |a|/2  0
 
 Tomemos delta = min( a^2*eps/2, |a|/2 )
 
 |x - a|  delta ==
 
 a - delta  x  a + delta ==
 
 a - |a|/2  x  a + |a|/2 ==
 
 se a  0, entao 3a/2  x  a/2
 e
 se a  0, entao a/2  x  3a/2 ==
 
 de qualquer jeito, |x|  |a|/2 == 1/|x|  1/(|a|/2)
 
 Assim:
 |1/x - 1/a| = |x - a|/(|a||x|)  delta/
(|a||a|/2) = 2delta/a^2 = eps
 
 
 Um abraco,
 Claudio.
 
 
 PS: Acabei nao respondendo a sua pergunta. O delta apropriad
o voce acha
 resolvendo o problema de tras pra frente, ou seja, fazendo:
 
 |1/x - 1/a| = |x - a|/(|a||x|)  delta/(|a||x|) = eps ==
 
 delta = eps*|a|*|x|
 
 A partir desse ponto, voce soh precisa achar um limitante in
ferior para |x|
 (no caso, eu achei |a|/2).
 
 
 
 
=
 Instruções para entrar na lista, sair da lista e usar a list
a em
 http://www.mat.puc-rio.br/~nicolau/olimp/obm-l.html
 
=
 

 
---
Acabe com aquelas janelinhas que pulam na sua tela.
AntiPop-up UOL - É grátis! 
http://antipopup.uol.com.br

=
Instruções para entrar na lista, sair da lista e usar a lista em
http://www.mat.puc-rio.br/~nicolau/olimp/obm-l.html
=


Re: [obm-l] ENQUETE - BELEZA MATEMATICA

2003-08-14 Por tôpico Frederico Reis Marques de Brito
Cláudio:

 A Matemática é a única ciência em que uma geração não poe abaixo o que a 
anterior construiu. 

As idéias matemáticas belas são permanentes e  a sua enquete em nada remetia 
a que enviássemos apenas teoremas modernosos, ademais a genialidade dos 
Matemáticos da Carochinha  parece não ter mais espaço nos últimos anos ...

Perplexo, com seu comentário,

Frederico.


From: Johann Peter Gustav Lejeune Dirichlet 
[EMAIL PROTECTED]
Reply-To: [EMAIL PROTECTED]
To: [EMAIL PROTECTED]
Subject: Re: [obm-l] ENQUETE - BELEZA MATEMATICA
Date: Tue, 12 Aug 2003 15:15:20 -0300 (ART)

Ah e,e??Veja o primeiro capitulo do
Proofs from THE BOOK.Ai ce vai ver como topologia
e ultrapassado
 Claudio Buffara wrote:
  Estou extremamente decepcionado com as listas
 de problemas supostamente
  bonitos que foram enviadas pra lista ateh o
 presente momento. Imaginem soh -
  teorema do valor intermediario, secoes
 conicas, poliedros regulares,
  conjuntos enumeraveis. Onde voces estao com a
 cabeca? Isso tudo eh
  matematica do tempo da carochinha. Infinitude
 dos primos? Isso eh tao velho
  que ja devia ter sido revogado!

 O Noga Alon conta que fizeram esta pergunta
 para ele uma vez que ele
 começou explicando a prova de Euclides de que
 há infinitos primos
 em um programa de televisão, eu acho:

 And today, are there still infinitely many
 primes?

 E sem sair do clima, deem uma olhada em

http://qsilver.queensu.ca/~phil158d/intro/montyh3.htm

 Eu deveria ter visto isso antes de escrever o
 meu artigo da Eureka!

 []s, N.

=
 Instruções para entrar na lista, sair da lista
 e usar a lista em

http://www.mat.puc-rio.br/~nicolau/olimp/obm-l.html

=
___
Conheça o novo Cadê? - Mais rápido, mais fácil e mais preciso.
Toda a web, 42 milhões de páginas brasileiras e nova busca por imagens!
http://www.cade.com.br
=
Instruções para entrar na lista, sair da lista e usar a lista em
http://www.mat.puc-rio.br/~nicolau/olimp/obm-l.html
=
_
MSN Hotmail, o maior webmail do Brasil.  http://www.hotmail.com
=
Instruções para entrar na lista, sair da lista e usar a lista em
http://www.mat.puc-rio.br/~nicolau/olimp/obm-l.html
=


Re: [obm-l] ENQUETE - BELEZA MATEMATICA

2003-08-14 Por tôpico Bernardo Vieira Emerick
Caro Claudio,

Em termos de probabilidade não há razão para se mudar de opção, ou dee 
porta. Digamos que haja um milhão de portas. A probabilidade de se acertar 
em todas elas serão iguais entre si e iguai a 1/1.000.000.
Digamos que o apresentador abra 999.998 portas, ou seja, que só sobrem duas. 
Está certo: a probabilidade de se acertar aumentou; mas ela aumenta para 
ambas as portas. O que modificou foi o número total de possibilidades. 
Antes, tinha-se que escolher uma porta entre um milhão; depois, uma porta 
entre duas. No entanto, a probabilidade de se ganhar em ambas as portas é de 
1/2. Só seria necessário mudar a porta se após a abertura das outras portas 
a probabilidade de se acertar de uma porta ficasse maior do que a da outra.


From: Claudio Buffara [EMAIL PROTECTED]
Reply-To: [EMAIL PROTECTED]
To: [EMAIL PROTECTED]
Subject: Re: [obm-l] ENQUETE - BELEZA MATEMATICA
Date: Mon, 11 Aug 2003 19:53:06 -0300
on 11.08.03 19:03, Domingos Jr. at [EMAIL PROTECTED] wrote:

 O Noga Alon conta que fizeram esta pergunta para ele uma vez que ele
 começou explicando a prova de Euclides de que há infinitos primos
 em um programa de televisão, eu acho:

 And today, are there still infinitely many primes?

 E sem sair do clima, deem uma olhada em
 http://qsilver.queensu.ca/~phil158d/intro/montyh3.htm

 Eu deveria ter visto isso antes de escrever o meu artigo da Eureka!

 --- x ---
 Putz, essa mulher do QI mais alto do mundo (bullshit!) não concorda com 
o
 Princípio da Indução Finita também! hehehe, o pior é que é sério!!!

Oi, Domingos e Nicolau:

Por mais que eu ache pedante e ridiculo alguem se vangloriar de ter o QI
mais alto do mundo, nesse caso acho que a Marilyn estah certa. Voce deve
trocar de porta.
Desculpem a minha ignorancia, mas o que ha de errado com o argumento de 1
milhao de portas? Me parece que, nesse caso, a probabilidade de voce ter
escolhido a porta certa de primeira eh apenas de 1/1.000.000. Logo, a
probabilidade da outra porta ter o premio eh de 999.999/1.000.000. Ou nao?
Um abraco,
Claudio.
=
Instruções para entrar na lista, sair da lista e usar a lista em
http://www.mat.puc-rio.br/~nicolau/olimp/obm-l.html
=
_
MSN Messenger: converse com os seus amigos online.  
http://messenger.msn.com.br

=
Instruções para entrar na lista, sair da lista e usar a lista em
http://www.mat.puc-rio.br/~nicolau/olimp/obm-l.html
=


Re: [obm-l] Re: not subject

2003-08-14 Por tôpico Eduardo Casagrande Stabel
Olá Dirichlet,

eu também pensei sobre o problema: demonstrar que não existe uma função nos
reais contínua nos racionais e somente neles. Sequer tenho alguma estratégia
ou alguma idéia de como atacar o problema. Será que alguém pode dar uma
sugestão? O único progresso que fiz - que nem sei se está certo - é intuir
que os racionais não são um conjunto tão especial neste enunciado, eu
suspeito que podemos substituir por enumeraveis densos nos reais.

Quem quiser fazer comentários, sinta-se à vontade.

Abraço,
Duda.

From: Johann Peter Gustav Lejeune Dirichlet
[EMAIL PROTECTED]
 Como nao consegui demonstrar isto em tempo
 finito,alguem poderia demonstrar pra mim?


=
Instruções para entrar na lista, sair da lista e usar a lista em
http://www.mat.puc-rio.br/~nicolau/olimp/obm-l.html
=


Re: [obm-l] ENQUETE - BELEZA MATEMATICA

2003-08-14 Por tôpico Johann Peter Gustav Lejeune Dirichlet
Ah e,e??Veja o primeiro capitulo do 
Proofs from THE BOOK.Ai ce vai ver como topologia
e ultrapassado

 Claudio Buffara wrote:
  Estou extremamente decepcionado com as listas
 de problemas supostamente
  bonitos que foram enviadas pra lista ateh o
 presente momento. Imaginem soh -
  teorema do valor intermediario, secoes
 conicas, poliedros regulares,
  conjuntos enumeraveis. Onde voces estao com a
 cabeca? Isso tudo eh
  matematica do tempo da carochinha. Infinitude
 dos primos? Isso eh tao velho
  que ja devia ter sido revogado!
 
 O Noga Alon conta que fizeram esta pergunta
 para ele uma vez que ele
 começou explicando a prova de Euclides de que
 há infinitos primos
 em um programa de televisão, eu acho:
 
 And today, are there still infinitely many
 primes?
 
 E sem sair do clima, deem uma olhada em

http://qsilver.queensu.ca/~phil158d/intro/montyh3.htm
 
 Eu deveria ter visto isso antes de escrever o
 meu artigo da Eureka!
 
 []s, N.

=
 Instruções para entrar na lista, sair da lista
 e usar a lista em

http://www.mat.puc-rio.br/~nicolau/olimp/obm-l.html

= 

___
Conheça o novo Cadê? - Mais rápido, mais fácil e mais preciso.
Toda a web, 42 milhões de páginas brasileiras e nova busca por imagens!
http://www.cade.com.br
=
Instruções para entrar na lista, sair da lista e usar a lista em
http://www.mat.puc-rio.br/~nicolau/olimp/obm-l.html
=


Re: [obm-l] EsSA

2003-08-14 Por tôpico Bernardo Vieira Emerick
Existe uma solução mais rápida. Basta subtrairmos 900 por 888, que nos dará 
12. Como sairam dois trabalhadores, dividimos 20 por dois - já que as 
parcelas são iguais -, o que nos dá 6. Note-se que essa resposta só serve 
porque as parcelas de todos os trabalhadores permanecerá constante.


From: Henrique Patrício Sant'Anna Branco [EMAIL PROTECTED]
Reply-To: [EMAIL PROTECTED]
To: [EMAIL PROTECTED]
Subject: Re: [obm-l] EsSA
Date: Sat, 9 Aug 2003 15:09:15 -0300
 Numa fábrica, trabalhadores reuniram-se para
 presentear um amigo que iria casar. O presente
 escolhido foi a quantia de 900,00, que seria dividida
 igualmente entre eles. Por razões particulares, dois
 daqueles trabalhadores tiraram seus nomes da lista e,
 por isso, decidiu-se diminuir a quantia para 888,00,
 de modo que na nova divisão coubesse a cada
 participante a mesma cota de antes da saída dos dois
 colegas. Com isso, coube a cada um dos participantes a
 quantia de :
Vamos chamar de x o número de trabalhadores no início da divisão. Assim,
(x-2) é o número final de participantes. Como as cotas tem que ser iguais,
temos:
900/x = 888/(x-2) == 12x - 1800 = 0
Resolvendo, achamos x = 150. Agora, 900/150 = 6.
Segunda opção.
Abraços,
Henrique.
=
Instruções para entrar na lista, sair da lista e usar a lista em
http://www.mat.puc-rio.br/~nicolau/olimp/obm-l.html
=
_
MSN Hotmail, o maior webmail do Brasil.  http://www.hotmail.com
=
Instruções para entrar na lista, sair da lista e usar a lista em
http://www.mat.puc-rio.br/~nicolau/olimp/obm-l.html
=


Re: [obm-l] Geomeria Plana

2003-08-14 Por tôpico André Martin Timpanaro
Como DF=DE+EF, A=(75/5).2=30

André T.


From: elton francisco ferreira [EMAIL PROTECTED]
Reply-To: [EMAIL PROTECTED]
To: [EMAIL PROTECTED]
Subject: [obm-l] Geomeria Plana
Date: Mon, 11 Aug 2003 15:35:59 -0300 (ART)
Um triângulo ABC tem área 75m^2. os pontos D,E,F e G
dividem o lado AC em 5 partes congruentes:
AD=DE=EF=FG=GC. Desse modo, a área do triangulo BDF é:
20
30
40
50
55
___
Conheça o novo Cadê? - Mais rápido, mais fácil e mais preciso.
Toda a web, 42 milhões de páginas brasileiras e nova busca por imagens!
http://www.cade.com.br
=
Instruções para entrar na lista, sair da lista e usar a lista em
http://www.mat.puc-rio.br/~nicolau/olimp/obm-l.html
=
_
MSN Messenger: converse com os seus amigos online.  
http://messenger.msn.com.br

=
Instruções para entrar na lista, sair da lista e usar a lista em
http://www.mat.puc-rio.br/~nicolau/olimp/obm-l.html
=


Re: [obm-l] ENQUETE - BELEZA MATEMATICA

2003-08-14 Por tôpico Frederico Reis Marques de Brito
1) Acho que esse será praticamente unânime: Teorema de Euclides sobre a 
exist~encia de  infinitos primos.

2) Teorema de Bezout sobre MDC: O máximo dvisor comum de dois inteiros é uma 
comb. linear inteira ( em realidade a menor positiva ) desses números , 
pelas várias aplicações deste na Teoria dos Números.

3) O Princípio de Dirichlet, pela potência .

4) O posto-linha = posto-coluna. Não sei mais sempre achei este resultado 
muito inusitado, já que uma matriz nada mais é que um amontoado de 
números...

5) Teorema do Valor Intermediário ( Acho que podemos abordar este tema no 
ensino médio... )

Como o Morgado, pensarei um pouco mais antes de enviar outros 5. (  A 
propósito é tentador citar o Teorema dos Números Primos, mas acho que esse 
tema não seria acessível. Fica um voto de louvor então!)

Frederico.

From: Claudio Buffara [EMAIL PROTECTED]
Reply-To: [EMAIL PROTECTED]
To: Lista OBM [EMAIL PROTECTED]
CC: Claudio Buffara [EMAIL PROTECTED]
Subject: [obm-l] ENQUETE - BELEZA MATEMATICA
Date: Sat, 09 Aug 2003 10:24:26 -0300
Caros colegas da lista:

Gostaria de contar com sua participacao numa enquete sobre beleza
matematica.
O que eu precisao eh que cada um de voces me envie uma lista contendo algo
como 5 a 10 problemas/teoremas que voces consideram os mais bonitos e cujas
solucoes/demonstracoes sao as mais elegantes e/ou inusitadas e/ou
engenhosas. Nao precisa incluir a solucao/demonstracao, apenas o enunciado.
No entanto, se voce tiver em mente uma solucao/demonstracao especifica
(entre varias existentes) nao deixe de mencionar pelo menos o metodo
utilizado.
A unica restricao eh que estes resultados devem ser de um nivel acessivel a
um aluno normal de 2o. grau (ou seja, o Ultimo Teorema de Fermat e o 
Porisma
de Poncelet estao fora, mas o caso n = 4 do UTF e a versao para triangulos
do Porisma poderiam ser incluidos).

Importante: os resultados devem ser acessiveis a um aluno normal de 2o.
grau, mas nao necessariamente fazer parte do curriculo normal do 2o. grau.
Tambem nao precisa responder hoje ou amanha ou mesmo na semana que vem. 
Acho
que vale a pena pensar por um tempo e consultar a literatura - as vezes 
pode
ter um resultado belissimo do qual voce simplesmente se esqueceu por nao
encontra-lo ha muito tempo. As Eurekas sao uma otima referencia. O Proofs
from the Book tambem, apesar de nem tudo lah ter nivel de 2o. grau.

Se houver um numero suficiente de respostas, eu me comprometo a publicar 
uma
compilacao dos problemas e teoremas mais votados.

Desde jah a gradeco o interesse de quem quiser participar.

Um abraco,
Claudio.
=
Instruções para entrar na lista, sair da lista e usar a lista em
http://www.mat.puc-rio.br/~nicolau/olimp/obm-l.html
=
_
MSN Hotmail, o maior webmail do Brasil.  http://www.hotmail.com
=
Instruções para entrar na lista, sair da lista e usar a lista em
http://www.mat.puc-rio.br/~nicolau/olimp/obm-l.html
=


[obm-l] geometria

2003-08-14 Por tôpico elton francisco ferreira
A a´rea de um triangulo de perímetro 54m circunscrito
a um círcuo de 25pim^2, em m^2 e´?

___
Conheça o novo Cadê? - Mais rápido, mais fácil e mais preciso.
Toda a web, 42 milhões de páginas brasileiras e nova busca por imagens!
http://www.cade.com.br
=
Instruções para entrar na lista, sair da lista e usar a lista em
http://www.mat.puc-rio.br/~nicolau/olimp/obm-l.html
=


Re: [obm-l] Ajuda

2003-08-14 Por tôpico Cláudio \(Prática\)
Oi, Thyago:

A solução padrão pra esse tipo de problema realmente envolve complexos e
polinômios.

Tentando resolver outros problemas similares, você vai perceber que
complexos e polinômios são uma forma de resolução bastante natural.

Os resultados básicos são os seguintes:
1) Todo número complexo pode ser representado na forma R*(cos(a) +
i*sen(a)), onde R é um real não negativo e a é um real qualquer (mas
normalmente limitado ao intervalo [0, 2pi) ou então (-pi,pi]);
2) e^(i*a) = cos(a) + i*sen(a): essa é a definição da função exponencial
complexa, que permite, por exemplo, que você transforme sequências de senos
e cossenos de números reais em PA em sequências de complexos em PG, que as
vezes são mais fáceis de manipular;
3) Um polinômio com coeficientes reais pode ser expresso como o produto de
binômios da forma (x - b) e/ou trinômios da forma (x^2 - 2*R*cos(a)*x +
R^2), onde a e b são números reais quaisquer e R é um real positivo.

Um abraço,
Claudio.


- Original Message -
From: dex [EMAIL PROTECTED]
To: [EMAIL PROTECTED]
Sent: Monday, August 11, 2003 11:05 AM
Subject: [obm-l] Ajuda


 Olá pessoal

 Gostaria de saber uma boa demonstração para o exercício abaixo

 P = sen(pi/n) . sen(2pi/n) . sen(3pi/n) . ... . sen[(n-1)pi/n]
 com n Inteiro positivo

 A resposta é P = n/[2^(n-1)], mas cheguei até este resultado de uma
maneira
 muito pouco prática, nada natural para uma questão de matemática (de
 vestibular). Consegui prová-la utilizando o resultado de uma outra
questão,
 que versava sobre polinômios e complexos. Ou seja, se eu não tivesse visto
 esta outra questão não conseguiria provar nada!

 Atneciosamente
 ¡Thyago!


=
Instruções para entrar na lista, sair da lista e usar a lista em
http://www.mat.puc-rio.br/~nicolau/olimp/obm-l.html
=


Re: [obm-l] ENQUETE - BELEZA MATEMATICA

2003-08-14 Por tôpico Johann Peter Gustav Lejeune Dirichlet
Um que e o meu preferido, alias o meu par teorema-problema preferido:a 
Desigualdade de Erdös-Mordell e o problema de geometria da IMO 1996.Ja 
enviei ha bastante tempo uma mensagem completa com a solução do segundo mas 
poucos tiveram tempo de ler (eu mesmo nem tive tempo de rever esta perola 
problemistica!!!).Quem achar uma desigualdade mais legal,meus parabens.Caso 
contrario continue tentando!!
Ass.;


From: Eduardo Casagrande Stabel [EMAIL PROTECTED]
Reply-To: [EMAIL PROTECTED]
To: [EMAIL PROTECTED]
Subject: Re: [obm-l] ENQUETE - BELEZA MATEMATICA
Date: Tue, 12 Aug 2003 16:32:27 -0300
Olá!

É minha vez de enviar meus problemas/teoremas bonitos...

1) O teorema, devido a Euler, que diz que quando s  1 temos ZETA(s) =
SOMA{ 1/n^s, n=1...infinito } = PRODUTORIO { (1 - p^(-s) ), p primo }.
2) A surpreendente constatação de que um problema aparentemente não tão
complicado como o último teorema de Fermat tenha uma solução tão extensa e
complicada.
3) A demonstração (enviada para a lista) de que um número irracional 
elevado
a um número irracional pode resultar um número racional. A saber, se p =
raiz(2) ^ raiz(2) é racional, está acabado; se p não for racional, é
irracional, e q = p ^ raiz(2) = 2 satisfaz o problema.

4) O método (não lembro de quem é, talvez Cauchy) para aproximar a soma de
séries SOMA{ s_n } onde s_1  s_2  s_3  ...  0, utilizando-se uma
integral.
5) A constatação maravilhosa de que certas constantes (como Pi e e) são
constantes em toda a matemática, isto é, aparecem em diversas áreas
(aparentemente desconexas) como geometria, análise, teoria dos números,
probabilidade, etc. dando a entender que toda a matemática tem um centro
firme (de onde saem os resultados) e uma arquitetura já planejada por 
alguém
bem mais inteligente que nós...

6) O método do escalonamento de matrizes descoberto por Gauss. Brilhante 
por
ser tão simples e ter levado tanto tempo para ser descoberto. Segundo o
Gilbert Strang: Ele é tão simples que mesmo qualquer um de nós poderíamos
tê-lo descoberto

7) O fato de que podemos definir num espaço vetorial uma função com
propriedades simples (=o produto interno) e dele derivarmos muitos e muitos
resultados interessantes. (isto realmente me surpreendeu, quando comecei a
estudar álgebra linear, pareceu mágica a existência e as conseqüências do
produto interno)
8) A menos de isomorfismo, o conjunto dos Reais é o único corpo ordenado
completo.
9) O primeiro teorema da inconsistência de Gödel.

E para finalizar:

10) O princípio da indução finita.

Abração,
Duda.
From: Claudio Buffara [EMAIL PROTECTED]
 Caros colegas da lista:

 Gostaria de contar com sua participacao numa enquete sobre beleza
 matematica.

 O que eu precisao eh que cada um de voces me envie uma lista contendo 
algo
 como 5 a 10 problemas/teoremas que voces consideram os mais bonitos e
cujas
 solucoes/demonstracoes sao as mais elegantes e/ou inusitadas e/ou
 engenhosas. Nao precisa incluir a solucao/demonstracao, apenas o
enunciado.
 No entanto, se voce tiver em mente uma solucao/demonstracao especifica
 (entre varias existentes) nao deixe de mencionar pelo menos o metodo
 utilizado.

 A unica restricao eh que estes resultados devem ser de um nivel 
acessivel
a
 um aluno normal de 2o. grau (ou seja, o Ultimo Teorema de Fermat e o
Porisma
 de Poncelet estao fora, mas o caso n = 4 do UTF e a versao para 
triangulos
 do Porisma poderiam ser incluidos).

 Importante: os resultados devem ser acessiveis a um aluno normal de 2o.
 grau, mas nao necessariamente fazer parte do curriculo normal do 2o. 
grau.

 Tambem nao precisa responder hoje ou amanha ou mesmo na semana que vem.
Acho
 que vale a pena pensar por um tempo e consultar a literatura - as vezes
pode
 ter um resultado belissimo do qual voce simplesmente se esqueceu por nao
 encontra-lo ha muito tempo. As Eurekas sao uma otima referencia. O 
Proofs
 from the Book tambem, apesar de nem tudo lah ter nivel de 2o. grau.

 Se houver um numero suficiente de respostas, eu me comprometo a publicar
uma
 compilacao dos problemas e teoremas mais votados.

 Desde jah a gradeco o interesse de quem quiser participar.

 Um abraco,
 Claudio.


 
=
 Instruções para entrar na lista, sair da lista e usar a lista em
 http://www.mat.puc-rio.br/~nicolau/olimp/obm-l.html
 
=



=
Instruções para entrar na lista, sair da lista e usar a lista em
http://www.mat.puc-rio.br/~nicolau/olimp/obm-l.html
=
_
MSN Messenger: converse com os seus amigos online.  
http://messenger.msn.com.br

=
Instruções para entrar na lista, sair da lista e usar a lista em

[obm-l] Re: [obm-l] Problema das Tres Portas

2003-08-14 Por tôpico bmat
Bom, quanto aos argumentos matemáticos, acho que o Cláudio já falou tudo.
Mas é impressionante como este problema é difícil, pois ele envolve separar
o que da informação do apresentador é útil e o que é inútil. Este problema
me foi exposto recentemente durante o Colóquio Brasileiro de Matemática,
numa palestra de Probabilistic Reasoning, que citou outro problema que
o Nicolau comenta no seu artigo da Eureka!, também bastante contra-intuitivo
e genial.

Mas, voltando ao problema: Quando você escolhe a porta, qual a probabilidade
de ela estar certa? 1/3, correto?
Ok. O que acontece quando o apresentador abre uma porta? Ele diz para você:
Olha, se não estiver na que você escolheu, está na outra!!! Você agora
pode escolher entre duas portas, certo? Mas isso não quer dizer que as probabilidades
são iguais, pois você não ganhou NADA em relação à sua porta, já que o apresentador
não pode fazer NADA com a sua (ele não escolhe se abre a sua ou não, ele
NÃO pode abrir a sua). Mas a outra ele ESCOLHEU não abrir. Ou seja, ele
ESCOLHEU abrir a outra.

Pense assim: O prêmio pode estar na 1, na 2 ou na 3. Você escolhe a 1. O
apresentador pode abrir a 2 ou a 3, apenas. Se o prêmio estivesse numa delas,
ele FORÇOSAMENTE terá que abrir a OUTRA! OU seja, se você errou, trocando
você passa a ganhar. Por outro lado, se você acertou na escolha inicial,
tanto faz para o apresentador, ele pode abrir qualquer uma, que não tem
restrição.
Ou seja, a informação relevante é a seguinte: se você trocar, a probabilidade
de ganhar é a probabilidade de você ter errado. (o que talvez fique bem
claro com o exemplo de 1.000.000 de portas)

A grande sacada consiste em você se convencer de que, se você trocar,
você vai ganhar SEMPRE que tiver errado o primeiro palpite. Uma vez isso,
o jogo é fácil: você tem 1/3 de chances de acertar e o resto é erro! Ao
abrir as portas, o apresentador consolida o resto do jogo: ao trocar, você
troca seu estado, de certo para errado e vice-versa só que como P(errar
o primeiro palpite)  P(acertar o primeiro palpite), vale a pena trocar.

É isso... qualquer dúvida, pergunte.
Bernardo Costa

-- Mensagem original --

Claudio,

Matematicamente, tanto faz, porque a probabilidade de se ganhar é a mesma.

Agora, se você for supersticioso, a coisa muda um pouco de figura. Mas
eu,

pessoalmente, não mudaria.
Abraços,
Bernardo


From: Claudio Buffara [EMAIL PROTECTED]
Reply-To: [EMAIL PROTECTED]
To: [EMAIL PROTECTED]
Subject: [obm-l] Problema das Tres Portas
Date: Mon, 11 Aug 2003 22:58:22 -0300

Bernardo:

Pra resumir, qual eh a sua conclusao? O jogador deve ou nao deve trocar
de
porta?

Claudio.

on 11.08.03 21:51, Bernardo Vieira Emerick at [EMAIL PROTECTED]
wrote:

  Que piada!!! Marylin vos Savant, tida como a pessoa com o maior QI
do

mundo
  (concordo com o Domingos Jr.: bulsshit!) confundiu tudo. O problema
era
  assim: num jogo, a pessoa escolha uma entre três portas. O apresentador,
  então, abra uma das portas. Como ele sabe qual é a porta que contém
o
  prêmio, ele abre uma que não o contém - já que o jogo dar-se-ia por
  encerrado. A pergunta é: o jogador deveria trocar de porta?
  Segundo Marylin, sim!, porque a probabilidade da opção que ele teria
  continuaria 1/3, enquanto a outra aumentaria para 2/3!!! Qual a razão

disso?
  A probabilidade da porta que ele escolheu não poderia subir subitamente

para
  1/2, como sugerem os matemáticos. Ora, como então a outra porta pode???

Isso
  ela não explica.
  Ela aparentemente desconhece o conceito primeiro de probabilidade,
que
é 
a
  chance de se acertar, e por isso está atrelada ao número de 
possibilidades
  possíveis e o número de possibilidades requeridas para se acertar
o
  resultado. Então, a probabilidade será dada - como é de conhecimento

geral,
  exceto possivelmente de Marylin - pela divisão do número de 
possibilidades
  requeridas pelo número total de possibilidades. Parece-me que ela

acredita
  que a única forma de se aumentar a probabilidade é aumentando o número

de
  possibilidades requeridas. Isso justificaria o we've learned nothing

to
  allow us to revise the chances on the shell under your finger que
ela

diz.
  O que mudou, e que ela incrivelmente não percebeu, é o número total
de
  possibilidades. Simplificando para ela, o numerozinho de baixo diminuiu,
  então o número do outro lado do sinal de igualdade aumentou, já que
o
  numerozinho de cima da fração permaneceu constante. Será que assim
ela
  entenderia???
 
 
 
  From: Domingos Jr. [EMAIL PROTECTED]
  Reply-To: [EMAIL PROTECTED]
  To: [EMAIL PROTECTED]
  Subject: Re: [obm-l] ENQUETE - BELEZA MATEMATICA
  Date: Mon, 11 Aug 2003 19:03:11 -0300
 
  O Noga Alon conta que fizeram esta pergunta para ele uma vez que ele
  começou explicando a prova de Euclides de que há infinitos primos
  em um programa de televisão, eu acho:
 
  And today, are there still infinitely many primes?
 
  E sem sair do clima, deem uma olhada em
  http://qsilver.queensu.ca/~phil158d/intro/montyh3.htm
 
  

Re: [obm-l] ENQUETE - BELEZA MATEMATICA

2003-08-14 Por tôpico Helder Suzuki
 --- Henrique_Patrício_Sant'Anna_Branco
[EMAIL PROTECTED] escreveu:   Por mais que
eu ache pedante e ridiculo alguem se
 vangloriar de ter o QI
  mais alto do mundo, nesse caso acho que a Marilyn
 estah certa. Voce deve
  trocar de porta.
 
  Desculpem a minha ignorancia, mas o que ha de
 errado com o argumento de 1
  milhao de portas? Me parece que, nesse caso, a
 probabilidade de voce ter
  escolhido a porta certa de primeira eh apenas de
 1/1.000.000. Logo, a
  probabilidade da outra porta ter o premio eh de
 999.999/1.000.000. Ou nao?
 
 Cláudio,
 
 No problema original, temos três portas, escolhemos
 uma e o apresentador
 logo em seguida abre outra que, com certeza, não tem
 o prêmio. Inicialmente,
 havia uma chance de 1/3 de uma determinada porta
 conter o prêmio. Ao ser
 aberta uma das portas e mostrar que ela não contém o
 prêmio, sobram apenas
 duas portas: a que você escolheu e uma outra. É como
 se a probabilidade
 tivesse sido atualizada pelo fato do apresentador
 mostrar uma porta que
 não contém o prêmio (isso é o Teorema de Bayes se
 não me engano). Agora que
 sobraram apenas duas portas, cada uma delas tem uma
 em duas chances (1/2) de
 ter o prêmio e, portanto, não há justificativa
 (matematica) para trocar de
 porta ou não. O fato do apresentador abrir uma das
 portas muda a
 probabilidade das DUAS portas e não apenas para uma,
 como a Sra. Marilyn
 quer nos fazer crer.
 
 Quanto ao argumento de 1 milhão de portas... Como
 você disse, a
 probabilidade de você ter escolhido a porta certa de
 primeira é de 1/10^6
 que é a mesma probabilidade de cada uma das outras
 portas individualmente.
 Lembre-se que todas as probabilidades devem somar 1
 = 10^6/10^6. O caso que
 você apontou (999.999/10^6) é a probabilidade
 combinada de todas as outras
 portas (cada uma entre as 10^6 portas têm
 probabilidade de 1/10^6) que você
 não escolheu de terem o prêmio e não de uma única
 porta das que você não
 escolheu. Se você simplesmente muda de porta, a
 probabilidade continua sendo
 a mesma... E, se ele abrir 777.777 portas sem o
 prêmio, a probabilidade de
 TODAS as portas fica em 1/222.223 e, novamente, não
 faz diferença mudar a
 porta...
 
 Espero ter sido claro.
 Abraço,
 Henrique.

Oi Henrique!

Faz diferença mudar de porta nesses casos.
Se tem 1 milhão de portas, com apenas 1 premiada, a
chance de vc escolher ela é 10^-6
se o cara abre uma porta que não tem o prêmio, vc pode
mudar de porta e ter uma chance de (10^6-2)/(10^6-1)
de acertar a porta certa, caso vc não tenha pego ela
na primeira escolha.

[]'s,
Hélder T. Suzuki

___
Conheça o novo Cadê? - Mais rápido, mais fácil e mais preciso.
Toda a web, 42 milhões de páginas brasileiras e nova busca por imagens!
http://www.cade.com.br
=
Instruções para entrar na lista, sair da lista e usar a lista em
http://www.mat.puc-rio.br/~nicolau/olimp/obm-l.html
=


Re: [obm-l] EsSA

2003-08-14 Por tôpico Henrique Patrcio Sant'Anna Branco
  3  Sendo x= 19 e y= 81, ento a expresso (x+y)^2 +
  x^2  y^2 + 2x  divisvel por:
  a)2,19 e 81
  b)2,19 e 101
  c)2,81 e 100
  d)19,100 e 101
  e)81,100 e 101
 
  achei a letra B

 (x+y)^2 + x^2  y^2 + 2x

 100^2 +((x+y)*(x-y)) + 38

 1 + (100*(-62)) + 38

 =3838 que eh dividsivel por 101, mas nao por 2,19...2,81...100...

 O que fiz de errado ?

Fael,

3838  divisvel por 2 e 19 sim.
3838/2 = 1919/19 = 101/101 = 1
Alis, segundo minha HP, so os nicos fatores primos desse nmero... :)
D uma olhada na minha soluo (desenvolvendo a expresso). Acho mais
simples...

Abraos,
Henrique.

=
Instruções para entrar na lista, sair da lista e usar a lista em
http://www.mat.puc-rio.br/~nicolau/olimp/obm-l.html
=


[obm-l] Fwd: Fabrício sem avião 2

2003-08-14 Por tôpico Olimpiada Brasileira de Matematica


Date: Wed, 13 Aug 2003 08:40:44
-0300 (ART)
From: Paulo Rodrigues [EMAIL PROTECTED]
Subject: Fabrício sem avião 2
To: [EMAIL PROTECTED]
X-Spam-Status: No, hits=1.8 required=5.0
tests=HTML_00_10,HTML_FONT_BIG,HTML_FONT_COLOR_GRAY,HTML_MESSAGE,

HTML_TAG_EXISTS_TBODY
version=2.55
X-Spam-Level: *
X-Spam-Checker-Version: SpamAssassin 2.55
(1.174.2.19-2003-05-19-exp)

Escola privada carimba passaportes

Fábio Campos

[13 Agosto 04h22min]


Certas histórias tocam fundo a alma dos leitores. A do estudante Fabrício
Benevides, o sem-passagem, provocou a indignação de muitos. Fabrício é
aluno da graduação em ciências da Computação (UFC). Passou pelas
acirradas seletivas nacionais e se classificou para representar o Brasil
nas Olimpíadas Internacionais de Matemática, em Bucareste, Romênia. A
disputa já aconteceu, mas Fabrício não concorreu porque sua universidade
e as instituições que criamos para promover o progresso da
ciência negaram-lhe uma passagem aérea. A mesquinharia só faz
vítimas entre os que freqüentam os bancos da escola pública. A garotada
caxias que estuda nos melhores colégios da gloriosa Aldeota
carimba seu passaporte com a inestimável ajuda do contribuinte cearense.
E os melhores colégios estampam anúncios com o feito. 


OS AVIÕES APENAS SOBREVOAM O PICI 
Quando era aluno de uma escola particular, Fabrício nunca deixou de
viajar para disputar uma olimpíada. É um recorrente ganhador de medalhas.
Colocou-as no peito inclusive nas Olimpíadas Mundiais de 1999 e 2000.
Sempre com o apoio de seu colégio, tivesse ou não a sempre bem vinda
ajudinha das instituições públicas. Quando queimava as pestanas numa
escola inscrita na junta comercial, Fabrício costumava ganhar a mesma
atenção da que foi dedicada a Larissa Lima, 17 anos. Veja a notícia
divulgada no sítio
(www.sct.ce.gov.br)
da Secitece: Ao saber que o governo do Ceará iria patrocinar a
participação de Larissa na Global Young Leaders Conference (3 a 14 de
agosto, nos EUA), o secretário da CT, Hélio Barros, propôs que a
entrega do cheque de R$ 7.500 fosse feita pelo governador Lúcio
Alcântara. O pedido foi aceito e a solenidade ocorreu no Palácio Iracema,
dia 4 de maio. Pois é. O aluno da pública e gratuita UFC ficou a
ver aviões. A questão é: porque nossos gestores foram sensíveis a um caso
e tão impiedosos com outro? 


UMA AULA DE BOM SENSO E EQUILÍBRIO 
É de se esperar rompantes de revolta do estudante que, após a via-crucis
pelos gabinetes de instituições públicas, não conseguiu o apoio para
disputar as Olimpíadas Internacionais. O rancor seria próprio da idade e
das circunstâncias. Ledo engano. Mensagem enviada à Coluna pelo próprio
Fabrício desmente tal idéia. Fiquei surpreso e lisonjeado ao ler
sua coluna no Jornal O POVO. Nela, você trata de maneira
bastante crítica o descaso da administração pública e a falta de
incentivo a nós estudantes. De fato, participo de Olimpíadas de
Matemática desde a 7ª Série do Ensino Fundamental. Já tive diversas
oportunidades de viajar e trazer prêmios para o Brasil, inclusive nas
Olimpíadas Mundiais de 1999 e 2000. Nesta época, sempre tive apoio total
do meu colégio. Não com grande surpresa, percebi que na UFC as coisas se
passam de maneira bastante diferente. Assim, venho por meio deste,
agradecer-lhe pela publicação. O grifo a seguir é do editor:
Espero que a mesma tenha sido lida por nossas autoridades
universitárias e que a partir desse momento elas passem a refletir melhor
nas suas tomadas de decisão, procurando valorizar muito mais a todos nós
que fazemos a Universidade. 


MARASMO DE TECNOCRATAS CORPORATIVISTAS 
O caso despertou a indignação de leitores. Um deles foi muito duro com a
UFC. Apesar de ter cometido o pecado da generalização, vale fazer
reflexão acerca de suas ácidas palavras: O mais incrível é
verificar que os nossos professores da UFC levam um banho no que se
refere a gestão e marketing dos gestores das escolas particulares. Logo
eles que deveriam ser referencial de excelência. O que vemos são
profissionais corporativistas, pouco interessados nas demandas sociais,
agarrados aos seus empregos e sem a menor idéia de suas funções perante a
sociedade. São esses comportamentos que favorecem a idéia daqueles que
propõem a privatização de tudo. Aonde encontrar argumentos para defender
a universidade pública enterrada no marasmo do seus tecnocratas
incompetentes, que só enxergam seus contracheques e a reforma da
previdência? 


AS NUANCES DO TER E DO NÃO TER 
De gente que convive com Fabrício na UFC veio o seguinte: A
participação do Fabrício na Olímpiada não era imprescindível à sua
carreira. Ele fará carreira brilhante de qualquer forma, dada a sua
capacidade intelectual e emocional. Entretanto, ele teria orgulho, e para
tanto fez a sua parte, em representar o Brasil e o Ceará. Com certeza, a
sua participação contribuiria de maneira efetiva para o sucesso da equipe
brasileira. Para o Fabrício, na ocasião, disse algumas palavras de
consolo e encorajamento. Voltarei a fazê-lo. Mas, é difícil 

Re: [obm-l] ENQUETE - BELEZA MATEMATICA

2003-08-14 Por tôpico Bernardo Vieira Emerick
Caros Colegas,

Gostaria de pedir desculpas a todos pelos comentários horrorosos que teci 
sobre Marilyn vos Savant. Primeiro, porque ela acertou o problema que errei, 
e mais, que reiterei meu erro. Cabe aqui um agradecimento especial ao 
Claudio que solucionou corretamente, enquanto eu e o Henrique erramos. Eu, 
por outro lado, errei mais ao tentar ridicularizar alguém que estava correta 
o tempo inteiro. Explico aqui o meu erro, que creio ser o mesmo do Henrique, 
para que outros entendam bem: o problema não permitia que a análise fosse 
feita através de dois jogos diferentes. Se assim fosse, as probabilidades 
das duas portas seriam iguais. Como bem explicou o Claudio, o problema está 
na primeira escolha, porque ela altera a probabilidade do que considerei 
como o segundo jogo. O pior que fiz, e do que agora me arrependo 
profundamente, parece ter sido influenciar a última mensagem do Dirichlet: 
Colegas,nao acreditem em testes de QI. A ele peço desculpas, em 
especial.
Estou escrevendo esse texto para que outros não façam a mesma bobagem que 
eu. Antes de comentar algo, reflita bem sobre as possibilidades. Eu não fiz 
isso, e olha no que deu. Agora fica claro porque ela tem o QI mais alto do 
mundo (reitero: bullshit!!!), e eu não
Desculpas sinceras, e, desde já, agradeço pela compreensão da minha 
ignorância,
SAPIENTIAM AUTEM NON VICIT MALITIA,
Bernardo


From: Johann Peter Gustav Lejeune Dirichlet 
[EMAIL PROTECTED]
Reply-To: [EMAIL PROTECTED]
To: [EMAIL PROTECTED]
Subject: Re: [obm-l] ENQUETE - BELEZA MATEMATICA
Date: Tue, 12 Aug 2003 14:36:30 -0300 (ART)

Colegas,nao acreditem em testes de QI

 --- Bernardo Vieira Emerick
[EMAIL PROTECTED] escreveu:  Que
piada!!! Marylin vos Savant, tida como a
 pessoa com o maior QI do mundo
 (concordo com o Domingos Jr.: bulsshit!)
 confundiu tudo. O problema era
 assim: num jogo, a pessoa escolha uma entre
 três portas. O apresentador,
 então, abra uma das portas. Como ele sabe qual
 é a porta que contém o
 prêmio, ele abre uma que não o contém - já que
 o jogo dar-se-ia por
 encerrado. A pergunta é: o jogador deveria
 trocar de porta?
 Segundo Marylin, sim!, porque a probabilidade
 da opção que ele teria
 continuaria 1/3, enquanto a outra aumentaria
 para 2/3!!! Qual a razão disso?
 A probabilidade da porta que ele escolheu não
 poderia subir subitamente para
 1/2, como sugerem os matemáticos. Ora, como
 então a outra porta pode??? Isso
 ela não explica.
 Ela aparentemente desconhece o conceito
 primeiro de probabilidade, que é a
 chance de se acertar, e por isso está atrelada
 ao número de possibilidades
 possíveis e o número de possibilidades
 requeridas para se acertar o
 resultado. Então, a probabilidade será dada -
 como é de conhecimento geral,
 exceto possivelmente de Marylin - pela divisão
 do número de possibilidades
 requeridas pelo número total de
 possibilidades. Parece-me que ela acredita
 que a única forma de se aumentar a
 probabilidade é aumentando o número de
 possibilidades requeridas. Isso justificaria
 o we've learned nothing to
 allow us to revise the chances on the shell
 under your finger que ela diz.
 O que mudou, e que ela incrivelmente não
 percebeu, é o número total de
 possibilidades. Simplificando para ela, o
 numerozinho de baixo diminuiu,
 então o número do outro lado do sinal de
 igualdade aumentou, já que o
 numerozinho de cima da fração permaneceu
 constante. Será que assim ela
 entenderia???



 From: Domingos Jr. [EMAIL PROTECTED]
 Reply-To: [EMAIL PROTECTED]
 To: [EMAIL PROTECTED]
 Subject: Re: [obm-l] ENQUETE - BELEZA
 MATEMATICA
 Date: Mon, 11 Aug 2003 19:03:11 -0300
 
 O Noga Alon conta que fizeram esta pergunta
 para ele uma vez que ele
 começou explicando a prova de Euclides de que
 há infinitos primos
 em um programa de televisão, eu acho:
 
 And today, are there still infinitely many
 primes?
 
 E sem sair do clima, deem uma olhada em

http://qsilver.queensu.ca/~phil158d/intro/montyh3.htm
 
 Eu deveria ter visto isso antes de escrever o
 meu artigo da Eureka!
 
 --- x ---
 Putz, essa mulher do QI mais alto do mundo
 (bullshit!) não concorda com o
 Princípio da Indução Finita também! hehehe, o
 pior é que é sério!!!
 

=
 Instruções para entrar na lista, sair da lista
 e usar a lista em

http://www.mat.puc-rio.br/~nicolau/olimp/obm-l.html

=


_
 MSN Hotmail, o maior webmail do Brasil.
 http://www.hotmail.com


=
 Instruções para entrar na lista, sair da lista
 e usar a lista em

http://www.mat.puc-rio.br/~nicolau/olimp/obm-l.html

=
___
Conheça o novo Cadê? - Mais rápido, mais fácil e mais 

[obm-l] probleminha

2003-08-14 Por tôpico elton francisco ferreira
José se deslocou entre as cidades A e B tres vezes
pelo mesmo caminho, utilizando, em cada uma das vezes,
um meio de transporte diferente. Na primeira ves foi
de carro, com uma velocidade média de 60 Km/h. Na
segunda vez doi de bicicleta, com velocidade média de
30 Km/h, e na terceira vez foi de moto, com velocidade
média de 40 km/h. Sabendo que a soma dos tempos gastos
nos tres deslocamentos doi igual a 45 h, o tempo gasto
em cada um dos deslocamentos foi, respectivamente:

a) 11h;22h e 12h
b)12h; 25h e 7,5 h
c)10h; 20h e 15h
d)12h; 24h e 9h
e)10,5h;21h e 13,5

___
Conheça o novo Cadê? - Mais rápido, mais fácil e mais preciso.
Toda a web, 42 milhões de páginas brasileiras e nova busca por imagens!
http://www.cade.com.br
=
Instruções para entrar na lista, sair da lista e usar a lista em
http://www.mat.puc-rio.br/~nicolau/olimp/obm-l.html
=


Re: [obm-l] continuidade

2003-08-14 Por tôpico Claudio Buffara
on 10.08.03 20:58, edalbuquerque at [EMAIL PROTECTED] wrote:

 Como eu provo que f(x)=1/x² é contínua?Melhor,como determinar
 o delta apropriado?
 
 Grato por qualquer ajuda.
 
 Eder
 
Oi, Eder:

Devemos ter cuidado pra definir f, pois seu dominio nao contem x = 0.

Seja  a  0. Temos que provar que lim(x - a) 1/x = 1/a.


Seja eps  0.

Como a  0, teremos |a|  |a|/2  0

Tomemos delta = min( a^2*eps/2, |a|/2 )

|x - a|  delta ==

a - delta  x  a + delta ==

a - |a|/2  x  a + |a|/2 ==

se a  0, entao 3a/2  x  a/2
e
se a  0, entao a/2  x  3a/2 ==

de qualquer jeito, |x|  |a|/2 == 1/|x|  1/(|a|/2)

Assim:
|1/x - 1/a| = |x - a|/(|a||x|)  delta/(|a||a|/2) = 2delta/a^2 = eps


Um abraco,
Claudio.


PS: Acabei nao respondendo a sua pergunta. O delta apropriado voce acha
resolvendo o problema de tras pra frente, ou seja, fazendo:

|1/x - 1/a| = |x - a|/(|a||x|)  delta/(|a||x|) = eps ==

delta = eps*|a|*|x|

A partir desse ponto, voce soh precisa achar um limitante inferior para |x|
(no caso, eu achei |a|/2).



=
Instruções para entrar na lista, sair da lista e usar a lista em
http://www.mat.puc-rio.br/~nicolau/olimp/obm-l.html
=


[obm-l] Moedas em caixas

2003-08-14 Por tôpico Claudio Buffara
Title: Moedas em caixas



Oi, Alexandre:

Eu achei esse problema das moedas em caixas mais interessante do que o do no. de solucoes da equacao, onde a matematica legal acaba no momento em que voce estabelece a relacao entre o no. de solucoes de uma equacao e os coeficientes de um certo polinomio associado aquela equacao. Depois eh soh braco (ou de preferencia, transistor).

O das moedas eh uma aplicacao bem legal da representacao binaria de numeros naturais.

No caso de 10 caixas e 1000 moedas, a k-esima caixa tem que ter 2^(k-1) moedas, para k = 1, 2, ..., 9. Total das 9 primeiras caixas = 1 + 2 + 4 + ... + 128 + 256 = 511 moedas.
A 10a. caixa contera as 489 moedas restantes.

Usando as 9 primeiras caixas, voce consegue pegar qualquer numero de moedas entre 1 e 511 (inclusive) - pra ver isso basta reparar que qualquer numero tem uma (unica) representacao binaria (ou seja, como soma de potencias de 2).

Pra pegar 512 ou mais moedas, voce usa a 10. caixa e depois uma combinacao das 9 primeiras que contenha de 1 a 511 moedas. Por exemplo, 836 = 489 + 256 + 64 + 16 + 8 + 2 + 1 = 489 + 2^(9-1) + 2^(7-1) + 2^(5-1) + 2^(4-1) + 2^(2-1) + 2^(1-1) == pra pegar 836 moedas voce vai usar as caixas 1, 2, 4, 5, 7, 9 e 10.

O problema geral eh o seguinte:
1) Prove que com caixas contendo 1, 2, 4, ..., 2^(n-1) moedas, voce consegue pegar um numero qualquer de moedas entre 1 e 2^n - 1 (inclusive).
2) Prove que essa eh a unica forma de distribuir 2^n - 1 moedas por n caixas em que isso eh possivel.

Um abraco,
Claudio.


on 10.08.03 00:26, Alexandre Daibert at [EMAIL PROTECTED] wrote:

Não, isto não caiu em vestibular nenhum, eu cheguei nisto no meio de um problema, q eh o seguinte:
Tendo 10 caixas e 1000 moedas, colocar as caixas nas moedas de modo q qualquer quantidade de 1 a 1000 moedas possam ser pegas de modo a não abrir nenhuma caixa. Se não me engano este problema eh do homem q calculava. tem uma solução mais usual para ele q eh ir colocando 1, 2, 4, 8, ... moedas em cada caixa, e no fim as q sobrarem colocar na última caixa. 

*** De uma checada nas contas: 1+2+4+8+...+256 = 511 == sobram 489 moedas.
Esse 23 deve vir do fato de que 2^10 - 1 = 1023 e de que existem apenas 1000 moedas, ou seja, faltam 23 moedas pra completar uma eventual 11a. caixa, que conteria 2^9 = 512 moedas (489+23=512).

Sobram 23 moedas, mas elas não precisam ser colocadas necessariamente apenas na última caixa. Se vc pensar em cima do problema vc chega q o número de soluções do problema é o número de soluções inteiras não negativas da equação: 16a + 8b + 4c + 2d + e = 23. Claro q isto vai além do q se esperava q a pessoa fizesse no problema. Cheguei a este resultado e quis, por curiosidade, saber como calcular o número de soluções deste tipo de equação, visto q o cálculo no braço seria muito trabalhoso. Pensei q houvesse alguma solução por análise combinatória deste problema, porém mais avançada q a resolução clássica da equação a + b + c + d = 10 por exemplo. Mas pelo que eu entendi, este tipo de problema, pelo q vimos ateh aki, mesmo com problemas menores, ou vc calcula todas as soluções no braço mesmo ou joga em um computador. Não há método matemático q seja pouco trabalhoso. Mas mesmo assim gostaria de agradecer imensamente ao colega, pois as suas explicações contribuiram muito para mim. :-) 

Se alguém quiser a minha resolução deste problema das caixas e como eu cheguei a isso, depois me dê um toque q colocarei minha resolução aki com paciência

Alexandre Daibert








  1   2   3   >